Радиус Π²Π΅ΠΊΡ‚ΠΎΡ€Π° Ρ„ΠΎΡ€ΠΌΡƒΠ»Π°: 1.1. Радиус-Π²Π΅ΠΊΡ‚ΠΎΡ€ ΠΌΠ°Ρ‚Π΅Ρ€ΠΈΠ°Π»ΡŒΠ½ΠΎΠΉ Ρ‚ΠΎΡ‡ΠΊΠΈ – Π›Π΅ΠΊΡ†ΠΈΠΈ ΠΏΠΎ Ρ„ΠΈΠ·ΠΈΠΊΠ΅

Π‘ΠΎΠ΄Π΅Ρ€ΠΆΠ°Π½ΠΈΠ΅

Бвязь ΠΌΠ΅ΠΆΠ΄Ρƒ ΠΊΠΎΠΎΡ€Π΄ΠΈΠ½Π°Ρ‚Π°ΠΌΠΈ Π²Π΅ΠΊΡ‚ΠΎΡ€Π° ΠΈ ΠΊΠΎΠΎΡ€Π΄ΠΈΠ½Π°Ρ‚Π°ΠΌΠΈ Π΅Π³ΠΎ Π½Π°Ρ‡Π°Π»Π° ΠΈ ΠΊΠΎΠ½Ρ†Π° 9 класс ΠΎΠ½Π»Π°ΠΉΠ½-ΠΏΠΎΠ΄Π³ΠΎΡ‚ΠΎΠ²ΠΊΠ° Π½Π° РостСлСком Π›ΠΈΡ†Π΅ΠΉ |

Π’Π΅ΠΌΠ° 27.

Бвязь ΠΌΠ΅ΠΆΠ΄Ρƒ ΠΊΠΎΠΎΡ€Π΄ΠΈΠ½Π°Ρ‚Π°ΠΌΠΈ Π²Π΅ΠΊΡ‚ΠΎΡ€Π° ΠΈ ΠΊΠΎΠΎΡ€Π΄ΠΈΠ½Π°Ρ‚Π°ΠΌΠΈ Π΅Π³ΠΎ Π½Π°Ρ‡Π°Π»Π° ΠΈ ΠΊΠΎΠ½Ρ†Π°.

Π’Ρ‹ ΡƒΠΆΠ΅ Π·Π½Π°ΠΊΠΎΠΌ с понятиСм ΠΊΠΎΠΎΡ€Π΄ΠΈΠ½Π°Ρ‚ Π²Π΅ΠΊΡ‚ΠΎΡ€Π°. Ими Π½Π°Π·Ρ‹Π²Π°ΡŽΡ‚ коэффициСнты разлоТСния Π΄Π°Π½Π½ΠΎΠ³ΠΎ Π²Π΅ΠΊΡ‚ΠΎΡ€Π° ΠΏΠΎ Π΅Π΄ΠΈΠ½ΠΈΡ‡Π½Ρ‹ΠΌ ΠΊΠΎΠΎΡ€Π΄ΠΈΠ½Π°Ρ‚Π½Ρ‹ΠΌ Π²Π΅ΠΊΡ‚ΠΎΡ€Π°ΠΌ iβƒ— ΠΈ jβƒ—.

БСгодня ΠΌΡ‹ ΠΎΡ‚Π²Π΅Ρ‚ΠΈΠΌ Π½Π° вопрос «Как связаны ΠΊΠΎΠΎΡ€Π΄ΠΈΠ½Π°Ρ‚Ρ‹ Π²Π΅ΠΊΡ‚ΠΎΡ€Π° с ΠΊΠΎΠΎΡ€Π΄ΠΈΠ½Π°Ρ‚Π°ΠΌΠΈ Π΅Π³ΠΎ Π½Π°Ρ‡Π°Π»Π° ΠΈ ΠΊΠΎΠ½Ρ†Π°?Β».

Но для Π½Π°Ρ‡Π°Π»Π° вСрнёмся ΠΊ ΠΊΠΎΠΎΡ€Π΄ΠΈΠ½Π°Ρ‚Π°ΠΌ Ρ‚ΠΎΡ‡ΠΊΠΈ Π² ΠΏΡ€ΡΠΌΠΎΡƒΠ³ΠΎΠ»ΡŒΠ½ΠΎΠΉ систСмС ΠΊΠΎΠΎΡ€Π΄ΠΈΠ½Π°Ρ‚.

Напомним, Ρ‡Ρ‚ΠΎ для ΠΈΡ… опрСдСлСния Π½ΡƒΠΆΠ½ΠΎ ΠΎΠΏΡƒΡΡ‚ΠΈΡ‚ΡŒ пСрпСндикуляры ΠΈΠ· Π΄Π°Π½Π½ΠΎΠΉ Ρ‚ΠΎΡ‡ΠΊΠΈ ΠΊ осям.

Π’ΠΎΡ‡ΠΊΠΈ пСрСсСчСния Π΄Π°Π½Π½Ρ‹Ρ… прямых с осями ΠΎΠ±ΠΎΠ·Π½Π°Ρ‡ΠΈΠΌ ΠΊΠ°ΠΊ M1 ΠΈ M2.

Абсциссой Ρ‚ΠΎΡ‡ΠΊΠΈ М являСтся число x, ΠΊΠΎΡ‚ΠΎΡ€ΠΎΠ΅ являСтся Π΄Π»ΠΈΠ½ΠΎΠΉ ΠΎΡ‚Ρ€Π΅Π·ΠΊΠ° OM1. А ΠΎΡ€Π΄ΠΈΠ½Π°Ρ‚ΠΎΠΉ β€” число y, ΠΊΠΎΡ‚ΠΎΡ€ΠΎΠ΅ являСтся Π΄Π»ΠΈΠ½ΠΎΠΉ ΠΎΡ‚Ρ€Π΅Π·ΠΊΠ° OM2.

M(x; y) x = OM1, y = OM2

ΠœΡ‹ вспомнили, ΠΊΠ°ΠΊ ΠΎΠΏΡ€Π΅Π΄Π΅Π»ΡΡ‚ΡŒ ΠΊΠΎΠΎΡ€Π΄ΠΈΠ½Π°Ρ‚Ρ‹ Ρ‚ΠΎΡ‡Π΅ΠΊ, Π° Ρ‚Π΅ΠΏΠ΅Ρ€ΡŒ вСрнёмся ΠΊ ΠΎΠ±Ρ‰Π΅ΠΌΡƒ ΡΠ»ΡƒΡ‡Π°ΡŽ ΠΈ, ΡƒΠΆΠ΅ рассмотрСнной, Ρ‚ΠΎΡ‡ΠΊΠ΅ M.

ΠŸΡ€ΠΎΠ²Π΅Π΄Ρ‘ΠΌ Π²Π΅ΠΊΡ‚ΠΎΡ€ ΠΈΠ· Ρ‚ΠΎΡ‡ΠΊΠΈ O ΠΊ Ρ‚ΠΎΡ‡ΠΊΠ΅ M. Π—Π°ΠΏΠΎΠΌΠ½ΠΈ, Π²Π΅ΠΊΡ‚ΠΎΡ€ OMβƒ— Π½Π°Π·Ρ‹Π²Π°ΡŽΡ‚ радиус-Π²Π΅ΠΊΡ‚ΠΎΡ€ΠΎΠΌ Ρ‚ΠΎΡ‡ΠΊΠΈ M.

БСйчас Π΄ΠΎΠΊΠ°ΠΆΠ΅ΠΌ ΡΠ»Π΅Π΄ΡƒΡŽΡ‰Π΅Π΅ ΡƒΡ‚Π²Π΅Ρ€ΠΆΠ΄Π΅Π½ΠΈΠ΅: ΠΊΠΎΠΎΡ€Π΄ΠΈΠ½Π°Ρ‚Ρ‹ Ρ‚ΠΎΡ‡ΠΊΠΈ M Ρ€Π°Π²Π½Ρ‹ ΡΠΎΠΎΡ‚Π²Π΅Ρ‚ΡΡ‚Π²ΡƒΡŽΡ‰ΠΈΠΌ ΠΊΠΎΠΎΡ€Π΄ΠΈΠ½Π°Ρ‚Π°ΠΌ Π΅Ρ‘ радиус-Π²Π΅ΠΊΡ‚ΠΎΡ€Π°.

Π”ΠΎΠΊΠ°Π·Π°Ρ‚ΡŒ: M(x;y)=OMβƒ—x;y

ΠŸΠΎΠ½ΡΡ‚Π½ΠΎ, Ρ‡Ρ‚ΠΎ Π²Π΅ΠΊΡ‚ΠΎΡ€ OMβƒ—=ОM1βƒ—+ОM2βƒ— ΠΏΠΎ ΠΏΡ€Π°Π²ΠΈΠ»Ρƒ ΠΏΠ°Ρ€Π°Π»Π»Π΅Π»ΠΎΠ³Ρ€Π°ΠΌΠΌΠ°.

Π’Π΅ΠΏΠ΅Ρ€ΡŒ Π½Π΅ΠΎΠ±Ρ…ΠΎΠ΄ΠΈΠΌΠΎ Π΄ΠΎΠΊΠ°Π·Π°Ρ‚ΡŒ, Ρ‡Ρ‚ΠΎ Π²Π΅ΠΊΡ‚ΠΎΡ€

OM1⃗=xi⃗, а вСктор OM2⃗=yj⃗

Π’Π΅ΠΌ самым ΠΌΡ‹ Π΄ΠΎΠΊΠ°ΠΆΠ΅ΠΌ, Ρ‡Ρ‚ΠΎ Π²Π΅ΠΊΡ‚ΠΎΡ€ OMβƒ—x;y.

Если x > 0, Ρ‚ΠΎ x = OM1, Π° Π²Π΅ΠΊΡ‚ΠΎΡ€Ρ‹ OM1βƒ— ΠΈ iβƒ— сонаправлСны, поэтому

OM1βƒ—=OM1βˆ™iβƒ—=xiβƒ—

Если x x = OM1, Π° Π²Π΅ΠΊΡ‚ΠΎΡ€Ρ‹ OM1βƒ— ΠΈ iβƒ— ΠΏΡ€ΠΎΡ‚ΠΈΠ²ΠΎΠΏΠΎΠ»ΠΎΠΆΠ½ΠΎ Π½Π°ΠΏΡ€Π°Π²Π»Π΅Π½Ρ‹.

ΠŸΠΎΡΡ‚ΠΎΠΌΡƒ OM1βƒ—=-OM1βˆ™iβƒ—=xiβƒ—.

НаконСц, Ссли x = 0

OM1βƒ—=0βƒ— ΠΈ равСнство OM1=xiβƒ— Π² этом случаС Ρ‚Π°ΠΊ ΠΆΠ΅ справСдливо. Π’Π°ΠΊΠΈΠΌ ΠΎΠ±Ρ€Π°Π·ΠΎΠΌ, Π² любом случаС ОM1βƒ—=xiβƒ—. Аналогично доказываСтся, Ρ‡Ρ‚ΠΎ ОM2βƒ—=yjβƒ—.

Π‘Π»Π΅Π΄ΠΎΠ²Π°Ρ‚Π΅Π»ΡŒΠ½ΠΎ,OMβƒ—=ОM1βƒ—+ОM2βƒ—=xiβƒ—+yjβƒ—

ΠžΡ‚ΡΡŽΠ΄Π° слСдуСт, Ρ‡Ρ‚ΠΎ ΠΊΠΎΠΎΡ€Π΄ΠΈΠ½Π°Ρ‚Ρ‹ радиус-Π²Π΅ΠΊΡ‚ΠΎΡ€Π° OM Ρ€Π°Π²Π½Ρ‹ (x; y), Ρ‚ΠΎ Π΅ΡΡ‚ΡŒ Ρ€Π°Π²Π½Ρ‹ ΡΠΎΠΎΡ‚Π²Π΅Ρ‚ΡΡ‚Π²ΡƒΡŽΡ‰ΠΈΠΌ ΠΊΠΎΠΎΡ€Π΄ΠΈΠ½Π°Ρ‚Π°ΠΌ Ρ‚ΠΎΡ‡ΠΊΠΈ M

.

ΠŸΠΎΠ»ΡŒΠ·ΡƒΡΡΡŒ Π΄ΠΎΠΊΠ°Π·Π°Π½Π½Ρ‹ΠΌ ΡƒΡ‚Π²Π΅Ρ€ΠΆΠ΄Π΅Π½ΠΈΠ΅ΠΌ, Π²Ρ‹Ρ€Π°Π·ΠΈΠΌ ΠΊΠΎΠΎΡ€Π΄ΠΈΠ½Π°Ρ‚Ρ‹ Π²Π΅ΠΊΡ‚ΠΎΡ€Π° ABβƒ— Ρ‡Π΅Ρ€Π΅Π· ΠΊΠΎΠΎΡ€Π΄ΠΈΠ½Π°Ρ‚Ρ‹ Π΅Π³ΠΎ Π½Π°Ρ‡Π°Π»Π° A ΠΈ ΠΊΠΎΠ½Ρ†Π° B. ΠŸΡƒΡΡ‚ΡŒ Ρ‚ΠΎΡ‡ΠΊΠ° A ΠΈΠΌΠ΅Π΅Ρ‚ ΠΊΠΎΠΎΡ€Π΄ΠΈΠ½Π°Ρ‚Ρ‹ x1;y1, Π° Ρ‚ΠΎΡ‡ΠΊΠ° B – ΠΊΠΎΠΎΡ€Π΄ΠΈΠ½Π°Ρ‚Ρ‹ x2;y2.

Π’Π΅ΠΊΡ‚ΠΎΡ€ ABβƒ— Ρ€Π°Π²Π΅Π½ разности Π²Π΅ΠΊΡ‚ΠΎΡ€ΠΎΠ² OBβƒ— ΠΈ OAβƒ—, поэтому Π΅Π³ΠΎ ΠΊΠΎΠΎΡ€Π΄ΠΈΠ½Π°Ρ‚Ρ‹ Ρ€Π°Π²Π½Ρ‹ разностям ΡΠΎΠΎΡ‚Π²Π΅Ρ‚ΡΡ‚Π²ΡƒΡŽΡ‰ΠΈΡ… ΠΊΠΎΠΎΡ€Π΄ΠΈΠ½Π°Ρ‚ Π²Π΅ΠΊΡ‚ΠΎΡ€ΠΎΠ² OBβƒ— ΠΈ OAβƒ—. Но OBβƒ— ΠΈ OAβƒ— – радиус-Π²Π΅ΠΊΡ‚ΠΎΡ€Ρ‹ Ρ‚ΠΎΡ‡Π΅ΠΊ B ΠΈ A, ΠΈ, Π·Π½Π°Ρ‡ΠΈΡ‚, OBβƒ— ΠΈΠΌΠ΅Π΅Ρ‚ ΠΊΠΎΠΎΡ€Π΄ΠΈΠ½Π°Ρ‚Ρ‹ x2;y2, Π° OAβƒ— ΠΈΠΌΠ΅Π΅Ρ‚ ΠΊΠΎΠΎΡ€Π΄ΠΈΠ½Π°Ρ‚Ρ‹ x1;y1. Π‘Π»Π΅Π΄ΠΎΠ²Π°Ρ‚Π΅Π»ΡŒΠ½ΠΎ, Π²Π΅ΠΊΡ‚ΠΎΡ€ ABβƒ— ΠΈΠΌΠ΅Π΅Ρ‚ ΠΊΠΎΠΎΡ€Π΄ΠΈΠ½Π°Ρ‚Ρ‹ x2-x1;y2-y1.

Π’Π°ΠΊΠΈΠΌ ΠΎΠ±Ρ€Π°Π·ΠΎΠΌ, каТдая ΠΊΠΎΠΎΡ€Π΄ΠΈΠ½Π°Ρ‚Π° Π²Π΅ΠΊΡ‚ΠΎΡ€Π° Ρ€Π°Π²Π½Π° разности ΡΠΎΠΎΡ‚Π²Π΅Ρ‚ΡΡ‚Π²ΡƒΡŽΡ‰ΠΈΡ… ΠΊΠΎΠΎΡ€Π΄ΠΈΠ½Π°Ρ‚ Π΅Π³ΠΎ ΠΊΠΎΠ½Ρ†Π° ΠΈ Π½Π°Ρ‡Π°Π»Π°.

Π’Π²Π΅Π΄Π΅Π½ΠΈΠ΅ систСмы ΠΊΠΎΠΎΡ€Π΄ΠΈΠ½Π°Ρ‚ Π΄Π°Π΅Ρ‚ Π²ΠΎΠ·ΠΌΠΎΠΆΠ½ΠΎΡΡ‚ΡŒ ΠΈΠ·ΡƒΡ‡Π°Ρ‚ΡŒ гСомСтричСскиС Ρ„ΠΈΠ³ΡƒΡ€Ρ‹ ΠΈ ΠΈΡ… свойства с ΠΏΠΎΠΌΠΎΡ‰ΡŒΡŽ ΡƒΡ€Π°Π²Π½Π΅Π½ΠΈΠΉ ΠΈ нСравСнств ΠΈ, Ρ‚Π°ΠΊΠΈΠΌ ΠΎΠ±Ρ€Π°Π·ΠΎΠΌ, ΠΈΡΠΏΠΎΠ»ΡŒΠ·ΠΎΠ²Π°Ρ‚ΡŒ Π² Π³Π΅ΠΎΠΌΠ΅Ρ‚Ρ€ΠΈΠΈ ΠΌΠ΅Ρ‚ΠΎΠ΄Ρ‹ Π°Π»Π³Π΅Π±Ρ€Ρ‹. Π’Π°ΠΊΠΎΠΉ ΠΏΠΎΠ΄Ρ…ΠΎΠ΄ ΠΊ ΠΈΠ·ΡƒΡ‡Π΅Π½ΠΈΡŽ свойств гСомСтричСских Ρ„ΠΈΠ³ΡƒΡ€ называСтся ΠΌΠ΅Ρ‚ΠΎΠ΄ΠΎΠΌ ΠΊΠΎΠΎΡ€Π΄ΠΈΠ½Π°Ρ‚.

Рассмотрим Ρ‚Ρ€ΠΈ Π²ΡΠΏΠΎΠΌΠΎΠ³Π°Ρ‚Π΅Π»ΡŒΠ½Ρ‹Π΅ Π·Π°Π΄Π°Ρ‡ΠΈ:

  1. Как Π½Π°ΠΉΡ‚ΠΈ ΠΊΠΎΠΎΡ€Π΄ΠΈΠ½Π°Ρ‚Ρ‹ сСрСдины ΠΎΡ‚Ρ€Π΅Π·ΠΊΠ°.

    ΠŸΡƒΡΡ‚ΡŒ Π² систСмС ΠΊΠΎΠΎΡ€Π΄ΠΈΠ½Π°Ρ‚ Oxy Ρ‚ΠΎΡ‡ΠΊΠ° A ΠΈΠΌΠ΅Π΅Ρ‚ ΠΊΠΎΠΎΡ€Π΄ΠΈΠ½Π°Ρ‚Ρ‹ x1;y1, Π° Ρ‚ΠΎΡ‡ΠΊΠ° B – ΠΊΠΎΠΎΡ€Π΄ΠΈΠ½Π°Ρ‚Ρ‹ x2;y2. Π’Ρ‹Ρ€Π°Π·ΠΈΠΌ ΠΊΠΎΠΎΡ€Π΄ΠΈΠ½Π°Ρ‚Ρ‹ x;y сСрСдины C ΠΎΡ‚Ρ€Π΅Π·ΠΊΠ° AB Ρ‡Π΅Ρ€Π΅Π· ΠΊΠΎΠΎΡ€Π΄ΠΈΠ½Π°Ρ‚Ρ‹ Π΅Π³ΠΎ ΠΊΠΎΠ½Ρ†ΠΎΠ². Π’Π°ΠΊ ΠΊΠ°ΠΊ Ρ‚ΠΎΡ‡ΠΊΠ° C – сСрСдина ΠΎΡ‚Ρ€Π΅Π·ΠΊΠ°

    AB, Ρ‚ΠΎ

    OC⃗=12OA⃗+OB⃗.

    x=x1+x22; y=y1+y22

    Π’Π°ΠΊΠΈΠΌ ΠΎΠ±Ρ€Π°Π·ΠΎΠΌ, каТдая ΠΊΠΎΠΎΡ€Π΄ΠΈΠ½Π°Ρ‚Π° сСрСдины ΠΎΡ‚Ρ€Π΅Π·ΠΊΠ° Ρ€Π°Π²Π½Π° полусуммС ΡΠΎΠΎΡ‚Π²Π΅Ρ‚ΡΡ‚Π²ΡƒΡŽΡ‰ΠΈΡ… ΠΊΠΎΠΎΡ€Π΄ΠΈΠ½Π°Ρ‚ Π΅Π³ΠΎ ΠΊΠΎΠ½Ρ†ΠΎΠ².

  2. ВычислСниС Π΄Π»ΠΈΠ½Ρ‹ Π²Π΅ΠΊΡ‚ΠΎΡ€Π° ΠΏΠΎ Π΅Π³ΠΎ ΠΊΠΎΠΎΡ€Π΄ΠΈΠ½Π°Ρ‚Π°ΠΌ.

    ΠŸΡƒΡΡ‚ΡŒ Π²Π΅ΠΊΡ‚ΠΎΡ€ aβƒ—x;y, Ρ‚ΠΎΠ³Π΄Π° Π΄Π»ΠΈΠ½Π° Π²Π΅ΠΊΡ‚ΠΎΡ€Π° вычисляСтся ΠΏΠΎ Ρ„ΠΎΡ€ΠΌΡƒΠ»Π΅:

    a⃗=x2+y2

  3. ВычислСниС расстояния ΠΌΠ΅ΠΆΠ΄Ρƒ двумя Ρ‚ΠΎΡ‡ΠΊΠ°ΠΌΠΈ. ΠŸΡƒΡΡ‚ΡŒ Ρ‚ΠΎΡ‡ΠΊΠ° M1 ΠΈΠΌΠ΅Π΅Ρ‚ ΠΊΠΎΠΎΡ€Π΄ΠΈΠ½Π°Ρ‚Ρ‹ (x1; y1), Ρ‚ΠΎΡ‡ΠΊΠ°

    M2 – ΠΊΠΎΠΎΡ€Π΄ΠΈΠ½Π°Ρ‚Ρ‹ (x2; y2). Π’Ρ‹Ρ€Π°Π·ΠΈΠΌ расстояниС d ΠΌΠ΅ΠΆΠ΄Ρƒ Ρ‚ΠΎΡ‡ΠΊΠ°ΠΌΠΈ M1 ΠΈ M2 Ρ‡Π΅Ρ€Π΅Π· ΠΈΡ… ΠΊΠΎΠΎΡ€Π΄ΠΈΠ½Π°Ρ‚Ρ‹.

Рассмотрим Π²Π΅ΠΊΡ‚ΠΎΡ€ M1M2βƒ—. Π•Π³ΠΎ ΠΊΠΎΠΎΡ€Π΄ΠΈΠ½Π°Ρ‚Ρ‹ Ρ€Π°Π²Π½Ρ‹ x2-x1;y2-y1. Π‘Π»Π΅Π΄ΠΎΠ²Π°Ρ‚Π΅Π»ΡŒΠ½ΠΎ, Π΄Π»ΠΈΠ½Π° этого Π²Π΅ΠΊΡ‚ΠΎΡ€Π° ΠΌΠΎΠΆΠ΅Ρ‚ Π±Ρ‹Ρ‚ΡŒ Π½Π°ΠΉΠ΄Π΅Π½Π° ΠΏΠΎ Ρ„ΠΎΡ€ΠΌΡƒΠ»Π΅:

M1M2βƒ—=x2-x12+y2-y12

ΠŸΡ€ΠΈΠΌΠ΅Ρ€:

  1. Найти Π΄Π»ΠΈΠ½Ρƒ Π²Π΅ΠΊΡ‚ΠΎΡ€Π° aβƒ—-3;4

    a⃗=x2+y2=-32+42=25=5

    ΠžΡ‚Π²Π΅Ρ‚: 5

  2. Найти расстояниС ΠΌΠ΅ΠΆΠ΄Ρƒ Ρ‚ΠΎΡ‡ΠΊΠΎΠΉ A(2; 7) ΠΈ Ρ‚ΠΎΡ‡ΠΊΠΎΠΉ B(-2; 7)

    d=-2-22+7-72=16=4

    ΠžΡ‚Π²Π΅Ρ‚: 4

Радиус-Π²Π΅ΠΊΡ‚ΠΎΡ€ ΠΈΠ½Π΅Ρ€Ρ†ΠΈΠΈ – ЭнциклопСдия ΠΏΠΎ ΠΌΠ°ΡˆΠΈΠ½ΠΎΡΡ‚Ρ€ΠΎΠ΅Π½ΠΈΡŽ XXL

Из Ρ„ΠΎΡ€ΠΌΡƒΠ»Ρ‹ (6. 22) слСдуСт, Ρ‡Ρ‚ΠΎ Π³Π»Π°Π²Π½Ρ‹ΠΉ Π²Π΅ΠΊΡ‚ΠΎΡ€ сил ΠΈΠ½Π΅Ρ€Ρ†ΠΈΠΈ Ρ€ΠΎΡ‚ΠΎΡ€Π° пСрпСндикулярСн Π΅Π³ΠΎ оси вращСния Ρƒ, Ρ‚. Π΅. располоТСн Π² плоскости 0x2, пСрпСндикулярной ΠΊ ΡƒΠΊΠ°Π·Π°Π½Π½ΠΎΠΉ оси. ΠžΠ±ΠΎΠ·Π½Π°Ρ‡Π°Ρ Ρ‡Π΅Ρ€Π΅Π· радиус-Π²Π΅ΠΊΡ‚ΠΎΡ€ частицы (этот Π²Π΅ΠΊΡ‚ΠΎΡ€ Π½Π° рисункС Π½Π΅ ΠΏΠΎΠΊΠ°Π·Π°Π½), Π½Π°Ρ…ΠΎΠ΄ΠΈΠΌ ΠΌΠΎΠΌΠ΅Π½Ρ‚ силы ΠΈΠ½Π΅Ρ€Ρ†ΠΈΠΈ частицы ΠΎΡ‚Π½ΠΎΡΠΈΡ‚Π΅Π»ΡŒΠ½ΠΎ Π²Ρ‹Π±Ρ€Π°Π½Π½ΠΎΠ³ΠΎ Ρ†Π΅Π½Ρ‚Ρ€Π° О  [c.97]

Π—Π°Π΄Π°Ρ‡Π° динамичСской балансировки Π·Π°ΠΊΠ»ΡŽΡ‡Π°Π΅Ρ‚ΡΡ Π² ΠΏΠΎΠ΄Π±ΠΎΡ€Π΅ масс Ρ‚ ΠΈ Ρ‚ ΠΈ противовСсов ΠΈ ΠΈΡ… радиус-Π²Π΅ΠΊΡ‚ΠΎΡ€ΠΎΠ² pj ΠΈ Ρ€Π¬ Π² плоскостях I Vi II Ρ‚Π°ΠΊΠΈΠΌ ΠΎΠ±Ρ€Π°Π·ΠΎΠΌ, Ρ‡Ρ‚ΠΎΠ±Ρ‹ Ρ†Π΅Π½Ρ‚Ρ€ΠΎΠ±Π΅ΠΆΠ½Ρ‹Π΅ силы ΠΈΠ½Π΅Ρ€Ρ†ΠΈΠΈ этих масс  

[c.99]


Π¦Π΅Π½Ρ‚Ρ€ΠΎΠΌ ΠΈΠ½Π΅Ρ€Ρ†ΠΈΠΈ систСмы называСтся гСомСтричСская Ρ‚ΠΎΡ‡ΠΊΠ° Π‘ пространства, опрСдСляСмая радиусом-Π²Π΅ΠΊΡ‚ΠΎΡ€ΠΎΠΌ  [c.70]

Рассмотрим эллипсоид ΠΈΠ½Π΅Ρ€Ρ†ΠΈΠΈ, построСнный для Π½Π΅ΠΏΠΎΠ΄Π²ΠΈΠΆΠ½ΠΎΠΉ Ρ‚ΠΎΡ‡ΠΊΠΈ О (рис. V.10). НазовСм ΠΌΠ³Π½ΠΎΠ²Π΅Π½Π½Ρ‹ΠΌ полюсом Π  Ρ‚ΠΎΡ‡ΠΊΡƒ, Π² ΠΊΠΎΡ‚ΠΎΡ€ΠΎΠΉ мгновСнная ось пСрСсСкаСт этот эллипсоид ΠΈΠ½Π΅Ρ€Ρ†ΠΈΠΈ, ΠΎΠ±ΠΎΠ·Π½Π°Ρ‡ΠΈΠΌ Ρ‡Π΅Ρ€Π΅Π· Π“Ρ€ радиус-Π²Π΅ΠΊΡ‚ΠΎΡ€ Ρ‚ΠΎΡ‡ΠΊΠΈ Π  ΠΈ ΠΏΠΎΠ»ΠΎΠΆΠΈΠΌ  [c.198]

Π—Π΄Π΅ΡΡŒ Π·Π° Π½Π΅Π·Π°Π²ΠΈΡΠΈΠΌΡƒΡŽ ΠΏΠ΅Ρ€Π΅ΠΌΠ΅Π½Π½ΡƒΡŽ принято Π±Π΅Π·Ρ€Π°Π·ΠΌΠ΅Ρ€Π½ΠΎΠ΅ врСмя Ρ‚ = Π¨ΠΎΠ—, Π³Π΄Π΅ ΠΈ>ΠΎ β€” угловая ΡΠΊΠΎΡ€ΠΎΡΡ‚ΡŒ ΠΎΡ€Π±ΠΈΡ‚Π°Π»ΡŒΠ½ΠΎΡŽ двиТСния Ρ†Π΅Π½Ρ‚Ρ€Π° масс систСмы О.

Π’ уравнСниях (2.25) Π²Π²Π΅Π΄Π΅Π½Ρ‹ ΡΠ»Π΅Π΄ΡƒΡŽΡ‰ΠΈΠ΅ обозначСния 1, 2 – ΡƒΠ³Π»Ρ‹, ΠΎΠΏΡ€Π΅Π΄Π΅Π»ΡΡŽΡ‰ΠΈΠ΅ отклонСния спутника ΠΈ стабилизатора ΠΎΡ‚ направлСния радиуса-Π²Π΅ΠΊΡ‚ΠΎΡ€Π° К Ρ†Π΅Π½Ρ‚Ρ€Π° масс систСмы Ρƒ4Ρ† Π™/. 0 = 1,2) β€” Π³Π»Π°Π²Π½Ρ‹Π΅ Ρ†Π΅Π½Ρ‚Ρ€Π°Π»ΡŒΠ½Ρ‹Π΅ ΠΌΠΎΠΌΠ΅Π½Ρ‚Ρ‹ ΠΈΠ½Π΅Ρ€Ρ†ΠΈΠΈ Ρ‚Π΅Π» Π²Π΅Π»ΠΈΡ‡ΠΈΠ½Ρ‹ ΠΈ 2 Ρ…Π°Ρ€Π°ΠΊΡ‚Π΅Ρ€ΠΈΠ·ΡƒΡŽΡ‚ Π²ΡΠ·ΠΊΠΎΡΡ‚ΡŒ ΠΈ ΡƒΠΏΡ€ΡƒΠ³ΠΎΡΡ‚ΡŒ подвСса.  [c.91]

Радиус-Π²Π΅ΠΊΡ‚ΠΎΡ€ ΠΈ всС Π΄Π»ΠΈΠ½Ρ‹ Π² эллипсоидС ΠΈΠ½Π΅Ρ€Ρ†ΠΈΠΈ Коши ΠΈΠΌΠ΅ΡŽΡ‚, Ρ€Π°Π·ΠΌΠ΅Ρ€Π½ΠΎΡΡ‚ΡŒΡŽ Π²Π΅Π»ΠΈΡ‡ΠΈΠ½Ρƒ, ΠΎΠ±Ρ€Π°Ρ‚Π½ΡƒΡŽ ΠΊΠ²Π°Π΄Ρ€Π°Ρ‚Π½ΠΎΠΌΡƒ ΠΊΠΎΡ€Π½ΡŽ ΠΈΠ· размСрности ΠΌΠΎΠΌΠ΅Π½Ρ‚Π° ΠΈΠ½Π΅Ρ€Ρ†ΠΈΠΈ, Ρ‡Ρ‚ΠΎ вносит ряд ослоТнСний, особСнно Π² графичСскиС построСния. Π—Π½Π°Ρ‡ΠΈΡ‚Π΅Π»ΡŒΠ½ΠΎ ΡƒΠ΄ΠΎΠ±Π½Π΅Π΅ ΠΎΡ‚ΠΊΠ»Π°Π΄Ρ‹Π²Π°Ρ‚ΡŒ вдоль  [c.341]

Радиус-Π²Π΅ΠΊΡ‚ΠΎΡ€ 124 Радиус ΠΈΠ½Π΅Ρ€Ρ†ΠΈΠΈ 337 Π Π°Π·Π»ΠΎΠΆΠ΅Π½ΠΈΠ΅ двиТСния Ρ‚ΠΎΡ‡ΠΊΠΈ 130 Π Π°Π·Π»ΠΎΠΆΠ΅Π½ΠΈΠ΅ силы Π½Π° ΡΠΎΡΡ‚Π°Π²Π»ΡΡŽΡ‰ΠΈΠ΅ 37,  

[c.455]

Радиус-Π²Π΅ΠΊΡ‚ΠΎΡ€ 17 Радиус ΠΈΠ½Π΅Ρ€Ρ†ΠΈΠΈ 107  [c.301]

ΠžΠ±ΠΎΠ·Π½Π°Ρ‡ΠΈΠΌ X радиус-Π²Π΅ΠΊΡ‚ΠΎΡ€ Ρ‚ΠΎΡ‡ΠΊΠΈ пСрСсСчСния Π³Π»Π°Π²Π½ΠΎΠΉ оси с эллипсоидом ΠΈΠ½Π΅Ρ€Ρ†ΠΈΠΈ. Π’ΠΎΠ³Π΄Π°, согласно ΠΎΠΏΡ€Π΅Π΄Π΅Π»Π΅Π½ΠΈΡŽ Π³Π»Π°Π²Π½ΠΎΠΉ оси ΠΈ Π² соотвСтствии с Ρ‚Π΅ΠΎΡ€Π΅ΠΌΠΎΠΉ 1.8.4, Π±ΡƒΠ΄Π΅ΠΌ ΠΈΠΌΠ΅Ρ‚ΡŒ  [c.49]

Но Π³ X Π΅ Π΅ΡΡ‚ΡŒ ΠΏΡ€ΠΎΠΈΠ·Π²Π΅Π΄Π΅Π½ΠΈΠ΅ модуля Π³ радиуса-Π²Π΅ΠΊΡ‚ΠΎΡ€Π° Π½Π° синус ΡƒΠ³Π»Π° ΠΌΠ΅ΠΆΠ΄Ρƒ Π³ ΠΈ Π΅, Ρ‡Ρ‚ΠΎ прСдставляСт собой расстояниС 0. ΠΌΠ΅ΠΆΠ΄Ρƒ осями. Как слСдствиС ΠΎΡ‚ΠΌΠ΅Ρ‚ΠΈΠΌ, Ρ‡Ρ‚ΠΎ ΠΌΠΎΠΌΠ΅Π½Ρ‚ ΠΈΠ½Π΅Ρ€Ρ†ΠΈΠΈ ΠΎΡ‚Π½ΠΎΡΠΈΡ‚Π΅Π»ΡŒΠ½ΠΎ оси, проходящСй Ρ‡Π΅Ρ€Π΅Π· Ρ†Π΅Π½Ρ‚Ρ€ масс, мСньшС ΠΌΠΎΠΌΠ΅Π½Ρ‚Π° ΠΈΠ½Π΅Ρ€Ρ†ΠΈΠΈ ΠΎΡ‚Π½ΠΎΡΠΈΡ‚Π΅Π»ΡŒΠ½ΠΎ любой Π΄Ρ€ΡƒΠ³ΠΎΠΉ ΠΏΠ°Ρ€Π°Π»Π»Π΅Π»ΡŒΠ½ΠΎΠΉ оси.  [c.53]

Π’Π΅ΠΎΡ€Π΅ΠΌΠ° 1.10.4. Если Π½Π°ΠΏΡ€Π°Π²Π»Π΅Π½ΠΈΠ΅ Π΅Π³ β€” Π³Π»Π°Π²Π½ΠΎΠ΅ для Ρ†Π΅Π½Ρ‚Ρ€Π°Π»ΡŒΠ½ΠΎΠ³ΠΎ эллипсоида ΠΈΠ½Π΅Ρ€Ρ†ΠΈΠΈ, Ρ‚ΠΎ ΠΎΠ½ΠΎ Π±ΡƒΠ΄Π΅Ρ‚ Π³Π»Π°Π²Π½Ρ‹ΠΌ ΠΈ для любой Ρ‚ΠΎΡ‡ΠΊΠΈ О, ΠΎΠΏΡ€Π΅Π΄Π΅Π»Π΅Π½Π½ΠΎΠΉ радиусом-Π²Π΅ΠΊΡ‚ΠΎΡ€ΠΎΠΌ Π³ = Π³Π²Π³ ΠΏΡ€ΠΈ ΠΏΡ€ΠΎΠΈΠ·Π²ΠΎΠ»ΡŒΠ½ΠΎΠΌ Π·Π½Π°Ρ‡Π΅Π½ΠΈΠΈ Π³. И Π½Π°ΠΎΠ±ΠΎΡ€ΠΎΡ‚, Ссли Π½Π°ΠΏΡ€Π°Π²Π»Π΅Π½ΠΈΠ΅ Π²Π³ Π½Π΅ Π±Ρ‹Π»ΠΎ Π³Π»Π°Π²Π½Ρ‹ΠΌ для Ρ†Π΅Π½Ρ‚Ρ€Π°Π»ΡŒΠ½ΠΎΠ³ΠΎ эллипсоида ΠΈΠ½Π΅Ρ€Ρ†ΠΈΠΈ, Ρ‚ΠΎ ΠΎΠ½ΠΎ Π½Π΅ ΠΌΠΎΠΆΠ΅Ρ‚ ΡΡ‚Π°Ρ‚ΡŒ Π³Π»Π°Π²Π½Ρ‹ΠΌ ΠΏΠΈ ΠΏΡ€ΠΈ ΠΊΠ°ΠΊΠΎΠΌ Π·Π½Π°Ρ‡Π΅Π½ΠΈΠΈ Π³.  

[c.55]

Π”ΠΎΠΊΠ°Π·Π°Ρ‚Π΅Π»ΡŒΡΡ‚Π²ΠΎ. ΠŸΡƒΡΡ‚ΡŒ рассматриваСмая ось ΠΏΡ€ΠΎΡ…ΠΎΠ΄ΠΈΡ‚ Ρ‡Π΅Ρ€Π΅Π· Ρ‚ΠΎΡ‡ΠΊΠΈ 0 ΠΈ О2, Π·Π°Π΄Π°Π½Π½Ρ‹Π΅ соотвСтствСнно радиусами-Π²Π΅ΠΊΡ‚ΠΎΡ€Π°ΠΌΠΈ ri ΠΈ Π“2, ΠΈΠΌΠ΅ΡŽΡ‰ΠΈΠΌΠΈ Π½Π°Ρ‡Π°Π»ΠΎ Π² Ρ†Π΅Π½Ρ‚Ρ€Π΅ масс Π‘. Эллипсоид ΠΈΠ½Π΅Ρ€Ρ†ΠΈΠΈ для Ρ‚ΠΎΡ‡ΠΊΠΈ Oi ΠΎΠ±ΠΎΠ·Π½Π°Ρ‡ΠΈΠΌ Эь Эллипсоид ΠΈΠ½Π΅Ρ€Ρ†ΠΈΠΈ для Ρ‚ΠΎΡ‡ΠΊΠΈ О2 ΠΎΠ±ΠΎΠ·Π½Π°Ρ‡ΠΈΠΌ Π­2. Π‘Ρ€Π°Π²Π½ΠΈΠΌ Π²Π΅ΠΊΡ‚ΠΎΡ€Ρ‹ zi(x) ΠΈ Z2(x) для эллипсоидов 9i ΠΈ Π­2. По Ρ‚Π΅ΠΎΡ€Π΅ΠΌΠ΅ 1.10.3 Π±ΡƒΠ΄Π΅ΠΌ ΠΈΠΌΠ΅Ρ‚ΡŒ  [c.56]


Π’Π°ΠΊΠΈΠΌ ΠΎΠ±Ρ€Π°Π·ΠΎΠΌ опСрация ΡƒΠ΄ΠΎΠ±Π½Π° ΠΏΡ€ΠΈ вычислСнии Ρ‚Π΅Π½Π·ΠΎΡ€ΠΎΠ² ΠΈΠ½Π΅Ρ€Ρ†ΠΈΠΈ мноТСства Ρ‚ΠΎΡ‡Π΅Ρ‡Π½Ρ‹Ρ… масс, Ссли радиусы-Π²Π΅ΠΊΡ‚ΠΎΡ€Ρ‹ Ρ‚ΠΎΡ‡Π΅ΠΊ Π²Ρ‹Ρ€Π°ΠΆΠ°ΡŽΡ‚ΡΡ ΠΊΠ°ΠΊ Π»ΠΈΠ½Π΅ΠΉΠ½Ρ‹Π΅ ΠΊΠΎΠΌΠ±ΠΈΠ½Π°Ρ†ΠΈΠΈ Π΄Ρ€ΡƒΠ³ΠΈΡ… ΠΊΠ°ΠΊΠΈΡ…-Π½ΠΈΠ±ΡƒΠ΄ΡŒ Π²Π΅ΠΊΡ‚ΠΎΡ€ΠΎΠ².
 [c.58]

ΠŸΡƒΡΡ‚ΡŒ ΠΏΠ»ΠΎΡΠΊΠΎΡΡ‚ΡŒ Π Π΅ ΠΈΠΌΠ΅Π΅Ρ‚ Π½ΠΎΡ€ΠΌΠ°Π»ΡŒ Π΅ ΠΈ ΠΏΡ€ΠΎΡ…ΠΎΠ΄ΠΈΡ‚ Ρ‡Π΅Ρ€Π΅Π· полюс О. ΠœΠΎΠΌΠ΅Π½Ρ‚ΠΎΠΌ ΠΈΠ½Π΅Ρ€Ρ†ΠΈΠΈ ΠΎΡ‚Π½ΠΎΡΠΈΡ‚Π΅Π»ΡŒΠ½ΠΎ плоскости Π’Π΅ мноТСства Ρ‚ΠΎΡ‡Π΅Ρ‡Π½Ρ‹Ρ… масс Ρ‚, с радиусами-Π²Π΅ΠΊΡ‚ΠΎΡ€Π°ΠΌΠΈ Π³, называСтся Π²Π΅Π»ΠΈΡ‡ΠΈΠ½Π°  [c.61]

ΠŸΡƒΡΡ‚ΡŒ Π³ β€” радиус-Π²Π΅ΠΊΡ‚ΠΎΡ€, ΠΈΠΌΠ΅ΡŽΡ‰ΠΈΠΉ Π½Π°Ρ‡Π°Π»ΠΎ Π² Ρ‚ΠΎΡ‡ΠΊΠ΅ О ΠΈ ΠΊΠΎΠ½Π΅Ρ† Π² Ρ†Π΅Π½Ρ‚Ρ€Π΅ масс мноТСства Q Ρ‚ΠΎΡ‡Π΅Ρ‡Π½Ρ‹Ρ… масс. А, Π’, Π‘ β€” Π³Π»Π°Π²Π½Ρ‹Π΅ Ρ†Π΅Π½Ρ‚Ρ€Π°Π»ΡŒΠ½Ρ‹Π΅ ΠΌΠΎΠΌΠ΅Π½Ρ‚Ρ‹ ΠΈΠ½Π΅Ρ€Ρ†ΠΈΠΈ мноТСства Q. Найти ΠΌΠΎΠΌΠ΅Π½Ρ‚ ΠΈΠ½Π΅Ρ€Ρ†ΠΈΠΈ мноТСства Q ΠΎΡ‚Π½ΠΎΡΠΈΡ‚Π΅Π»ΡŒΠ½ΠΎ оси с Π½Π°ΠΏΡ€Π°Π²Π»ΡΡŽΡ‰ΠΈΠΌ Π²Π΅ΠΊΡ‚ΠΎΡ€ΠΎΠΌ Π΅, проходящСй Ρ‡Π΅Ρ€Π΅Π· Ρ‚ΠΎΡ‡ΠΊΡƒ О.  [c.74]

НазовСм апСксом Ρ‚ΠΎΡ‡ΠΊΡƒ, Π² ΠΊΠΎΡ‚ΠΎΡ€ΠΎΠΉ Π»ΡƒΡ‡, выходящий ΠΈΠ· Π½Π΅ΠΏΠΎΠ΄Π²ΠΈΠΆΠ½ΠΎΠΉ Ρ‚ΠΎΡ‡ΠΊΠΈ О ΠΊΠΎΠ»Π»ΠΈΠ½Π΅Π°Ρ€Π½ΠΎ Π²Π΅ΠΊΡ‚ΠΎΡ€Ρƒ ΠΎ , пСрСсСкаСт эллипсоид ΠΈΠ½Π΅Ρ€Ρ†ΠΈΠΈ. ΠŸΡƒΡΡ‚ΡŒ Π³ β€” радиус-Π²Π΅ΠΊΡ‚ΠΎΡ€ апСкса.  [c.467]

ΠŸΠ΅Ρ€Π²Ρ‹Π΅ Π΄Π²Π° Ρ‡Π»Π΅Π½Π° этой Ρ„ΠΎΡ€ΠΌΡƒΠ»Ρ‹ постоянны ΠΏΡ€ΠΈ фиксированном радиусС ΠΎΡ€Π±ΠΈΡ‚Ρ‹ ΠΈ ΠΏΠΎΡ‚ΠΎΠΌΡƒ Π½Π΅ сущСствСнны. Π’Ρ‹Ρ€Π°ΠΆΠ΅Π½ΠΈΠ΅ Π² скобках Ρƒ Ρ‚Ρ€Π΅Ρ‚ΡŒΠ΅Π³ΠΎ Ρ‡Π»Π΅Π½Π° Π΅ΡΡ‚ΡŒ ΠΌΠΎΠΌΠ΅Π½Ρ‚ ΠΈΠ½Π΅Ρ€Ρ†ΠΈΠΈ спутника ΠΎΡ‚Π½ΠΎΡΠΈΡ‚Π΅Π»ΡŒΠ½ΠΎ оси с Π½Π°ΠΏΡ€Π°Π²Π»ΡΡŽΡ‰ΠΈΠΌ Π²Π΅ΠΊΡ‚ΠΎΡ€ΠΎΠΌ Оз, Ρ‚.Π΅. ΠΎΡ‚Π½ΠΎΡΠΈΡ‚Π΅Π»ΡŒΠ½ΠΎ радиуса-Π²Π΅ΠΊΡ‚ΠΎΡ€Π° Ρ†Π΅Π½Ρ‚Ρ€Π°  [c.506]

Π’Π΅ΠΎΡ€Π΅ΠΌΠ° 6. 13.1. Π’ принятых прСдполоТСниях сумма ΠΏΠΎΡ‚Π΅Π½Ρ†ΠΈΠ°Π»ΡŒΠ½Ρ‹Ρ… энСргий Π³Ρ€Π°Π²ΠΈΡ‚Π°Ρ†ΠΈΠΎΠ½Π½Ρ‹Ρ… сил ΠΈ сил ΠΈΠ½Π΅Ρ€Ρ†ΠΈΠΈ ΠΏΡ€ΠΈΠ½ΠΈΠΌΠ°Π΅Ρ‚ минимальноС Π·Π½Π°Ρ‡Π΅Π½ΠΈΠ΅, ΠΊΠΎΠ³Π΄Π° наибольшая ось эллипсоида ΠΈΠ½Π΅Ρ€Ρ†ΠΈΠΈ Π½Π°ΠΏΡ€Π°Π²Π»Π΅Π½Π° вдоль радиуса-Π²Π΅ΠΊΡ‚ΠΎΡ€Π° Ρ†Π΅Π½Ρ‚Ρ€Π° масс, Π° наимСньшая β€” ΠΏΠΎ Π½ΠΎΡ€ΠΌΠ°Π»ΠΈ ΠΊ плоскости ΠΎΡ€Π±ΠΈΡ‚Ρ‹.  [c.508]

Из повСрхностСй Π²Ρ‚ΠΎΡ€ΠΎΠ³ΠΎ порядка этому ΡƒΡΠ»ΠΎΠ²ΠΈΡŽ удовлСтворяСт Ρ‚ΠΎΠ»ΡŒΠΊΠΎ ΠΎΠ΄Π½Π°, Π° ΠΈΠΌΠ΅Π½Π½ΠΎ эллипсоид. НайдСнный эллипсоид Π½Π°Π·Ρ‹Π²Π°ΡŽΡ‚ эллипсоидом ΠΈΠ½Π΅Ρ€Ρ†ΠΈΠΈ для Π΄Π°Π½Π½ΠΎΠ³ΠΎ Ρ‚Π΅Π»Π° Π² Ρ‚ΠΎΡ‡ΠΊΠ΅ О. ΠžΡ‡Π΅Π²ΠΈΠ΄Π½ΠΎ, эллипсоид ΠΈΠ½Π΅Ρ€Ρ†ΠΈΠΈ для Π΄Π°Π½Π½ΠΎΠ³ΠΎ Ρ‚Π΅Π»Π° ΠΌΠΎΠΆΠ½ΠΎ ΠΏΠΎΡΡ‚Ρ€ΠΎΠΈΡ‚ΡŒ Π² любой Ρ‚ΠΎΡ‡ΠΊΠ΅ пространства. ΠŸΠΎΡΡ‚ΠΎΠΌΡƒ эллипсоидом ΠΈΠ½Π΅Ρ€Ρ†ΠΈΠΈ для Π΄Π°Π½Π½ΠΎΠ³ΠΎ Ρ‚Π΅Π»Π° Π² ΠΊΠ°ΠΊΠΎΠΉ-Π»ΠΈΠ±ΠΎ Ρ‚ΠΎΡ‡ΠΊΠ΅ Π½Π°Π·Ρ‹Π²Π°ΡŽΡ‚ эллипсоид с Ρ†Π΅Π½Ρ‚Ρ€ΠΎΠΌ Π² этой Ρ‚ΠΎΡ‡ΠΊΠ΅, Ρ†Π΅Π½Ρ‚Ρ€Π°Π»ΡŒΠ½Ρ‹Π΅ радиусы-Π²Π΅ΠΊΡ‚ΠΎΡ€Ρ‹ Ρ‚ΠΎΡ‡Π΅ΠΊ ΠΊΠΎΡ‚ΠΎΡ€ΠΎΠ³ΠΎ Ρ€Π°Π²Π½Ρ‹ ΠΎΠ±Ρ€Π°Ρ‚Π½Ρ‹ΠΌ значСниям ΠΊΠ²Π°Π΄Ρ€Π°Ρ‚Π½Ρ‹Ρ… ΠΊΠΎΡ€Π½Π΅ΠΉ ΠΈΠ· ΠΌΠΎΠΌΠ΅Π½Ρ‚ΠΎΠ² ΠΈΠ½Π΅Ρ€Ρ†ΠΈΠΈ Ρ‚Π΅Π»Π° ΠΎΡ‚Π½ΠΎΡΠΈΡ‚Π΅Π»ΡŒΠ½ΠΎ осСй, Π½Π°ΠΏΡ€Π°Π²Π»Π΅Π½Π½Ρ‹Ρ… ΠΏΠΎ этим радиусам-Π²Π΅ΠΊΡ‚ΠΎΡ€Π°ΠΌ.  [c.249]

ΠŸΠΎΠ΄ΡΡ‚Π°Π²Π»ΡΡ Π² (29) Π£ = Π£,, ΠΏΠΎΠ»ΡƒΡ‡ΠΈΠΌ Ρ‚ΠΎΠ»ΡŒΠΊΠΎ Π΄Π²Π° нСзависимых уравнСния для опрСдСлСния ΠΊΠΎΠΎΡ€Π΄ΠΈΠ½Π°Ρ‚ Ρ‚ΠΎΡ‡ΠΊΠΈ Ρ…, Ρƒ, Π³ эллипсоида ΠΈΠ½Π΅Ρ€Ρ†ΠΈΠΈ, ΡΠΎΠΎΡ‚Π²Π΅Ρ‚ΡΡ‚Π²ΡƒΡŽΡ‰ΠΈΡ… Π³Π»Π°Π²Π½ΠΎΠΉ оси ΠΈΠ½Π΅Ρ€Ρ†ΠΈΠΈ. Для ΠΊΠΎΡ‚ΠΎΡ€ΠΎΠΉ Π³Π»Π°Π²Π½Ρ‹ΠΉ ΠΌΠΎΠΌΠ΅Π½Ρ‚ ΠΈΠ½Π΅Ρ€Ρ†ΠΈΠΈ Π΅ΡΡ‚ΡŒ Π’Ρ€Π΅Ρ‚ΡŒΠ΅ ΡƒΡ€Π°Π²Π½Π΅Π½ΠΈΠ΅ систСмы Π±ΡƒΠ΄Π΅Ρ‚ слСдствиСм Π΄Π²ΡƒΡ… Π΄Ρ€ΡƒΠ³ΠΈΡ… ΡƒΡ€Π°Π²Π½Π΅Π½ΠΈΠΉ, Ρ‚Π°ΠΊ ΠΊΠ°ΠΊ ΠΎΠΏΡ€Π΅Π΄Π΅Π»ΠΈΡ‚Π΅Π»ΡŒ этой систСмы Ρ€Π°Π²Π΅Π½ Π½ΡƒΠ»ΡŽ. Из (29) ΠΌΠΎΠΆΠ½ΠΎ Π½Π°ΠΉΡ‚ΠΈ Ρ‚ΠΎΠ»ΡŒΠΊΠΎ Π΄Π²Π΅ Π²Π΅Π»ΠΈΡ‡ΠΈΠ½Ρ‹, Π½Π°ΠΏΡ€ΠΈΠΌΠ΅Ρ€ Ρ…/Π³ ΠΈ Ρƒ Π³. Они опрСдСлят Π½Π°ΠΏΡ€Π°Π²Π»Π΅Π½ΠΈΠ΅ Π²Π΅ΠΊΡ‚ΠΎΡ€Π° Π³, вдоль Π³Π»Π°Π²Π½ΠΎΠΉ оси ΠΈΠ½Π΅Ρ€Ρ†ΠΈΠΈ, ΠΌΠΎΠΌΠ΅Π½Ρ‚ ΠΈΠ½Π΅Ρ€Ρ†ΠΈΠΈ ΠΎΡ‚Π½ΠΎΡΠΈΡ‚Π΅Π»ΡŒΠ½ΠΎ ΠΊΠΎΡ‚ΠΎΡ€ΠΎΠΉ Π΅ΡΡ‚ΡŒ ΠœΠΎΠ΄ΡƒΠ»ΡŒ радиус-Π²Π΅ΠΊΡ‚ΠΎΡ€Π° ΠΏ остаСтся Π½Π΅ΠΎΠΏΡ€Π΅Π΄Π΅Π»Π΅Π½Π½Ρ‹ΠΌ. Аналогично ΠΎΠΏΡ€Π΅Π΄Π΅Π»ΡΡŽΡ‚ΡΡ направлСния Π²Π΅ΠΊΡ‚ΠΎΡ€ΠΎΠ² Π“Π΄ ΠΈ Π“Π΄ вдоль Π΄Π²ΡƒΡ… Π΄Ρ€ΡƒΠ³ΠΈΡ… Π³Π»Π°Π²Π½Ρ‹Ρ… осСй ΠΈΠ½Π΅Ρ€Ρ†ΠΈΠΈ, для ΠΊΠΎΡ‚ΠΎΡ€Ρ‹Ρ… Π³Π»Π°Π²Π½Ρ‹Π΅ ΠΌΠΎΠΌΠ΅Π½Ρ‚Ρ‹ ΠΈΠ½Π΅Ρ€Ρ†ΠΈΠΈ Ρ€Π°Π²Π½Ρ‹ Π£.2 ΠΈ Π£Π΄. МоТно Π΄ΠΎΠΊΠ°Π·Π°Ρ‚ΡŒ, Ρ‡Ρ‚ΠΎ Π²Π΅ΠΊΡ‚ΠΎΡ€Ρ‹ Π³,, ΠΈ Π“Π΄, Π½Π°ΠΏΡ€Π°Π²Π»Π΅Π½Π½Ρ‹Π΅ вдоль Π³Π»Π°Π²Π½Ρ‹Ρ… осСй ΠΈΠ½Π΅Ρ€Ρ†ΠΈΠΈ, Π²Π·Π°ΠΈΠΌΠ½ΠΎ пСрпСндикулярны.  [c.277]

Для выраТСния Ρ†Π΅Π½Ρ‚Ρ€ΠΎΠ±Π΅ΠΆΠ½Ρ‹Ρ… ΠΌΠΎΠΌΠ΅Π½Ρ‚ΠΎΠ² ΠΈΠ½Π΅Ρ€Ρ†ΠΈΠΈ Ρ‡Π΅Ρ€Π΅Π· Π³Π»Π°Π²Π½Ρ‹Π΅ ΠΌΠΎΠΌΠ΅Π½Ρ‚Ρ‹ ΠΈΠ½Π΅Ρ€Ρ†ΠΈΠΈ ΠΈΡΠΏΠΎΠ»ΡŒΠ·ΡƒΠ΅ΠΌ Ρ„ΠΎΡ€ΠΌΡƒΠ»Ρ‹ прСобразования ΠΊΠΎΠΎΡ€Π΄ΠΈΠ½Π°Ρ‚ Ρ‚ΠΎΡ‡Π΅ΠΊ Ρ‚Π΅Π»Π° ΠΏΡ€ΠΈ ΠΏΠΎΠ²ΠΎΡ€ΠΎΡ‚Π΅ осСй ΠΊΠΎΠΎΡ€Π΄ΠΈΠ½Π°Ρ‚ Π²ΠΎΠΊΡ€ΡƒΠ³ Ρ‚ΠΎΡ‡ΠΊΠΈ О (рис. 36). Π­Ρ‚ΠΈ Ρ„ΠΎΡ€ΠΌΡƒΠ»Ρ‹ ΠΏΠΎΠ»ΡƒΡ‡ΠΈΠΌ ΠΏΡ€ΠΎΠ΅Ρ†ΠΈΡ€ΠΎΠ²Π°Π½ΠΈΠ΅ΠΌ Π½Π° оси ΠžΡ…ΡƒΠ³ радиус-Π²Π΅ΠΊΡ‚ΠΎΡ€Π° Ρ‚ΠΎΡ‡ΠΊΠΈ М , Ρ€Π°Π·Π»ΠΎΠΆΠ΅Π½Π½ΠΎΠ³ΠΎ ΠΏΡ€Π΅Π΄Π²Π°Ρ€ΠΈΡ‚Π΅Π»ΡŒΠ½ΠΎ Π½Π° ΡΠΎΡΡ‚Π°Π²Π»ΡΡŽΡ‰ΠΈΠ΅, ΠΏΠ°Ρ€Π°Π»Π»Π΅Π»ΡŒΠ½Ρ‹Π΅ осям Π΄Π²ΡƒΡ… систСм осСй ΠΊΠΎΠΎΡ€Π΄ΠΈΠ½Π°Ρ‚ Π² Ρ‚ΠΎΡ‡ΠΊΠ΅ О. ИмССм  [c.278]

ГСомСтричСская Ρ‚ΠΎΡ‡ΠΊΠ°, для ΠΊΠΎΡ‚ΠΎΡ€ΠΎΠΉ сумма ΠΏΡ€ΠΎΠΈΠ·Π²Π΅Π΄Π΅Π½ΠΈΠΉ масс всСх ΠΌΠ°Ρ‚Π΅Ρ€ΠΈΠ°Π»ΡŒΠ½Ρ‹Ρ… Ρ‚ΠΎΡ‡Π΅ΠΊ, ΠΎΠ±Ρ€Π°Π·ΡƒΡŽΡ‰ΠΈΡ… ΠΌΠ΅Ρ…Π°Π½ΠΈΡ‡Π΅ΡΠΊΡƒΡŽ систСму, Π½Π° ΠΈΡ… радиусы-Π²Π΅ΠΊΡ‚ΠΎΡ€Ρ‹, ΠΏΡ€ΠΎΠ²Π΅Π΄Ρ‘Π½Π½Ρ‹Π΅ ΠΈΠ· этой Ρ‚ΠΎΡ‡ΠΊΠΈ, Ρ€Π°Π²Π½Π° Π½ΡƒΠ»ΡŽ (Ρ‚ΠΎ ΠΆΠ΅, Ρ‡Ρ‚ΠΎ ΠΈ Ρ†Π΅Π½Ρ‚Ρ€ ΠΈΠ½Π΅Ρ€Ρ†ΠΈΠΈ).  [c.98]

Эллипсоид с Ρ†Π΅Π½Ρ‚Ρ€ΠΎΠΌ Π² Π΄Π°Π½Π½ΠΎΠΉ Ρ‚ΠΎΡ‡ΠΊΠ΅, для ΠΊΠΎΡ‚ΠΎΡ€ΠΎΠ³ΠΎ ΠΊΠ²Π°Π΄Ρ€Π°Ρ‚ радиуса-Π²Π΅ΠΊΡ‚ΠΎΡ€Π° ΠΊΠ°ΠΆΠ΄ΠΎΠΉ Π΅Π³ΠΎ Ρ‚ΠΎΡ‡ΠΊΠΈ, ΠΏΡ€ΠΎΠ²Π΅Π΄Ρ‘Π½Π½ΠΎΠ³ΠΎ ΠΈΠ· этого Ρ†Π΅Π½Ρ‚Ρ€Π°, ΠΎΠ±Ρ€Π°Ρ‚Π½ΠΎ ΠΏΡ€ΠΎΠΏΠΎΡ€Ρ†ΠΈΠΎΠ½Π°Π»Π΅Π½ ΠΌΠΎΠΌΠ΅Π½Ρ‚Ρƒ ΠΈΠ½Π΅Ρ€Ρ†ΠΈΠΈ мСханичСской систСмы ΠΎΡ‚Π½ΠΎΡΠΈΡ‚Π΅Π»ΡŒΠ½ΠΎ оси, Π½Π°ΠΏΡ€Π°Π²Π»Π΅Π½Π½ΠΎΠΉ вдоль радиуса-Π²Π΅ΠΊΡ‚ΠΎΡ€Π°.  [c.104]

ΠŸΡ€ΠΈΠΌΠ΅Π½ΠΈΠΌ ΠΊ систСмС ΠΏΠ°Ρ€Π°Π»Π»Π΅Π»ΡŒΠ½Ρ‹Ρ… сил m,w Ρ„ΠΎΡ€ΠΌΡƒΠ»Ρƒ (III. 56) ΠΏΠ΅Ρ€Π²ΠΎΠ³ΠΎ Ρ‚ΠΎΠΌΠ°. Π’ΠΎΠ³Π΄Π° послС ΠΎΡ‡Π΅Π²ΠΈΠ΄Π½Ρ‹Ρ… ΡƒΠΏΡ€ΠΎΡ‰Π΅Π½ΠΈΠΉ Π½Π°ΠΉΠ΄Π΅ΠΌ, Ρ‡Ρ‚ΠΎ радиус-Π²Π΅ΠΊΡ‚ΠΎΡ€ Ρ†Π΅Π½Ρ‚Ρ€Π° ΠΈΠ½Π΅Ρ€Ρ†ΠΈΠΈ систСмы ΠΌΠ°Ρ‚Π΅Ρ€ΠΈΠ°Π»ΡŒΠ½Ρ‹Ρ… Ρ‚ΠΎΡ‡Π΅ΠΊ опрСдСляСтся Ρ„ΠΎΡ€ΠΌΡƒΠ»ΠΎΠΉ  [c.41]

Π—Π΄Π΅ΡΡŒ Π³ β€” радиус-Π²Π΅ΠΊΡ‚ΠΎΡ€ Ρ‚ΠΎΡ‡ΠΊΠΈ ΠΎΡ‚Π½ΠΎΡΠΈΡ‚Π΅Π»ΡŒΠ½ΠΎ Ρ†Π΅Π½Ρ‚Ρ€Π° ΠΈΠ½Π΅Ρ€Ρ†ΠΈΠΈ систСмы. Богласно Ρ„ΠΎΡ€ΠΌΡƒΠ»Π΅ (I. 49) ΠΈΠΌΠ΅Π΅ΠΌ  [c.55]


Π—Π΄Π΅ΡΡŒ Гс β€” ΠΎΡ‚Π½ΠΎΡΠΈΡ‚Π΅Π»ΡŒΠ½Ρ‹ΠΉ радиус-Π²Π΅ΠΊΡ‚ΠΎΡ€ Ρ†Π΅Π½Ρ‚Ρ€Π° ΠΈΠ½Π΅Ρ€Ρ†ΠΈΠΈ,Ус β€” ΠΎΡ‚Π½ΠΎΡΠΈΡ‚Π΅Π»ΡŒΠ½Π°Ρ ΡΠΊΠΎΡ€ΠΎΡΡ‚ΡŒ Ρ†Π΅Π½Ρ‚Ρ€Π° ΠΈΠ½Π΅Ρ€Ρ†ΠΈΠΈ.  [c.55]

Если Π₯( Π€ Π₯Π°, Ρ‚ΠΎ Π²Ρ‚ΠΎΡ€ΠΎΠΉ ΠΌΠ½ΠΎΠΆΠΈΡ‚Π΅Π»ΡŒ Π² Π»Π΅Π²ΠΎΠΉ части равСнства (ΠΊ) Ρ€Π°Π²Π΅Π½ Π½ΡƒΠ»ΡŽ. Однако этот ΠΌΠ½ΠΎΠΆΠΈΡ‚Π΅Π»ΡŒ являСтся скалярным ΠΏΡ€ΠΎΠΈΠ·Π²Π΅Π΄Π΅Π½ΠΈΠ΅ΠΌ радиусов-Π²Π΅ΠΊΡ‚ΠΎΡ€ΠΎΠ² Ρ‚ΠΎΡ‡Π΅ΠΊ пСрСсСчСния Π΄Π²ΡƒΡ… Π³Π»Π°Π²Π½Ρ‹Ρ… осСй с эллипсоидом ΠΈΠ½Π΅Ρ€Ρ†ΠΈΠΈ. Π‘Π»Π΅Π΄ΠΎΠ²Π°Ρ‚Π΅Π»ΡŒΠ½ΠΎ, направлСния этих Π³Π»Π°Π²Π½Ρ‹Ρ… осСй ΠΎΡ€Ρ‚ΠΎΠ³ΠΎΠ½Π°Π»ΡŒΠ½Ρ‹.  [c.83]

Π—Π΄Π΅ΡΡŒ Π“ β€” радиусы-Π²Π΅ΠΊΡ‚ΠΎΡ€Ρ‹ Ρ‚ΠΎΡ‡Π΅ΠΊ систСмы ΠΎΡ‚Π½ΠΎΡΠΈΡ‚Π΅Π»ΡŒΠ½ΠΎ Π΅Π΅ Ρ†Π΅Π½Ρ‚Ρ€Π° ΠΈΠ½Π΅Ρ€Ρ†ΠΈΠΈ, Пг β€” ΠΏΠΎΡ‚Π΅Π½Ρ†ΠΈΠ°Π»ΡŒΠ½Π°Ρ энСргия Π² ΠΎΡ‚Π½ΠΎΡΠΈΡ‚Π΅Π»ΡŒΠ½Ρ‹Ρ… ΠΊΠΎΠΎΡ€Π΄ΠΈΠ½Π°Ρ‚Π°Ρ…, -с β€” ускорСниС Ρ†Π΅Π½Ρ‚Ρ€Π° ΠΈΠ½Π΅Ρ€Ρ†ΠΈΠΈ.  [c.101]

Π’Π°ΠΊΠΈΠΌ ΠΎΠ±Ρ€Π°Π·ΠΎΠΌ, ΠΏΡ€ΠΈΡ…ΠΎΠ΄ΠΈΠΌ ΠΊ ΡΠ»Π΅Π΄ΡƒΡŽΡ‰Π΅ΠΉ, ΠΏΠΎΠ»ΡƒΡ‡Π΅Π½Π½ΠΎΠΉ ΠŸΡƒΠ°Π½ΡΠΎ, гСомСтричСской ΠΈΠ½Ρ‚Π΅Ρ€ΠΏΡ€Π΅Ρ‚Π°Ρ†ΠΈΠΈ двиТСния Ρ‚Π²Π΅Ρ€Π΄ΠΎΠ³ΠΎ Ρ‚Π΅Π»Π° Π² случаС Π­ΠΉΠ»Π΅Ρ€Π° эллипсоид ΠΈΠ½Π΅Ρ€Ρ†ΠΈΠΈ для Π½Π΅ΠΏΠΎΠ΄Π²ΠΈΠΆΠ½ΠΎΠΉ Ρ‚ΠΎΡ‡ΠΊΠΈ катится Π±Π΅Π· скольТСния ΠΏΠΎ плоскости, Π½Π΅ΠΏΠΎΠ΄Π²ΠΈΠΆΠ½ΠΎΠΉ Π² пространствС-, эта ΠΏΠ»ΠΎΡΠΊΠΎΡΡ‚ΡŒ пСрпСндикулярна кинСтичСскому ΠΌΠΎΠΌΠ΅Π½Ρ‚Ρƒ угловая ΡΠΊΠΎΡ€ΠΎΡΡ‚ΡŒ Ρ‚Π΅Π»Π° ΠΏΡ€ΠΎΠΏΠΎΡ€Ρ†ΠΈΠΎΠ½Π°Π»ΡŒΠ½Π° Π΄Π»ΠΈΠ½Π΅ радиуса-Π²Π΅ΠΊΡ‚ΠΎΡ€Π° Ρ‚ΠΎΡ‡ΠΊΠΈ касания, Π° ΠΏΠΎ Π½Π°ΠΏΡ€Π°Π²Π»Π΅Π½ΠΈΡŽ с Π½ΠΈΠΌ совпадаСт.  [c.162]

Π€ΠΎΡ€ΠΌΡƒΠ»Π°ΠΌΠΈ (5) ΠΈ (6) ΠΎΠΏΡ€Π΅Π΄Π΅Π»ΡΡŽΡ‚ΡΡ соотвСтствСнно радиус-Π²Π΅ΠΊΡ‚ΠΎΡ€ ΠΈΠ»ΠΈ ΠΊΠΎΠΎΡ€Π΄ΠΈΠ½Π°Ρ‚Ρ‹ Ρ†Π΅Π½Ρ‚Ρ€Π° масс Ρ†Π΅Π½Ρ‚Ρ€Π° ΠΈΠ½Π΅Ρ€Ρ†ΠΈΠΈ) Ρ‚Π΅Π»Π°. Как Π²ΠΈΠ΄Π½ΠΎ ΠΈΠ· этих Ρ„ΠΎΡ€ΠΌΡƒΠ», ΠΏΠΎΠ»ΠΎΠΆΠ΅Π½ΠΈΠ΅ Ρ†Π΅Π½Ρ‚Ρ€Π° масс зависит Ρ‚ΠΎΠ»ΡŒΠΊΠΎ ΠΎΡ‚ распрСдСлСния масс Π² объСмС, Π·Π°Π½ΠΈΠΌΠ°Π΅ΠΌΠΎΠΌ Ρ‚Π΅Π»ΠΎΠΌ. ΠŸΠΎΠ½ΡΡ‚ΠΈΠ΅ ΠΎ Ρ†Π΅Π½Ρ‚Ρ€Π΅ масс являСтся Π±ΠΎΠ»Π΅Π΅ ΠΎΠ±Ρ‰ΠΈΠΌ, Ρ‡Π΅ΠΌ понятиС ΠΎ Ρ†Π΅Π½Ρ‚Ρ€Π΅ тяТСсти, Ρ‚Π°ΠΊ ΠΊΠ°ΠΊ ΠΎΠ½ΠΎ ΠΈΠΌΠ΅Π΅Ρ‚ смысл Π½Π΅ Ρ‚ΠΎΠ»ΡŒΠΊΠΎ для ΠΎΠ΄Π½ΠΎΠ³ΠΎ Ρ‚Π²Π΅Ρ€Π΄ΠΎΠ³ΠΎ Ρ‚Π΅Π»Π°, Π½ΠΎ ΠΈ для любой мСханичСской систСмы ΠΊΡ€ΠΎΠΌΠ΅ Ρ‚ΠΎΠ³ΠΎ, это понятиС Π½Π΅ связано с Ρ‚Π΅ΠΌ, находится Ρ‚Π΅Π»ΠΎ Π² ΠΏΠΎΠ»Π΅ тяТСсти ΠΈΠ»ΠΈ Π½Π΅Ρ‚. Для Ρ‚Π΅Π»Π°, находящСгося Π² ΠΎΠ΄Π½ΠΎΡ€ΠΎΠ΄Π½ΠΎΠΌ ΠΏΠΎΠ»Π΅ тяТСсти (Π² ΠΏΠΎΠ»Π΅ тяТСсти, Π³Π΄Π΅ -= onst), полоТСния Ρ†Π΅Π½Ρ‚Ρ€Π° тяТСсти ΠΈ Ρ†Π΅Π½Ρ‚Ρ€Π° масс ΡΠΎΠ²ΠΏΠ°Π΄Π°ΡŽΡ‚.  [c.213]

Радиус-Π²Π΅ΠΊΡ‚ΠΎΡ€ Π³ β€” Π“1Π΅ 1 -Π¬ Π“2Π²2 -Π¬ Π³Π·Π΅ , Ρ‚ΠΎΡ‡ΠΊΠΈ Π½Π° ΠΏΠΎΠ»ΠΎΠ΄ΠΈΠΈ Π΄ΠΎΠ»ΠΆΠ΅Π½ ΡƒΠ΄ΠΎΠ²Π»Π΅Ρ‚Π²ΠΎΡ€ΡΡ‚ΡŒ ΠΊΠ°ΠΊ ΡƒΡ€Π°Π²Π½Π΅Π½ΠΈΡŽ эллипсоида ΠΈΠ½Π΅Ρ€Ρ†ΠΈΠΈ  [c.468]

Π’ ΠΎΠ±Ρ‰Π΅ΠΌ случаС полодия слуТит пСрСсСчСниСм эллипсоида ΠΈΠ½Π΅Ρ€Ρ†ΠΈΠΈ ΠΈ конуса Π²Ρ‚ΠΎΡ€ΠΎΠ³ΠΎ порядка, ΠΈΠΌΠ΅ΡŽΡ‰Π΅Π³ΠΎ Ρ‚Π΅ ΠΆΠ΅ плоскости симмСтрии, Ρ‡Ρ‚ΠΎ ΠΈ эллипсоид. Она состоит ΠΈΠ· Π΄Π²ΡƒΡ… Ρ€Π°Π·Π»ΠΈΡ‡Π½Ρ‹Ρ… Π·Π°ΠΌΠΊΠ½ΡƒΡ‚Ρ‹Ρ… Π²Π΅Ρ‚Π²Π΅ΠΉ, симмСтричных Π΄Ρ€ΡƒΠ³ ΠΊ Π΄Ρ€ΡƒΠ³Ρƒ ΠΎΡ‚Π½ΠΎΡΠΈΡ‚Π΅Π»ΡŒΠ½ΠΎ Π½Π΅ΠΏΠΎΠ΄Π²ΠΈΠΆΠ½ΠΎΠΉ Ρ‚ΠΎΡ‡ΠΊΠΈ ΠΈ ΠΎΠ΄Π½ΠΎΠΉ ΠΈΠ· Π³Π»Π°Π²Π½Ρ‹Ρ… плоскостСй эл.липсоида, ΠΈ ΠΎΠ±Π»Π°Π΄Π°Π΅Ρ‚ Ρ‡Π΅Ρ‚Ρ‹Ρ€ΡŒΠΌΡ Π²Π΅Ρ€-Ρ‰ΠΈΠ½Π°ΠΌΠΈ, для ΠΊΠΎΡ‚ΠΎΡ€Ρ‹Ρ… радиус-Π²Π΅ΠΊΡ‚ΠΎΡ€ Π³, выходящий ΠΈΠ· Π½Π΅ΠΏΠΎΠ΄Π²ΠΈΠΆΠ½ΠΎΠΉ Ρ‚ΠΎΡ‡ΠΊΠΈ, ΠΈΠΌΠ΅Π΅Ρ‚ максимум ΠΈΠ»ΠΈ ΠΌΠΈΠ½ΠΈΠΌΡƒΠΌ модуля. ΠŸΡ€ΠΈ Π΄Π²ΠΈΠΆΠ΅Π½ΠΈΠΈ ΠΎΠ΄Π½Π° ΠΈΠ· Π²Π΅Ρ‚Π²Π΅ΠΉ ΠΏΠΎΠ»ΠΎΠ΄ΠΈΠΈ катится ΠΏΠΎ Π½Π΅ΠΏΠΎΠ΄Π²ΠΈΠΆΠ½ΠΎΠΉ плоскости Π . Вторая Π²Π΅Ρ‚Π²ΡŒ катится ΠΏΠΎ плоскости, симмСтричной Π  ΠΎΡ‚Π½ΠΎΡΠΈΡ‚Π΅Π»ΡŒΠ½ΠΎ Π½Π΅ΠΏΠΎΠ΄Π²ΠΈΠΆΠ½ΠΎΠΉ Ρ‚ΠΎΡ‡ΠΊΠΈ. ΠžΠ±Ρ‰ΠΈΠΉ Π²ΠΈΠ΄ располоТСния ΠΏΠΎΠ»ΠΎΠ΄ΠΈΠΉ Π½Π° эл.липсоидС ΠΈΠ½Π΅Ρ€Ρ†ΠΈΠΈ прСдставлСн Π½Π° рис. 6.7.1. ИмССм однопарамСтричСскоС ΠΏΠΎ Π’ сСмСйство ΠΊΡ€ΠΈΠ²Ρ‹Ρ….  [c.469]

Из Ρ†Π΅Π½Ρ‚Ρ€Π° О Π—Π΅ΠΌΠ»ΠΈ ΠΏΡ€ΠΎΠ²Π΅Π΄Π΅ΠΌ радиус-Π²Π΅ΠΊΡ‚ΠΎΡ€ Н Ρ†Π΅Π½Ρ‚Ρ€Π° масс спутника. Π’Ρ‹Π±Π΅Ρ€Π΅ΠΌ Π²Ρ€Π°Ρ‰Π°ΡŽΡ‰ΠΈΠΉΡΡ Ρ€Π΅ΠΏΠ΅Ρ€ ОС /Π΅2Π΅Π· Ρ‚Π°ΠΊ, Ρ‡Ρ‚ΠΎΠ±Ρ‹ ось 03 Π±Ρ‹Π»Π° ΠΊΠΎΠ»Π»ΠΈΠ½Π΅Π°Ρ€Π½Π° К, ось Π΅ ΠΎ β€” ΠΏΠ°Ρ€Π°Π»Π»Π΅Π»ΡŒΠ½Π° скорости V Ρ†Π΅Π½Ρ‚Ρ€Π° масс, ось Π΅” β€” пСрпСндикулярна ΠΊ плоскости ΠΎΡ€Π±ΠΈΡ‚Ρ‹ ΠΈ составляла с ΡƒΠΊΠ°Π·Π°Π½Π½Ρ‹ΠΌΠΈ двумя ΠΏΡ€Π°Π²ΡƒΡŽ Ρ‚Ρ€ΠΎΠΉΠΊΡƒ. ΠžΡ‚Π½ΠΎΡΠΈΡ‚Π΅Π»ΡŒΠ½ΠΎ Π°Π±ΡΠΎΠ»ΡŽΡ‚Π½ΠΎΠ³ΠΎ (см. 3.14) Ρ€Π΅ΠΏΠ΅Ρ€Π° 0010203 Ρ€Π΅ΠΏΠ΅Ρ€ О0 /020Π· вращаСтся с постоянной ΡƒΠ³Π»ΠΎΠ²ΠΎΠΉ ΡΠΊΠΎΡ€ΠΎΡΡ‚ΡŒΡŽ Π° β€” ь/К Π²ΠΎΠΊΡ€ΡƒΠ³ Π²Π΅ΠΊΡ‚ΠΎΡ€Π° Π΅” = 01- НайдСм условиС, ΠΏΡ€ΠΈ ΠΊΠΎΡ‚ΠΎΡ€ΠΎΠΌ спутник Π±ΡƒΠ΄Π΅Ρ‚ Π½Π°Ρ…ΠΎΠ΄ΠΈΡ‚ΡŒΡΡ Π² равновСсии ΠΎΡ‚Π½ΠΎΡΠΈΡ‚Π΅Π»ΡŒΠ½ΠΎ Π²Ρ€Π°Ρ‰Π°ΡŽΡ‰Π΅Π³ΠΎΡΡ Ρ€Π΅ΠΏΠ΅Ρ€Π° ОС Π΅ Π΅ ΠΏΠΎΠ΄ дСйствиСм сил тяготСния ΠΈ сил ΠΈΠ½Π΅Ρ€Ρ†ΠΈΠΈ Ρ†Π΅Π½Ρ‚1ю-Π±Π΅ΠΆΠ½Ρ‹Ρ… ΠΈ кориолисовых.  [c.504]

Если оси ΠΊΠΎΠΎΡ€Π΄ΠΈΠ½Π°Ρ‚ Ox y z ΡΠ²Π»ΡΡŽΡ‚ΡΡ Π³Π»Π°Π²Π½Ρ‹ΠΌΠΈ осями ΠΈΠ½Π΅Ρ€Ρ†ΠΈΠΈ, Ρ‚ΠΎ радиус-Π²Π΅ΠΊΡ‚ΠΎΡ€ Π³ Ρ‚ΠΎΡ‡ΠΊΠΈ М эллипсоида ΠΈΠ½Π΅Ρ€Ρ†ΠΈΠΈ, располоТСнной Π½Π° Π³Π»Π°Π²Π½ΠΎΠΉ оси ΠΈΠ½Π΅Ρ€Ρ†ΠΈΠΈ, Π½Π°ΠΏΡ€ΠΈΠΌΠ΅Ρ€ оси Oz (рис. 35), Π½Π°ΠΏΡ€Π°Π²Π»Π΅Π½ ΠΏΠΎ Π½ΠΎΡ€ΠΌΠ°Π»ΠΈ ΠΊ эллипсоиду, Ρ‚. Π΅. ΠΏΠ°Ρ€Π°Π»Π»Π΅Π»ΡŒΠ½ΠΎ Π²Π΅ΠΊΡ‚ΠΎΡ€Ρƒ grad Ρ„, ΠΊΠΎΡ‚ΠΎΡ€Ρ‹ΠΉ, согласно Π΅Π³ΠΎ ΠΎΠΏΡ€Π΅Π΄Π΅Π»Π΅Π½ΠΈΡŽ, вычисляСтся ΠΏΠΎ Ρ„ΠΎΡ€ΠΌΡƒΠ»Π΅  [c.276]

Π›Π΅Π³ΠΊΠΎ Π²ΠΈΠ΄Π΅Ρ‚ΡŒ, Ρ‡Ρ‚ΠΎ Π² Ρ‚Π΅Ρ… случаях, ΠΊΠΎΠ³Π΄Π° ΠΎΠ΄Π½Π° ось систСмы ΠΊΠΎΠΎΡ€Π΄ΠΈΠ½Π°Ρ‚ совпадаСт с ΠΎΠ΄Π½ΠΎΠΉ ΠΈΠ· Π³Π»Π°Π²Π½Ρ‹Ρ… осСй ΠΈΠ½Π΅Ρ€Ρ†ΠΈΠΈ, Π΄Π²Π° ΡΠΎΠΎΡ‚Π²Π΅Ρ‚ΡΡ‚Π²ΡƒΡŽΡ‰ΠΈΡ… Ρ†Π΅Π½Ρ‚Ρ€ΠΎΠ±Π΅ΠΆΠ½Ρ‹Ρ… ΠΌΠΎΠΌΠ΅Π½Ρ‚Π° ΠΈΠ½Π΅Ρ€Ρ†ΠΈΠΈ ΠΎΠ±Ρ€Π°Ρ‰Π°ΡŽΡ‚ΡΡ Π² Π½ΡƒΠ»ΡŒ. Π”Π΅ΠΉΡΡ‚Π²ΠΈΡ‚Π΅Π»ΡŒΠ½ΠΎ, Π² Ρ‚ΠΎΡ‡ΠΊΠ΅ пСрСсСчСния Π³Π»Π°Π²Π½ΠΎΠΉ оси с ΠΏΠΎΠ²Π΅Ρ€Ρ…Π½ΠΎΡΡ‚ΡŒΡŽ эллипсоида радиус-Π²Π΅ΠΊΡ‚ΠΎΡ€, ΠΏΡ€ΠΎΠ²Π΅Π΄Π΅Π½Π½Ρ‹ΠΉ ΠΈΠ· Π½Π°Ρ‡Π°Π»Π° ΠΊΠΎΠΎΡ€Π΄ΠΈΠ½Π°Ρ‚, ΠΈ ΠΎΡ€Ρ‚ Π½ΠΎΡ€ΠΌΠ°Π»ΠΈ ΠΊ повСрхности эллипсоида ΠΊΠΎΠ»Π»ΠΈΠ½Π΅Π°Ρ€ΠΈΡ‹ (рис. 13).  [c.81]

Yt Мп(Ρƒ1 +Π³1),ΠΏΡ€Π΅Π΄.ставляСг собой сумму ΠΏΡ€ΠΎΠΈΠ·Π²Π΅Π΄Π΅Π½ΠΈΠΉ ΠΊΠ°ΠΆΠ΄ΠΎΠΉ массы Π½Π° ΠΊΠ²Π°Π΄Ρ€Π°Ρ‚ Π΅Π΅ расстояния ΠΎΡ‚ оси вращСния поэтому ΠΌΡ‹ Π΅Π³ΠΎ Π½Π°Π·Ρ‹Π²Π°Π΅ΠΌ ΠΌΠΎΠΌΠ΅Π½Ρ‚ΠΎΠΌ ΠΈΠ½Π΅Ρ€Ρ†ΠΈΠΈ ΠΎΡ‚Π½ΠΎΡΠΈΡ‚Π΅Π»ΡŒΠ½ΠΎ оси . Если Ρ€(Π³) прСдставляСт собой ΠΏΠ»ΠΎΡ‚Π½ΠΎΡΡ‚ΡŒ Ρ‚Π΅Π»Π° Π² Ρ‚ΠΎΡ‡ΠΊΠ΅, радиус-Π²Π΅ΠΊΡ‚ΠΎΡ€  [c.248]

РСшСниС. НайдСм Π²Π΅Π»ΠΈΡ‡ΠΈΠ½Ρƒ радиуса-Π²Π΅ΠΊΡ‚ΠΎΡ€Π° Ρ†Π΅Π½Ρ‚Ρ€Π° масс R= = 0Π‘ ΠΊΠ°ΠΊ Ρ„ΡƒΠ½ΠΊΡ†ΠΈΡŽ ΠΎΠ±ΠΎΠ±Ρ‰Π΅Π½Π½ΠΎΠΉ ΠΊΠΎΠΎΡ€Π΄ΠΈΠ½Π°Ρ‚Ρ‹ Ρ„, Ρ„ β€” ΡƒΠ³ΠΎΠ» ΠΌΠ΅ΠΆΠ΄Ρƒ Π²Π΅Ρ€Ρ‚ΠΈΠΊΠ°Π»ΡŒΡŽ ΠΈ ΠΎΡ‚Ρ€Π΅Π·ΠΊΠΎΠΌ ОБ (рис. 3.18). ΠŸΡƒΡΡ‚ΡŒ Π°, Π¬ β€” стороны ΡƒΠ³Π»Π°, Π¬>Π°. Π’ΠΎΠ³Π΄Π° / =72Π£Π° Π§-Π¬ (Π°+6) . ΠœΠΎΠΌΠ΅Π½Ρ‚ ΠΈΠ½Π΅Ρ€Ρ†ΠΈΠΈ ΠΎΡ‚Π½ΠΎΡΠΈΡ‚Π΅Π»ΡŒΠ½ΠΎ осп, проходящСй Ρ‡Π΅Ρ€Π΅Π· Ρ‚ΠΎΡ‡ΠΊΡƒ 0,/gg = β€” (Π° + 6 ) (Π° +. Ки-  [c.215]


ЭлСмСнтарная ΠΌΠ°Ρ‚Π΅ΠΌΠ°Ρ‚ΠΈΠΊΠ°

ΠŸΡƒΡΡ‚ΡŒΒ   –  ΡƒΠ³ΠΎΠ» ΠΌΠ΅ΠΆΠ΄Ρƒ ΠΏΠΎΠ΄Π²ΠΈΠΆΠ½Ρ‹ΠΌ радиус-Π²Π΅ΠΊΡ‚ΠΎΡ€ΠΎΠΌΒ  OM= { x, y} Β ΠΈ Π΅Π³ΠΎ Π½Π°Ρ‡Π°Π»ΡŒΠ½Ρ‹ΠΌΒ  ΠΏΠΎΠ»ΠΎΠΆΠ΅Π½ΠΈΠ΅ΠΌΒ  OA.

Β Β Β Β  Π°) Бинусом  ΡƒΠ³Π»Π°Β Β    называСтся ΠΎΡ‚Π½ΠΎΡˆΠ΅Π½ΠΈΠ΅ ΠΎΡ€Π΄ΠΈΠ½Π°Ρ‚Ρ‹Β  yΒ  ΠΊΠΎΠ½Ρ†Π° ΠΏΠΎΠ΄Π²ΠΈΠΆΠ½ΠΎΠ³ΠΎΒ  радиус-Π²Π΅ΠΊΡ‚ΠΎΡ€Π°Β  r = OMΒ  Β ΠΊΒ  Π΄Π»ΠΈΠ½Π΅Β Β  r = | rΒ  | этого радиус-Π²Π΅ΠΊΡ‚ΠΎΡ€Π°, Ρ‚.Π΅.Β 

Π±)Β  ΠšΠΎΡΠΈΠ½ΡƒΡΠΎΠΌ ΡƒΠ³Π»Π°Β   называСтся ΠΎΡ‚Π½ΠΎΡˆΠ΅Π½ΠΈΠ΅ абсциссы  xΒ  ΠΊΠΎΠ½Ρ†Π° ΠΏΠΎΠ΄Π²ΠΈΠΆΠ½ΠΎΠ³ΠΎ радиус-Π²Π΅ΠΊΡ‚ΠΎΡ€Π°Β  r = OMΒ  Β ΠΊ Π΄Π»ΠΈΠ½Π΅Β Β  r = | rΒ  |Β  этого радиус-Π²Π΅ΠΊΡ‚ΠΎΡ€Π°, Ρ‚. Π΅.

Β Β Β Β  Π²)Β  ВангСнсом ΡƒΠ³Π»Π°  называСтся ΠΎΡ‚Π½ΠΎΡˆΠ΅Π½ΠΈΠ΅ ΠΎΡ€Π΄ΠΈΠ½Π°Ρ‚Ρ‹Β  yΒ  ΠΊ абсциссС  x
ΠΊΠΎΠ½Ρ†Π° ΠΏΠΎΠ΄Π²ΠΈΠΆΠ½ΠΎΠ³ΠΎ радиус-Π²Π΅ΠΊΡ‚ΠΎΡ€Π°Β Β  OM .Β  Ρ‚.Π΅.

Β Β Β Β  Π³)Β  ΠšΠΎΡ‚Π°Π½Π³Π΅Π½ΡΠΎΠΌΒ  Β ΡƒΠ³Π»Π°Β    называСтся ΠΎΡ‚Π½ΠΎΡˆΠ΅Π½ΠΈΠ΅ абсциссы  xΒ  ΠΊ ΠΎΡ€Π΄ΠΈΠ½Π°Ρ‚Π΅Β  y
ΠΊΠΎΠ½Ρ†Π° ΠΏΠΎΠ΄Π²ΠΈΠΆΠ½ΠΎΠ³ΠΎ радиус-Π²Π΅ΠΊΡ‚ΠΎΡ€Π°Β Β  Β OM ,Β  Ρ‚.Π΅. Β 

Β Β Β Β Β  Π΄) Π€ΡƒΠ½ΠΊΡ†ΠΈΠΈΒ  сСканс  ΠΈΒ  косСканс ΠΎΠΏΡ€Π΅Π΄Π΅Π»ΡΡŽΡ‚ΡΡ ΡΠΎΠΎΡ‚Π½ΠΎΡˆΠ΅Π½ΠΈΡΠΌΠΈ

Β Β Β Β  ΠŸΠΎΠ΄Ρ‡Π΅Ρ€ΠΊΠ½Π΅ΠΌ, Ρ‡Ρ‚ΠΎ ΠΎΡ‚Π½ΠΎΡˆΠ΅Π½ΠΈΡ

зависят Ρ‚ΠΎΠ»ΡŒΠΊΠΎ ΠΎΡ‚ Π²Π΅Π»ΠΈΡ‡ΠΈΠ½Ρ‹Β  ΡƒΠ³Π»Π°Β  Β Β Β ΠΈ Π½Π΅ зависят ΠΎΡ‚ Π΄Π»ΠΈΠ½Ρ‹Β  r радиус-Π²Π΅ΠΊΡ‚ΠΎΡ€Π° OM.Β  Π­Ρ‚ΠΎ ΠΎΠ·Π½Π°Ρ‡Π°Π΅Ρ‚,Β  Ρ‡Ρ‚ΠΎ тригономСтричСскиС Ρ„ΡƒΠ½ΠΊΡ†ΠΈΠΈΒ 

ΡΠ²Π»ΡΡŽΡ‚ΡΡ функциями Ρ‚ΠΎΠ»ΡŒΠΊΠΎ ΡƒΠ³Π»Π°Β  .Β  ΠŸΡ€ΠΈ этом ΡƒΠ³ΠΎΠ»Β    часто Π½Π°Π·Ρ‹Π²Π°ΡŽΡ‚ Π°Ρ€Π³ΡƒΠΌΠ΅Π½Ρ‚ΠΎΠΌ тригономСтричСских Ρ„ΡƒΠ½ΠΊΡ†ΠΈΠΉ.
ΠŸΡ€ΠΈ вычислСнии тригономСтричСских Ρ„ΡƒΠ½ΠΊΡ†ΠΈΠΉ ΠΌΠΎΠΆΠ½ΠΎ ΠΏΠΎΠ»ΡŒΠ·ΠΎΠ²Π°Ρ‚ΡŒΡΡ ΠΏΠΎΠ΄Π²ΠΈΠΆΠ½Ρ‹ΠΌΠΈΒ  радиус-Π²Π΅ΠΊΡ‚ΠΎΡ€Π°ΠΌΠΈ Π΄Π»ΠΈΠ½Ρ‹Β  r = 1. Β Β  ΠšΠΎΠ½Ρ†Ρ‹ Ρ‚Π°ΠΊΠΈΡ… Π²Π΅ΠΊΡ‚ΠΎΡ€ΠΎΠ² Π»Π΅ΠΆΠ°Ρ‚ Π½Π° Π΅Π΄ΠΈΠ½ΠΈΡ‡Π½ΠΎΠΉ окруТности   .Β Β  Π’ этом случаС

Π’Π΅ΠΊΡ‚ΠΎΡ€Π½ΠΎΠ΅ ΠŸΡ€ΠΎΠΈΠ·Π²Π΅Π΄Π΅Π½ΠΈΠ΅ Π’Π΅ΠΊΡ‚ΠΎΡ€ΠΎΠ². Бвойства, ΠΎΠΏΡ€Π΅Π΄Π΅Π»Π΅Π½ΠΈΠ΅

Π‘Ρ‚Π°Ρ‚ΡŒΡ находится Π½Π° ΠΏΡ€ΠΎΠ²Π΅Ρ€ΠΊΠ΅ Ρƒ мСтодистов Skysmart.
Если Π²Ρ‹ Π·Π°ΠΌΠ΅Ρ‚ΠΈΠ»ΠΈ ΠΎΡˆΠΈΠ±ΠΊΡƒ, сообщитС ΠΎΠ± этом Π² ΠΎΠ½Π»Π°ΠΉΠ½-Ρ‡Π°Ρ‚ (Π² ΠΏΡ€Π°Π²ΠΎΠΌ Π½ΠΈΠΆΠ½Π΅ΠΌ ΡƒΠ³Π»Ρƒ экрана).

ΠžΠΏΡ€Π΅Π΄Π΅Π»Π΅Π½ΠΈΠ΅ Π²Π΅ΠΊΡ‚ΠΎΡ€Π½ΠΎΠ³ΠΎ произвСдСния

БистСма ΠΊΠΎΠΎΡ€Π΄ΠΈΠ½Π°Ρ‚ β€” способ ΠΎΠΏΡ€Π΅Π΄Π΅Π»ΠΈΡ‚ΡŒ ΠΏΠΎΠ»ΠΎΠΆΠ΅Π½ΠΈΠ΅ ΠΈ ΠΏΠ΅Ρ€Π΅ΠΌΠ΅Ρ‰Π΅Π½ΠΈΠ΅ Ρ‚ΠΎΡ‡ΠΊΠΈ ΠΈΠ»ΠΈ Ρ‚Π΅Π»Π° с ΠΏΠΎΠΌΠΎΡ‰ΡŒΡŽ чисСл ΠΈΠ»ΠΈ Π΄Ρ€ΡƒΠ³ΠΈΡ… символов.

ΠšΠΎΠΎΡ€Π΄ΠΈΠ½Π°Ρ‚Ρ‹ β€” это ΡΠΎΠ²ΠΎΠΊΡƒΠΏΠ½ΠΎΡΡ‚ΡŒ чисСл, ΠΊΠΎΡ‚ΠΎΡ€Ρ‹Π΅ ΠΎΠΏΡ€Π΅Π΄Π΅Π»ΡΡŽΡ‚ ΠΏΠΎΠ»ΠΎΠΆΠ΅Π½ΠΈΠ΅ ΠΊΠ°ΠΊΠΎΠ³ΠΎ-Π»ΠΈΠ±ΠΎ ΠΎΠ±ΡŠΠ΅ΠΊΡ‚Π° Π½Π° прямой, плоскости, повСрхности ΠΈΠ»ΠΈ Π² пространствС. Как Π½Π°ΠΉΡ‚ΠΈ ΠΊΠΎΠΎΡ€Π΄ΠΈΠ½Π°Ρ‚Ρ‹ Ρ‚ΠΎΡ‡ΠΊΠΈ ΠΌΡ‹ рассказали Π² этой ΡΡ‚Π°Ρ‚ΡŒΠ΅.

Бкаляр β€” это Π²Π΅Π»ΠΈΡ‡ΠΈΠ½Π°, которая ΠΏΠΎΠ»Π½ΠΎΡΡ‚ΡŒΡŽ опрСдСляСтся Π² любой ΠΊΠΎΠΎΡ€Π΄ΠΈΠ½Π°Ρ‚Π½ΠΎΠΉ систСмС ΠΎΠ΄Π½ΠΈΠΌ числом ΠΈΠ»ΠΈ Ρ„ΡƒΠ½ΠΊΡ†ΠΈΠ΅ΠΉ.

Π’Π΅ΠΊΡ‚ΠΎΡ€ β€” Π½Π°ΠΏΡ€Π°Π²Π»Π΅Π½Π½Ρ‹ΠΉ ΠΎΡ‚Ρ€Π΅Π·ΠΎΠΊ прямой, для ΠΊΠΎΡ‚ΠΎΡ€ΠΎΠ³ΠΎ ΡƒΠΊΠ°Π·Π°Π½ΠΎ, какая Ρ‚ΠΎΡ‡ΠΊΠ° являСтся Π½Π°Ρ‡Π°Π»ΠΎΠΌ, Π° какая β€” ΠΊΠΎΠ½Ρ†ΠΎΠΌ.


Π’Π΅ΠΊΡ‚ΠΎΡ€ с Π½Π°Ρ‡Π°Π»ΠΎΠΌ Π² Ρ‚ΠΎΡ‡ΠΊΠ΅ A ΠΈ ΠΊΠΎΠ½Ρ†ΠΎΠΌ Π² Ρ‚ΠΎΡ‡ΠΊΠ΅ B принято ΠΎΠ±ΠΎΠ·Π½Π°Ρ‡Π°Ρ‚ΡŒ ΠΊΠ°ΠΊ β†’AB. Π’Π΅ΠΊΡ‚ΠΎΡ€Ρ‹ Ρ‚Π°ΠΊΠΆΠ΅ ΠΌΠΎΠΆΠ½ΠΎ ΠΎΠ±ΠΎΠ·Π½Π°Ρ‡Π°Ρ‚ΡŒ ΠΌΠ°Π»Ρ‹ΠΌΠΈ латинскими Π±ΡƒΠΊΠ²Π°ΠΌΠΈ со стрСлкой ΠΈΠ»ΠΈ Ρ‡Π΅Ρ€Ρ‚ΠΎΡ‡ΠΊΠΎΠΉ Π½Π°Π΄ Π½ΠΈΠΌΠΈ, Π²ΠΎΡ‚ Ρ‚Π°ΠΊ: β†’a.

ΠšΠΎΠ»Π»ΠΈΠ½Π΅Π°Ρ€Π½ΠΎΡΡ‚ΡŒ β€” ΠΎΡ‚Π½ΠΎΡˆΠ΅Π½ΠΈΠ΅ ΠΏΠ°Ρ€Π°Π»Π»Π΅Π»ΡŒΠ½ΠΎΡΡ‚ΠΈ Π²Π΅ΠΊΡ‚ΠΎΡ€ΠΎΠ². Π”Π²Π° Π½Π΅Π½ΡƒΠ»Π΅Π²Ρ‹Ρ… Π²Π΅ΠΊΡ‚ΠΎΡ€Π° Π½Π°Π·Ρ‹Π²Π°ΡŽΡ‚ΡΡ ΠΊΠΎΠ»Π»ΠΈΠ½Π΅Π°Ρ€Π½Ρ‹ΠΌΠΈ, Ссли ΠΎΠ½ΠΈ Π»Π΅ΠΆΠ°Ρ‚ Π½Π° ΠΏΠ°Ρ€Π°Π»Π»Π΅Π»ΡŒΠ½Ρ‹Ρ… прямых ΠΈΠ»ΠΈ Π½Π° ΠΎΠ΄Π½ΠΎΠΉ прямой.

ΠŸΡ€ΠΎΡ‰Π΅ говоря это Β«ΠΏΠ°Ρ€Π°Π»Π»Π΅Π»ΡŒΠ½Ρ‹Π΅Β» Π²Π΅ΠΊΡ‚ΠΎΡ€Ρ‹. ΠšΠΎΠ»Π»ΠΈΠ½Π΅Π°Ρ€Π½Ρ‹Π΅ Π²Π΅ΠΊΡ‚ΠΎΡ€Ρ‹ ΠΌΠΎΠ³ΡƒΡ‚ Π±Ρ‹Ρ‚ΡŒ ΠΎΠ΄ΠΈΠ½Π°ΠΊΠΎΠ²ΠΎ Π½Π°ΠΏΡ€Π°Π²Π»Π΅Π½Ρ‹ ΠΈΠ»ΠΈ ΠΏΡ€ΠΎΡ‚ΠΈΠ²ΠΎΠΏΠΎΠ»ΠΎΠΆΠ½ΠΎ Π½Π°ΠΏΡ€Π°Π²Π»Π΅Π½Ρ‹. ОсновноС ΠΎΠ±ΠΎΠ·Π½Π°Ρ‡Π΅Π½ΠΈΠ΅ β€” β†’a || β†’b. Π‘ΠΎΠ½Π°ΠΏΡ€Π°Π²Π»Π΅Π½Π½Ρ‹Π΅ ΠΊΠΎΠ»Π»ΠΈΠ½Π΅Π°Ρ€Π½Ρ‹Π΅ Π²Π΅ΠΊΡ‚ΠΎΡ€Ρ‹ ΠΎΠ±ΠΎΠ·Π½Π°Ρ‡Π°ΡŽΡ‚ΡΡ Ρ‚Π°ΠΊ β†’a ↑↑ β†’b, ΠΏΡ€ΠΎΡ‚ΠΈΠ²ΠΎΠΏΠΎΠ»ΠΎΠΆΠ½ΠΎ Π½Π°ΠΏΡ€Π°Π²Π»Π΅Π½Π½Ρ‹Π΅ β€” β†’a ↑↓ β†’b.

ΠŸΡ€Π΅ΠΆΠ΄Π΅ Ρ‡Π΅ΠΌ Π΄Π°Ρ‚ΡŒ ΠΎΠΏΡ€Π΅Π΄Π΅Π»Π΅Π½ΠΈΠ΅ Π²Π΅ΠΊΡ‚ΠΎΡ€Π½ΠΎΠ³ΠΎ произвСдСния, разбСрСмся с ΠΎΡ€ΠΈΠ΅Π½Ρ‚Π°Ρ†ΠΈΠ΅ΠΉ упорядочСнной Ρ‚Ρ€ΠΎΠΉΠΊΠΈ Π²Π΅ΠΊΡ‚ΠΎΡ€ΠΎΠ² β†’a, β†’b, β†’c Π² Ρ‚Ρ€Π΅Ρ…ΠΌΠ΅Ρ€Π½ΠΎΠΌ пространствС.

ΠžΡ‚Π»ΠΎΠΆΠΈΠΌ Π²Π΅ΠΊΡ‚ΠΎΡ€Ρ‹ β†’a, β†’b, β†’c ΠΎΡ‚ ΠΎΠ΄Π½ΠΎΠΉ Ρ‚ΠΎΡ‡ΠΊΠΈ. Π’ зависимости ΠΎΡ‚ направлСния Π²Π΅ΠΊΡ‚ΠΎΡ€Π° β†’c Ρ‚Ρ€ΠΎΠΉΠΊΠ° β†’a, β†’b, β†’c ΠΌΠΎΠΆΠ΅Ρ‚ Π±Ρ‹Ρ‚ΡŒ ΠΏΡ€Π°Π²ΠΎΠΉ ΠΈΠ»ΠΈ Π»Π΅Π²ΠΎΠΉ.

ΠŸΠΎΡΠΌΠΎΡ‚Ρ€ΠΈΠΌ с ΠΊΠΎΠ½Ρ†Π° Π²Π΅ΠΊΡ‚ΠΎΡ€Π° β†’c Π½Π° Ρ‚ΠΎ, ΠΊΠ°ΠΊ происходит ΠΊΡ€Π°Ρ‚Ρ‡Π°ΠΉΡˆΠΈΠΉ ΠΏΠΎΠ²ΠΎΡ€ΠΎΡ‚ ΠΎΡ‚ Π²Π΅ΠΊΡ‚ΠΎΡ€Π° β†’a ΠΊ β†’b. Если ΠΊΡ€Π°Ρ‚Ρ‡Π°ΠΉΡˆΠΈΠΉ ΠΏΠΎΠ²ΠΎΡ€ΠΎΡ‚ происходит ΠΏΡ€ΠΎΡ‚ΠΈΠ² часовой стрСлки, Ρ‚ΠΎ Ρ‚Ρ€ΠΎΠΉΠΊΠ° Π²Π΅ΠΊΡ‚ΠΎΡ€ΠΎΠ² β†’a, β†’b, β†’c называСтся ΠΏΡ€Π°Π²ΠΎΠΉ, ΠΏΠΎ часовой стрСлкС β€” Π»Π΅Π²ΠΎΠΉ.



Π’Π΅ΠΏΠ΅Ρ€ΡŒ возьмСм Π΄Π²Π° Π½Π΅ΠΊΠΎΠ»Π»ΠΈΠ½Π΅Π°Ρ€Π½Ρ‹Ρ… Π²Π΅ΠΊΡ‚ΠΎΡ€Π° β†’a ΠΈ β†’b. ΠžΡ‚Π»ΠΎΠΆΠΈΠΌ ΠΎΡ‚ Ρ‚ΠΎΡ‡ΠΊΠΈ А Π²Π΅ΠΊΡ‚ΠΎΡ€Ρ‹ β†’AB = β†’a ΠΈ β†’AC = β†’b. ΠŸΠΎΡΡ‚Ρ€ΠΎΠΈΠΌ Π½Π΅ΠΊΠΎΡ‚ΠΎΡ€Ρ‹ΠΉ Π²Π΅ΠΊΡ‚ΠΎΡ€ β†’AD = β†’c, пСрпСндикулярный ΠΎΠ΄Π½ΠΎΠ²Ρ€Π΅ΠΌΠ΅Π½Π½ΠΎ ΠΈ β†’AB ΠΈ β†’AC.

ΠžΡ‡Π΅Π²ΠΈΠ΄Π½ΠΎ, Ρ‡Ρ‚ΠΎ ΠΏΡ€ΠΈ построСнии Π²Π΅ΠΊΡ‚ΠΎΡ€Π° β†’AD = β†’c ΠΌΡ‹ ΠΌΠΎΠΆΠ΅ΠΌ ΠΏΠΎΡΡ‚ΡƒΠΏΠΈΡ‚ΡŒ ΠΏΠΎ-Ρ€Π°Π·Π½ΠΎΠΌΡƒ, Ссли Π·Π°Π΄Π°Π΄ΠΈΠΌ Π΅ΠΌΡƒ Π»ΠΈΠ±ΠΎ ΠΎΠ΄Π½ΠΎ Π½Π°ΠΏΡ€Π°Π²Π»Π΅Π½ΠΈΠ΅, Π»ΠΈΠ±ΠΎ ΠΏΡ€ΠΎΡ‚ΠΈΠ²ΠΎΠΏΠΎΠ»ΠΎΠΆΠ½ΠΎΠ΅.


Π’ зависимости ΠΎΡ‚ направлСния Π²Π΅ΠΊΡ‚ΠΎΡ€Π° β†’AD = β†’c упорядочСнная Ρ‚Ρ€ΠΎΠΉΠΊΠ° Π²Π΅ΠΊΡ‚ΠΎΡ€ΠΎΠ² β†’a, β†’b, β†’c ΠΌΠΎΠΆΠ΅Ρ‚ Π±Ρ‹Ρ‚ΡŒ ΠΏΡ€Π°Π²ΠΎΠΉ ΠΈΠ»ΠΈ Π»Π΅Π²ΠΎΠΉ.

И сСйчас ΠΌΡ‹ подошли ΠΊ ΠΎΠΏΡ€Π΅Π΄Π΅Π»Π΅Π½ΠΈΡŽ Π²Π΅ΠΊΡ‚ΠΎΡ€Π½ΠΎΠ³ΠΎ произвСдСния. Оно даСтся для Π΄Π²ΡƒΡ… Π²Π΅ΠΊΡ‚ΠΎΡ€ΠΎΠ², ΠΊΠΎΡ‚ΠΎΡ€Ρ‹Π΅ Π·Π°Π΄Π°Π½Ρ‹ Π² ΠΏΡ€ΡΠΌΠΎΡƒΠ³ΠΎΠ»ΡŒΠ½ΠΎΠΉ систСмС ΠΊΠΎΠΎΡ€Π΄ΠΈΠ½Π°Ρ‚ Ρ‚Ρ€Π΅Ρ…ΠΌΠ΅Ρ€Π½ΠΎΠ³ΠΎ пространства.

Π•Ρ‰Π΅ Π½Π΅ устали ΠΎΡ‚ Ρ‚Π΅ΠΎΡ€ΠΈΠΈ? Онлайн-школа Skysmart ΠΏΡ€Π΅Π΄Π»Π°Π³Π°Π΅Ρ‚ ΠΎΠ±ΡƒΡ‡Π΅Π½ΠΈΠ΅ Π½Π° курсах ΠΏΠΎ ΠΌΠ°Ρ‚Π΅ΠΌΠ°Ρ‚ΠΈΠΊΠ΅ β€”Β ΠΌΠ½ΠΎΠ³ΠΎ ΠΏΡ€Π°ΠΊΡ‚ΠΈΠΊΠΈ ΠΈ ΠΏΠΎΠ΄Π΄Π΅Ρ€ΠΆΠΊΠ° Π²Π½ΠΈΠΌΠ°Ρ‚Π΅Π»ΡŒΠ½Ρ‹Ρ… ΠΏΡ€Π΅ΠΏΠΎΠ΄Π°Π²Π°Ρ‚Π΅Π»Π΅ΠΉ!

Π’Π΅ΠΊΡ‚ΠΎΡ€Π½Ρ‹ΠΌ ΠΏΡ€ΠΎΠΈΠ·Π²Π΅Π΄Π΅Π½ΠΈΠ΅ΠΌ Π΄Π²ΡƒΡ… Π²Π΅ΠΊΡ‚ΠΎΡ€ΠΎΠ² β†’a ΠΈ β†’b, ΠΊΠΎΡ‚ΠΎΡ€Ρ‹Π΅ Π·Π°Π΄Π°Π½Ρ‹ Π² ΠΏΡ€ΡΠΌΠΎΡƒΠ³ΠΎΠ»ΡŒΠ½ΠΎΠΉ систСмС ΠΊΠΎΠΎΡ€Π΄ΠΈΠ½Π°Ρ‚ Ρ‚Ρ€Π΅Ρ…ΠΌΠ΅Ρ€Π½ΠΎΠ³ΠΎ пространства, называСтся Ρ‚Π°ΠΊΠΎΠΉ Π²Π΅ΠΊΡ‚ΠΎΡ€ β†’c, Ρ‡Ρ‚ΠΎ:

  • ΠΎΠ½ являСтся Π½ΡƒΠ»Π΅Π²Ρ‹ΠΌ, Ссли Π²Π΅ΠΊΡ‚ΠΎΡ€Ρ‹ β†’a ΠΈ β†’b ΠΊΠΎΠ»Π»ΠΈΠ½Π΅Π°Ρ€Π½Ρ‹;
  • ΠΎΠ½ пСрпСндикулярСн ΠΈ Π²Π΅ΠΊΡ‚ΠΎΡ€Ρƒ β†’a ΠΈ Π²Π΅ΠΊΡ‚ΠΎΡ€Ρƒ β†’b;
  • Π΄Π»ΠΈΠ½Π° Π²Π΅ΠΊΡ‚ΠΎΡ€Π½ΠΎΠ³ΠΎ произвСдСния Ρ€Π°Π²Π½Π° ΠΏΡ€ΠΎΠΈΠ·Π²Π΅Π΄Π΅Π½ΠΈΡŽ Π΄Π»ΠΈΠ½ Π²Π΅ΠΊΡ‚ΠΎΡ€ΠΎΠ² β†’a ΠΈ β†’b Π½Π° синус ΡƒΠ³Π»Π° ΠΌΠ΅ΠΆΠ΄Ρƒ Π½ΠΈΠΌΠΈ
  • Ρ‚Ρ€ΠΎΠΉΠΊΠ° Π²Π΅ΠΊΡ‚ΠΎΡ€ΠΎΠ² β†’a, β†’b, β†’c ΠΎΡ€ΠΈΠ΅Π½Ρ‚ΠΈΡ€ΠΎΠ²Π°Π½Π° Ρ‚Π°ΠΊ ΠΆΠ΅, ΠΊΠ°ΠΊ ΠΈ заданная систСма ΠΊΠΎΠΎΡ€Π΄ΠΈΠ½Π°Ρ‚.

Π’Π΅ΠΊΡ‚ΠΎΡ€Π½Ρ‹ΠΌ ΠΏΡ€ΠΎΠΈΠ·Π²Π΅Π΄Π΅Π½ΠΈΠ΅ΠΌ Π²Π΅ΠΊΡ‚ΠΎΡ€Π° β†’a Π½Π° Π²Π΅ΠΊΡ‚ΠΎΡ€ β†’b называСтся Π²Π΅ΠΊΡ‚ΠΎΡ€ β†’c, Π΄Π»ΠΈΠ½Π° ΠΊΠΎΡ‚ΠΎΡ€ΠΎΠ³ΠΎ числСнно Ρ€Π°Π²Π½Π° ΠΏΠ»ΠΎΡ‰Π°Π΄ΠΈ ΠΏΠ°Ρ€Π°Π»Π»Π΅Π»ΠΎΠ³Ρ€Π°ΠΌΠΌΠ° построСнного Π½Π° Π²Π΅ΠΊΡ‚ΠΎΡ€Π°Ρ… β†’a ΠΈ β†’b, пСрпСндикулярный ΠΊ плоскости этих Π²Π΅ΠΊΡ‚ΠΎΡ€ΠΎΠ² ΠΈ Π½Π°ΠΏΡ€Π°Π²Π»Π΅Π½Π½Ρ‹ΠΉ Ρ‚Π°ΠΊ, Ρ‡Ρ‚ΠΎΠ±Ρ‹ наимСньшСС Π²Ρ€Π°Ρ‰Π΅Π½ΠΈΠ΅ ΠΎΡ‚ β†’a ΠΊ β†’b Π²ΠΎΠΊΡ€ΡƒΠ³ Π²Π΅ΠΊΡ‚ΠΎΡ€Π° c ΠΎΡΡƒΡ‰Π΅ΡΡ‚Π²Π»ΡΠ»ΠΎΡΡŒ ΠΏΡ€ΠΎΡ‚ΠΈΠ² часовой стрСлки, Ссли ΡΠΌΠΎΡ‚Ρ€Π΅Ρ‚ΡŒ с ΠΊΠΎΠ½Ρ†Π° Π²Π΅ΠΊΡ‚ΠΎΡ€Π° β†’c.


Π’Π΅ΠΊΡ‚ΠΎΡ€Π½ΠΎΠ΅ ΠΏΡ€ΠΎΠΈΠ·Π²Π΅Π΄Π΅Π½ΠΈΠ΅ Π΄Π²ΡƒΡ… Π²Π΅ΠΊΡ‚ΠΎΡ€ΠΎΠ² a = {ax; ay; az} ΠΈ b = {bx; by; bz} Π² Π΄Π΅ΠΊΠ°Ρ€Ρ‚ΠΎΠ²ΠΎΠΉ систСмС ΠΊΠΎΠΎΡ€Π΄ΠΈΠ½Π°Ρ‚ β€” это Π²Π΅ΠΊΡ‚ΠΎΡ€, Π·Π½Π°Ρ‡Π΅Π½ΠΈΠ΅ ΠΊΠΎΡ‚ΠΎΡ€ΠΎΠ³ΠΎ ΠΌΠΎΠΆΠ½ΠΎ Π²Ρ‹Ρ‡ΠΈΡΠ»ΠΈΡ‚ΡŒ, ΠΈΡΠΏΠΎΠ»ΡŒΠ·ΡƒΡ Ρ„ΠΎΡ€ΠΌΡƒΠ»Ρ‹ вычислСния Π²Π΅ΠΊΡ‚ΠΎΡ€Π½ΠΎΠ³ΠΎ произвСдСния Π²Π΅ΠΊΡ‚ΠΎΡ€ΠΎΠ²:



Π’Π΅ΠΊΡ‚ΠΎΡ€Π½ΠΎΠ΅ ΠΏΡ€ΠΎΠΈΠ·Π²Π΅Π΄Π΅Π½ΠΈΠ΅ Π²Π΅ΠΊΡ‚ΠΎΡ€ΠΎΠ² β†’a ΠΈ β†’b обозначаСтся ΠΊΠ°ΠΊ [β†’a β€’ β†’b].

Π”Ρ€ΡƒΠ³ΠΎΠ΅ ΠΎΠΏΡ€Π΅Π΄Π΅Π»Π΅Π½ΠΈΠ΅ связано с ΠΏΡ€Π°Π²ΠΎΠΉ Ρ€ΡƒΠΊΠΎΠΉ Ρ‡Π΅Π»ΠΎΠ²Π΅ΠΊΠ°, ΠΎΡ‚ΠΊΡƒΠ΄Π° ΠΈ Π΅ΡΡ‚ΡŒ Π½Π°Π·Π²Π°Π½ΠΈΠ΅. На рисункС Ρ‚Ρ€ΠΎΠΉΠΊΠ° Π²Π΅ΠΊΡ‚ΠΎΡ€ΠΎΠ² β†’a, β†’b, [β†’a β€’ β†’b] являСтся ΠΏΡ€Π°Π²ΠΎΠΉ.


Π•Ρ‰Π΅ Π΅ΡΡ‚ΡŒ аналитичСский способ опрСдСлСния ΠΏΡ€Π°Π²ΠΎΠΉ ΠΈ Π»Π΅Π²ΠΎΠΉ Ρ‚Ρ€ΠΎΠΉΠΊΠΈ Π²Π΅ΠΊΡ‚ΠΎΡ€ΠΎΠ² β€” ΠΎΠ½ Ρ‚Ρ€Π΅Π±ΡƒΠ΅Ρ‚ задания Π² рассматриваСмом пространствС ΠΏΡ€Π°Π²ΠΎΠΉ ΠΈΠ»ΠΈ Π»Π΅Π²ΠΎΠΉ систСмы ΠΊΠΎΠΎΡ€Π΄ΠΈΠ½Π°Ρ‚, ΠΏΡ€ΠΈΡ‡Ρ‘ΠΌ Π½Π΅ ΠΎΠ±ΡΠ·Π°Ρ‚Π΅Π»ΡŒΠ½ΠΎ ΠΏΡ€ΡΠΌΠΎΡƒΠ³ΠΎΠ»ΡŒΠ½ΠΎΠΉ ΠΈ ΠΎΡ€Ρ‚ΠΎΠ½ΠΎΡ€ΠΌΠΈΡ€ΠΎΠ²Π°Π½Π½ΠΎΠΉ.

НуТно ΡΠΎΡΡ‚Π°Π²ΠΈΡ‚ΡŒ ΠΌΠ°Ρ‚Ρ€ΠΈΡ†Ρƒ, ΠΏΠ΅Ρ€Π²ΠΎΠΉ строкой ΠΊΠΎΡ‚ΠΎΡ€ΠΎΠΉ Π±ΡƒΠ΄ΡƒΡ‚ ΠΊΠΎΠΎΡ€Π΄ΠΈΠ½Π°Ρ‚Ρ‹ Π²Π΅ΠΊΡ‚ΠΎΡ€Π° β†’a, Π²Ρ‚ΠΎΡ€ΠΎΠΉ β€” Π²Π΅ΠΊΡ‚ΠΎΡ€Π° β†’b, Ρ‚Ρ€Π΅Ρ‚ΡŒΠ΅ΠΉ β€” Π²Π΅ΠΊΡ‚ΠΎΡ€Π° β†’c. Π—Π°Ρ‚Π΅ΠΌ, Π² зависимости ΠΎΡ‚ Π·Π½Π°ΠΊΠ° опрСдСлитСля этой ΠΌΠ°Ρ‚Ρ€ΠΈΡ†Ρ‹, ΠΌΠΎΠΆΠ½ΠΎ ΡΠ΄Π΅Π»Π°Ρ‚ΡŒ ΡΠ»Π΅Π΄ΡƒΡŽΡ‰ΠΈΠ΅ Π²Ρ‹Π²ΠΎΠ΄Ρ‹:

  • Если ΠΎΠΏΡ€Π΅Π΄Π΅Π»ΠΈΡ‚Π΅Π»ΡŒ ΠΏΠΎΠ»ΠΎΠΆΠΈΡ‚Π΅Π»Π΅Π½, Ρ‚ΠΎ Ρ‚Ρ€ΠΎΠΉΠΊΠ° Π²Π΅ΠΊΡ‚ΠΎΡ€ΠΎΠ² ΠΈΠΌΠ΅Π΅Ρ‚ Ρ‚Ρƒ ΠΆΠ΅ ΠΎΡ€ΠΈΠ΅Π½Ρ‚Π°Ρ†ΠΈΡŽ, Ρ‡Ρ‚ΠΎ ΠΈ систСма ΠΊΠΎΠΎΡ€Π΄ΠΈΠ½Π°Ρ‚.
  • Если ΠΎΠΏΡ€Π΅Π΄Π΅Π»ΠΈΡ‚Π΅Π»ΡŒ ΠΎΡ‚Ρ€ΠΈΡ†Π°Ρ‚Π΅Π»Π΅Π½, Ρ‚ΠΎ Ρ‚Ρ€ΠΎΠΉΠΊΠ° Π²Π΅ΠΊΡ‚ΠΎΡ€ΠΎΠ² ΠΈΠΌΠ΅Π΅Ρ‚ ΠΎΡ€ΠΈΠ΅Π½Ρ‚Π°Ρ†ΠΈΡŽ, ΠΏΡ€ΠΎΡ‚ΠΈΠ²ΠΎΠΏΠΎΠ»ΠΎΠΆΠ½ΡƒΡŽ ΠΎΡ€ΠΈΠ΅Π½Ρ‚Π°Ρ†ΠΈΠΈ систСмы ΠΊΠΎΠΎΡ€Π΄ΠΈΠ½Π°Ρ‚.
  • Если ΠΎΠΏΡ€Π΅Π΄Π΅Π»ΠΈΡ‚Π΅Π»ΡŒ Ρ€Π°Π²Π΅Π½ Π½ΡƒΠ»ΡŽ, Ρ‚ΠΎ Π²Π΅ΠΊΡ‚ΠΎΡ€Ρ‹ ΠΊΠΎΠΌΠΏΠ»Π°Π½Π°Ρ€Π½Ρ‹ (Π»ΠΈΠ½Π΅ΠΉΠ½ΠΎ зависимы).

ΠšΠΎΠΎΡ€Π΄ΠΈΠ½Π°Ρ‚Ρ‹ Π²Π΅ΠΊΡ‚ΠΎΡ€Π½ΠΎΠ³ΠΎ произвСдСния

Рассмотрим Π²Π΅ΠΊΡ‚ΠΎΡ€Π½ΠΎΠ΅ ΠΏΡ€ΠΎΠΈΠ·Π²Π΅Π΄Π΅Π½ΠΈΠ΅ Π²Π΅ΠΊΡ‚ΠΎΡ€ΠΎΠ² Π² ΠΊΠΎΠΎΡ€Π΄ΠΈΠ½Π°Ρ‚Π°Ρ….

Π‘Ρ„ΠΎΡ€ΠΌΡƒΠ»ΠΈΡ€ΡƒΠ΅ΠΌ Π²Ρ‚ΠΎΡ€ΠΎΠ΅ ΠΎΠΏΡ€Π΅Π΄Π΅Π»Π΅Π½ΠΈΠ΅ Π²Π΅ΠΊΡ‚ΠΎΡ€Π½ΠΎΠ³ΠΎ произвСдСния, ΠΊΠΎΡ‚ΠΎΡ€ΠΎΠ΅ позволяСт Π½Π°Ρ…ΠΎΠ΄ΠΈΡ‚ΡŒ Π΅Π³ΠΎ ΠΊΠΎΠΎΡ€Π΄ΠΈΠ½Π°Ρ‚Ρ‹ ΠΏΠΎ ΠΊΠΎΠΎΡ€Π΄ΠΈΠ½Π°Ρ‚Π°ΠΌ Π·Π°Π΄Π°Π½Π½Ρ‹Ρ… Π²Π΅ΠΊΡ‚ΠΎΡ€ΠΎΠ².

Π’ ΠΏΡ€ΡΠΌΠΎΡƒΠ³ΠΎΠ»ΡŒΠ½ΠΎΠΉ систСмС ΠΊΠΎΠΎΡ€Π΄ΠΈΠ½Π°Ρ‚ Ρ‚Ρ€Π΅Ρ…ΠΌΠ΅Ρ€Π½ΠΎΠ³ΠΎ пространства Π²Π΅ΠΊΡ‚ΠΎΡ€Π½ΠΎΠ΅ ΠΏΡ€ΠΎΠΈΠ·Π²Π΅Π΄Π΅Π½ΠΈΠ΅ Π΄Π²ΡƒΡ… Π²Π΅ΠΊΡ‚ΠΎΡ€ΠΎΠ² β†’a = (ax, ay, az) ΠΈ β†’b = (bx, by, bz) Π΅ΡΡ‚ΡŒ Π²Π΅ΠΊΡ‚ΠΎΡ€


, Π³Π΄Π΅

β†’i, β†’j, β†’k β€” ΠΊΠΎΠΎΡ€Π΄ΠΈΠ½Π°Ρ‚Π½Ρ‹Π΅ Π²Π΅ΠΊΡ‚ΠΎΡ€Ρ‹.

Π­Ρ‚ΠΎ ΠΎΠΏΡ€Π΅Π΄Π΅Π»Π΅Π½ΠΈΠ΅ ΠΏΠΎΠΊΠ°Π·Ρ‹Π²Π°Π΅Ρ‚ Π½Π°ΠΌ Π²Π΅ΠΊΡ‚ΠΎΡ€Π½ΠΎΠ΅ ΠΏΡ€ΠΎΠΈΠ·Π²Π΅Π΄Π΅Π½ΠΈΠ΅ Π² ΠΊΠΎΠΎΡ€Π΄ΠΈΠ½Π°Ρ‚Π½ΠΎΠΉ Ρ„ΠΎΡ€ΠΌΠ΅.

Π’Π΅ΠΊΡ‚ΠΎΡ€Π½ΠΎΠ΅ ΠΏΡ€ΠΎΠΈΠ·Π²Π΅Π΄Π΅Π½ΠΈΠ΅ ΡƒΠ΄ΠΎΠ±Π½ΠΎ ΠΏΡ€Π΅Π΄ΡΡ‚Π°Π²Π»ΡΡ‚ΡŒ Π² Π²ΠΈΠ΄Π΅ опрСдСлитСля ΠΊΠ²Π°Π΄Ρ€Π°Ρ‚Π½ΠΎΠΉ ΠΌΠ°Ρ‚Ρ€ΠΈΡ†Ρ‹ Ρ‚Ρ€Π΅Ρ‚ΡŒΠ΅Π³ΠΎ порядка, пСрвая строка ΠΊΠΎΡ‚ΠΎΡ€ΠΎΠΉ Π΅ΡΡ‚ΡŒ ΠΎΡ€Ρ‚Ρ‹ β†’i, β†’j, β†’k, Π²ΠΎ Π²Ρ‚ΠΎΡ€ΠΎΠΉ строкС находятся ΠΊΠΎΠΎΡ€Π΄ΠΈΠ½Π°Ρ‚Ρ‹ Π²Π΅ΠΊΡ‚ΠΎΡ€Π° β†’a, Π° Π² Ρ‚Ρ€Π΅Ρ‚ΡŒΠ΅ΠΉ β€” ΠΊΠΎΠΎΡ€Π΄ΠΈΠ½Π°Ρ‚Ρ‹ Π²Π΅ΠΊΡ‚ΠΎΡ€Π° β†’b Π² Π·Π°Π΄Π°Π½Π½ΠΎΠΉ ΠΏΡ€ΡΠΌΠΎΡƒΠ³ΠΎΠ»ΡŒΠ½ΠΎΠΉ систСмС ΠΊΠΎΠΎΡ€Π΄ΠΈΠ½Π°Ρ‚:


Если Ρ€Π°Π·Π»ΠΎΠΆΠΈΠΌ этот ΠΎΠΏΡ€Π΅Π΄Π΅Π»ΠΈΡ‚Π΅Π»ΡŒ ΠΏΠΎ элСмСнтам ΠΏΠ΅Ρ€Π²ΠΎΠΉ строки, Ρ‚ΠΎ ΠΏΠΎΠ»ΡƒΡ‡ΠΈΠΌ равСнство ΠΈΠ· опрСдСлСния Π²Π΅ΠΊΡ‚ΠΎΡ€Π½ΠΎΠ³ΠΎ произвСдСния Π² ΠΊΠΎΠΎΡ€Π΄ΠΈΠ½Π°Ρ‚Π°Ρ…:


Π’Π°ΠΆΠ½ΠΎ ΠΎΡ‚ΠΌΠ΅Ρ‚ΠΈΡ‚ΡŒ, Ρ‡Ρ‚ΠΎ координатная Ρ„ΠΎΡ€ΠΌΠ° Π²Π΅ΠΊΡ‚ΠΎΡ€Π½ΠΎΠ³ΠΎ произвСдСния согласуСтся с ΠΎΠΏΡ€Π΅Π΄Π΅Π»Π΅Π½ΠΈΠ΅ΠΌ,ΠΊΠΎΡ‚ΠΎΡ€ΠΎΠ΅ ΠΌΡ‹ Π΄Π°Π»ΠΈ Π² ΠΏΠ΅Ρ€Π²ΠΎΠΌ ΠΏΡƒΠ½ΠΊΡ‚Π΅ этой ΡΡ‚Π°Ρ‚ΡŒΠΈ. Π‘ΠΎΠ»Π΅Π΅ Ρ‚ΠΎΠ³ΠΎ, эти Π΄Π²Π° опрСдСлСния Π²Π΅ΠΊΡ‚ΠΎΡ€Π½ΠΎΠ³ΠΎ произвСдСния эквивалСнтны.

Бвойства Π²Π΅ΠΊΡ‚ΠΎΡ€Π½ΠΎΠ³ΠΎ произвСдСния

Π’Π΅ΠΊΡ‚ΠΎΡ€Π½ΠΎΠ΅ ΠΏΡ€ΠΎΠΈΠ·Π²Π΅Π΄Π΅Π½ΠΈΠ΅ Π² ΠΊΠΎΠΎΡ€Π΄ΠΈΠ½Π°Ρ‚Π°Ρ… прСдставляСтся Π² Π²ΠΈΠ΄Π΅ опрСдСлитСля ΠΌΠ°Ρ‚Ρ€ΠΈΡ†Ρ‹:


На основании свойств опрСдСлитСля ΠΌΠΎΠΆΠ½ΠΎ Π»Π΅Π³ΠΊΠΎ ΠΎΠ±ΠΎΡΠ½ΠΎΠ²Π°Ρ‚ΡŒ свойства Π²Π΅ΠΊΡ‚ΠΎΡ€Π½ΠΎΠ³ΠΎ произвСдСния Π²Π΅ΠΊΡ‚ΠΎΡ€ΠΎΠ²:


  1. ΠΠ½Ρ‚ΠΈΠΊΠΎΠΌΠΌΡƒΡ‚Π°Ρ‚ΠΈΠ²Π½ΠΎΡΡ‚ΡŒ
  2. Бвойство дистрибутивности

    ΠΈΠ»ΠΈ


  3. Π‘ΠΎΡ‡Π΅Ρ‚Π°Ρ‚Π΅Π»ΡŒΠ½ΠΎΠ΅ свойство

    ΠΈΠ»ΠΈ


    , Π³Π΄Π΅ Ξ» ΠΏΡ€ΠΎΠΈΠ·Π²ΠΎΠ»ΡŒΠ½ΠΎΠ΅ Π΄Π΅ΠΉΡΡ‚Π²ΠΈΡ‚Π΅Π»ΡŒΠ½ΠΎΠ΅ число.

Для большСй ясности Π΄ΠΎΠΊΠ°ΠΆΠ΅ΠΌ свойство антикоммутативности Π²Π΅ΠΊΡ‚ΠΎΡ€Π½ΠΎΠ³ΠΎ произвСдСния.

По ΠΎΠΏΡ€Π΅Π΄Π΅Π»Π΅Π½ΠΈΡŽ


ΠΈ


Нам извСстно, Ρ‡Ρ‚ΠΎ Π·Π½Π°Ρ‡Π΅Π½ΠΈΠ΅ опрСдСлитСля ΠΌΠ°Ρ‚Ρ€ΠΈΡ†Ρ‹ измСняСтся Π½Π° ΠΏΡ€ΠΎΡ‚ΠΈΠ²ΠΎΠΏΠΎΠ»ΠΎΠΆΠ½ΠΎΠ΅, Ссли ΠΏΠ΅Ρ€Π΅ΡΡ‚Π°Π²ΠΈΡ‚ΡŒ мСстами Π΄Π²Π΅ строки, поэтому


Ρ‡Ρ‚ΠΎ Π΄ΠΎΠΊΠ°Π·Ρ‹Π²Π°Π΅Ρ‚ свойство антикоммутативности Π²Π΅ΠΊΡ‚ΠΎΡ€Π½ΠΎΠ³ΠΎ произвСдСния.

Π§Ρ‚ΠΎΠ±Ρ‹ Π½Π°ΠΉΡ‚ΠΈ ΠΌΠΎΠ΄ΡƒΠ»ΡŒ Π²Π΅ΠΊΡ‚ΠΎΡ€Π½ΠΎΠ³ΠΎ произвСдСния Π²Π΅ΠΊΡ‚ΠΎΡ€ΠΎΠ² u ΠΈ v Π½ΡƒΠΆΠ½ΠΎ Π½Π°ΠΉΡ‚ΠΈ ΠΏΠ»ΠΎΡ‰Π°Π΄ΡŒ ΠΏΠ°Ρ€Π°Π»Π»Π΅Π»ΠΎΠ³Ρ€Π°ΠΌΠΌΠ°, ΠΊΠΎΡ‚ΠΎΡ€Ρ‹ΠΉ построСн Π½Π° Π΄Π°Π½Π½Ρ‹Ρ… Π²Π΅ΠΊΡ‚ΠΎΡ€Π°Ρ…: S = | u Γ— v | = | u | * | v | * sinΞΈ, Π³Π΄Π΅ ΞΈ β€” ΡƒΠ³ΠΎΠ» ΠΌΠ΅ΠΆΠ΄Ρƒ Π²Π΅ΠΊΡ‚ΠΎΡ€Π°ΠΌΠΈ.

Π’Π΅ΠΊΡ‚ΠΎΡ€Π½ΠΎΠ΅ ΠΏΡ€ΠΎΠΈΠ·Π²Π΅Π΄Π΅Π½ΠΈΠ΅ Π²Π΅ΠΊΡ‚ΠΎΡ€ΠΎΠ² u ΠΈ v Ρ€Π°Π²Π½ΠΎ Π½ΡƒΠ»Π΅Π²ΠΎΠΌΡƒ Π²Π΅ΠΊΡ‚ΠΎΡ€Ρƒ, Ссли u ΠΈ v ΠΏΠ°Ρ€Π°Π»Π»Π΅Π»ΡŒΠ½Ρ‹ (ΠΊΠΎΠ»Π»ΠΈΠ½Π΅Π°Ρ€Π½Ρ‹): u Γ— v = 0, Ссли u βˆ₯ v (ΞΈ = 0).

Β 

ΠŸΡ€ΠΈΠΌΠ΅Ρ€Ρ‹ Ρ€Π΅ΡˆΠ΅Π½ΠΈΡ Π·Π°Π΄Π°Ρ‡

ΠŸΡ€ΠΈΠΌΠ΅Ρ€ 1

Π°) Найти Π΄Π»ΠΈΠ½Ρƒ Π²Π΅ΠΊΡ‚ΠΎΡ€Π½ΠΎΠ³ΠΎ произвСдСния Π²Π΅ΠΊΡ‚ΠΎΡ€ΠΎΠ² β†’a ΠΈ β†’b, Ссли |β†’a| = 2, |β†’b| = 3, ∠(β†’a, β†’b) = Ο€/3.

Π±) Найти ΠΏΠ»ΠΎΡ‰Π°Π΄ΡŒ ΠΏΠ°Ρ€Π°Π»Π»Π΅Π»ΠΎΠ³Ρ€Π°ΠΌΠΌΠ°, построСнного Π½Π° Π²Π΅ΠΊΡ‚ΠΎΡ€Π°Ρ… β†’a ΠΈ β†’b, Ссли |β†’a| = 2, |β†’b| = 3, ∠(β†’a, β†’b) = Ο€/3.

Как Ρ€Π΅ΡˆΠ°Π΅ΠΌ:

Π°) По ΡƒΡΠ»ΠΎΠ²ΠΈΡŽ трСбуСтся Π½Π°ΠΉΡ‚ΠΈ Π΄Π»ΠΈΠ½Ρƒ Π²Π΅ΠΊΡ‚ΠΎΡ€Π½ΠΎΠ³ΠΎ произвСдСния. ΠŸΠΎΠ΄ΡΡ‚Π°Π²Π»ΡΠ΅ΠΌ Π΄Π°Π½Π½Ρ‹Π΅ Π² Ρ„ΠΎΡ€ΠΌΡƒΠ»Ρƒ:


ΠžΡ‚Π²Π΅Ρ‚:


Π’Π°ΠΊ ΠΊΠ°ΠΊ Π² Π·Π°Π΄Π°Ρ‡Π΅ Ρ€Π΅Ρ‡ΡŒ ΠΈΠ΄Π΅Ρ‚ ΠΎ Π΄Π»ΠΈΠ½Π΅, Ρ‚ΠΎ Π² ΠΎΡ‚Π²Π΅Ρ‚Π΅ ΡƒΠΊΠ°Π·Ρ‹Π²Π°Π΅ΠΌ Ρ€Π°Π·ΠΌΠ΅Ρ€Π½ΠΎΡΡ‚ΡŒ β€” Π΅Π΄ΠΈΠ½ΠΈΡ†Ρ‹.

Π±) По ΡƒΡΠ»ΠΎΠ²ΠΈΡŽ трСбуСтся Π½Π°ΠΉΡ‚ΠΈ ΠΏΠ»ΠΎΡ‰Π°Π΄ΡŒ ΠΏΠ°Ρ€Π°Π»Π»Π΅Π»ΠΎΠ³Ρ€Π°ΠΌΠΌΠ°, ΠΊΠΎΡ‚ΠΎΡ€Ρ‹ΠΉ построСн Π½Π° Π²Π΅ΠΊΡ‚ΠΎΡ€Π°Ρ… β†’a ΠΈ β†’b. ΠŸΠ»ΠΎΡ‰Π°Π΄ΡŒ Ρ‚Π°ΠΊΠΎΠ³ΠΎ ΠΏΠ°Ρ€Π°Π»Π»Π΅Π»ΠΎΠ³Ρ€Π°ΠΌΠΌΠ° числСнно Ρ€Π°Π²Π½Π° Π΄Π»ΠΈΠ½Π΅ Π²Π΅ΠΊΡ‚ΠΎΡ€Π½ΠΎΠ³ΠΎ произвСдСния:


ΠžΡ‚Π²Π΅Ρ‚:


ΠŸΡ€ΠΈΠΌΠ΅Ρ€ 2

Найти |[-3β†’a x 2β†’b]|, Ссли |β†’a| = 1/2, |β†’b| = 1/6, ∠(β†’a, β†’b) = Ο€/2.

Как Ρ€Π΅ΡˆΠ°Π΅ΠΌ:

По ΡƒΡΠ»ΠΎΠ²ΠΈΡŽ снова Π½ΡƒΠΆΠ½ΠΎ Π½Π°ΠΉΡ‚ΠΈ Π΄Π»ΠΈΠ½Ρƒ Π²Π΅ΠΊΡ‚ΠΎΡ€Π½ΠΎΠ³ΠΎ произвСдСния. Π˜ΡΠΏΠΎΠ»ΡŒΠ·ΡƒΠ΅ΠΌ Π½Π°ΡˆΡƒ Ρ„ΠΎΡ€ΠΌΡƒΠ»Ρƒ:


Богласно ассоциативным Π·Π°ΠΊΠΎΠ½Π°ΠΌ, выносим константы Π·Π° ΠΏΠ΅Ρ€Π΅Π΄Π΅Π»Ρ‹ Π²Π΅ΠΊΡ‚ΠΎΡ€Π½ΠΎΠ³ΠΎ произвСдСния.

Выносим константу Π·Π° ΠΏΡ€Π΅Π΄Π΅Π»Ρ‹ модуля, ΠΏΡ€ΠΈ этом ΠΌΠΎΠ΄ΡƒΠ»ΡŒ позволяСт ΡƒΠ±Ρ€Π°Ρ‚ΡŒ Π·Π½Π°ΠΊ минус. Π”Π»ΠΈΠ½Π° ΠΆΠ΅ Π½Π΅ ΠΌΠΎΠΆΠ΅Ρ‚ Π±Ρ‹Ρ‚ΡŒ ΠΎΡ‚Ρ€ΠΈΡ†Π°Ρ‚Π΅Π»ΡŒΠ½ΠΎΠΉ.

ΠžΡ‚Π²Π΅Ρ‚:


ΠŸΡ€ΠΈΠΌΠ΅Ρ€ 3

Π”Π°Π½Ρ‹ Π²Π΅Ρ€ΡˆΠΈΠ½Ρ‹ Ρ‚Ρ€Π΅ΡƒΠ³ΠΎΠ»ΡŒΠ½ΠΈΠΊΠ° A (0, 2, 0), B (-2, 5,0), C (-2, 2, 6). Найти Π΅Π³ΠΎ ΠΏΠ»ΠΎΡ‰Π°Π΄ΡŒ.

Как Ρ€Π΅ΡˆΠ°Π΅ΠΌ:

Π‘Π½Π°Ρ‡Π°Π»Π° Π½Π°ΠΉΠ΄Ρ‘ΠΌ Π²Π΅ΠΊΡ‚ΠΎΡ€Ρ‹:


Π—Π°Ρ‚Π΅ΠΌ Π²Π΅ΠΊΡ‚ΠΎΡ€Π½ΠΎΠ΅ ΠΏΡ€ΠΎΠΈΠ·Π²Π΅Π΄Π΅Π½ΠΈΠ΅:


Вычислим Π΅Π³ΠΎ Π΄Π»ΠΈΠ½Ρƒ:


ΠŸΠΎΠ΄ΡΡ‚Π°Π²ΠΈΠΌ Π΄Π°Π½Π½Ρ‹Π΅ Π² Ρ„ΠΎΡ€ΠΌΡƒΠ»Ρ‹ ΠΏΠ»ΠΎΡ‰Π°Π΄Π΅ΠΉ ΠΏΠ°Ρ€Π°Π»Π»Π΅Π»ΠΎΠ³Ρ€Π°ΠΌΠΌΠ° ΠΈ Ρ‚Ρ€Π΅ΡƒΠ³ΠΎΠ»ΡŒΠ½ΠΈΠΊΠ°:


ΠžΡ‚Π²Π΅Ρ‚:


ГСомСтричСский смысл Π²Π΅ΠΊΡ‚ΠΎΡ€Π½ΠΎΠ³ΠΎ произвСдСния

По ΠΎΠΏΡ€Π΅Π΄Π΅Π»Π΅Π½ΠΈΡŽ Π΄Π»ΠΈΠ½Π° Π²Π΅ΠΊΡ‚ΠΎΡ€Π½ΠΎΠ³ΠΎ произвСдСния Π²Π΅ΠΊΡ‚ΠΎΡ€ΠΎΠ² Ρ€Π°Π²Π½Π°


А ΠΈΠ· курса Π³Π΅ΠΎΠΌΠ΅Ρ‚Ρ€ΠΈΠΈ срСднСй ΡˆΠΊΠΎΠ»Ρ‹ ΠΌΡ‹ Π·Π½Π°Π΅ΠΌ, Ρ‡Ρ‚ΠΎ ΠΏΠ»ΠΎΡ‰Π°Π΄ΡŒ Ρ‚Ρ€Π΅ΡƒΠ³ΠΎΠ»ΡŒΠ½ΠΈΠΊΠ° Ρ€Π°Π²Π½Π° ΠΏΠΎΠ»ΠΎΠ²ΠΈΠ½Π΅ произвСдСния Π΄Π»ΠΈΠ½ Π΄Π²ΡƒΡ… сторон Ρ‚Ρ€Π΅ΡƒΠ³ΠΎΠ»ΡŒΠ½ΠΈΠΊΠ° Π½Π° синус ΡƒΠ³Π»Π° ΠΌΠ΅ΠΆΠ΄Ρƒ Π½ΠΈΠΌΠΈ.

ΠŸΠΎΡΡ‚ΠΎΠΌΡƒ Π΄Π»ΠΈΠ½Π° Π²Π΅ΠΊΡ‚ΠΎΡ€Π½ΠΎΠ³ΠΎ произвСдСния Ρ€Π°Π²Π½Π° ΡƒΠ΄Π²ΠΎΠ΅Π½Π½ΠΎΠΉ ΠΏΠ»ΠΎΡ‰Π°Π΄ΠΈ Ρ‚Ρ€Π΅ΡƒΠ³ΠΎΠ»ΡŒΠ½ΠΈΠΊΠ°, ΠΈΠΌΠ΅ΡŽΡ‰Π΅Π³ΠΎ сторонами Π²Π΅ΠΊΡ‚ΠΎΡ€Ρ‹ β†’a ΠΈ β†’b, Ссли ΠΈΡ… ΠΎΡ‚Π»ΠΎΠΆΠΈΡ‚ΡŒ ΠΎΡ‚ ΠΎΠ΄Π½ΠΎΠΉ Ρ‚ΠΎΡ‡ΠΊΠΈ. ΠŸΡ€ΠΎΡ‰Π΅ говоря, Π΄Π»ΠΈΠ½Π° Π²Π΅ΠΊΡ‚ΠΎΡ€Π½ΠΎΠ³ΠΎ произвСдСния Π²Π΅ΠΊΡ‚ΠΎΡ€ΠΎΠ² β†’a ΠΈ β†’b Ρ€Π°Π²Π½Π° ΠΏΠ»ΠΎΡ‰Π°Π΄ΠΈ ΠΏΠ°Ρ€Π°Π»Π»Π΅Π»ΠΎΠ³Ρ€Π°ΠΌΠΌΠ° со сторонами |β†’a| ΠΈ |β†’b| ΠΈ ΡƒΠ³Π»ΠΎΠΌ ΠΌΠ΅ΠΆΠ΄Ρƒ Π½ΠΈΠΌΠΈ, Ρ€Π°Π²Π½Ρ‹ΠΌ (β†’a, β†’b). Π’ этом состоит гСомСтричСский смысл Π²Π΅ΠΊΡ‚ΠΎΡ€Π½ΠΎΠ³ΠΎ произвСдСния.


ЀизичСский смысл Π²Π΅ΠΊΡ‚ΠΎΡ€Π½ΠΎΠ³ΠΎ произвСдСния

Π’ ΠΌΠ΅Ρ…Π°Π½ΠΈΠΊΠ΅ β€” ΠΎΠ΄Π½ΠΎΠΌ ΠΈΠ· Ρ€Π°Π·Π΄Π΅Π»ΠΎΠ² Ρ„ΠΈΠ·ΠΈΠΊΠΈ β€” благодаря Π²Π΅ΠΊΡ‚ΠΎΡ€Π½ΠΎΠΌΡƒ ΠΏΡ€ΠΎΠΈΠ·Π²Π΅Π΄Π΅Π½ΠΈΡŽ ΠΌΠΎΠΆΠ½ΠΎ ΠΎΠΏΡ€Π΅Π΄Π΅Π»ΠΈΡ‚ΡŒ ΠΌΠΎΠΌΠ΅Π½Ρ‚ силы ΠΎΡ‚Π½ΠΎΡΠΈΡ‚Π΅Π»ΡŒΠ½ΠΎ Ρ‚ΠΎΡ‡ΠΊΠΈ пространства. ΠŸΠΎΡΡ‚ΠΎΠΌΡƒ сформулируСм Π΅Ρ‰Π΅ ΠΎΠ΄Π½ΠΎ Π²Π°ΠΆΠ½ΠΎΠ΅ ΠΎΠΏΡ€Π΅Π΄Π΅Π»Π΅Π½ΠΈΠ΅.

Под ΠΌΠΎΠΌΠ΅Π½Ρ‚ΠΎΠΌ силы β†’F, ΠΏΡ€ΠΈΠ»ΠΎΠΆΠ΅Π½Π½ΠΎΠΉ ΠΊ Ρ‚ΠΎΡ‡ΠΊΠ΅ B, ΠΎΡ‚Π½ΠΎΡΠΈΡ‚Π΅Π»ΡŒΠ½ΠΎ Ρ‚ΠΎΡ‡ΠΊΠΈ A понимаСтся ΡΠ»Π΅Π΄ΡƒΡŽΡ‰Π΅Π΅ Π²Π΅ΠΊΡ‚ΠΎΡ€Π½ΠΎΠ΅ ΠΏΡ€ΠΎΠΈΠ·Π²Π΅Π΄Π΅Π½ΠΈΠ΅ [β†’A B Γ— β†’F].


Π’Π΅ΠΊΡ‚ΠΎΡ€ Π»ΠΈΠ½Π΅ΠΉΠ½ΠΎΠΉ скорости β†’V Ρ‚ΠΎΡ‡ΠΊΠΈ M колСса Ρ€Π°Π²Π΅Π½ Π²Π΅ΠΊΡ‚ΠΎΡ€Π½ΠΎΠΌΡƒ ΠΏΡ€ΠΎΠΈΠ·Π²Π΅Π΄Π΅Π½ΠΈΡŽ Π²Π΅ΠΊΡ‚ΠΎΡ€Π° ΡƒΠ³Π»ΠΎΠ²ΠΎΠΉ скорости β†’W ΠΈ радиус-Π²Π΅ΠΊΡ‚ΠΎΡ€Π° Ρ‚ΠΎΡ‡ΠΊΠΈ колСса, Ρ‚ΠΎ Π΅ΡΡ‚ΡŒ β†’V = β†’W`β†’rM.


ΠžΠΏΠ΅Ρ€Π°Ρ†ΠΈΠΈ с Π²Π΅ΠΊΡ‚ΠΎΡ€Π°ΠΌΠΈ, слоТСниС Π²Π΅ΠΊΡ‚ΠΎΡ€ΠΎΠ², ΡƒΠΌΠ½ΠΎΠΆΠ΅Π½ΠΈΠ΅ Π²Π΅ΠΊΡ‚ΠΎΡ€Π° Π½Π° Π΄Π΅ΠΉΡΡ‚Π²ΠΈΡ‚Π΅Π»ΡŒΠ½ΠΎΠ΅ число.

Рассмотрим Π²Π΅ΠΊΡ‚ΠΎΡ€ v с Π½Π°Ρ‡Π°Π»ΡŒΠ½ΠΎΠΉ Ρ‚ΠΎΡ‡ΠΊΠΎΠΉ Π² Π½Π°Ρ‡Π°Π»Π΅ ΠΊΠΎΠΎΡ€Π΄ΠΈΠ½Π°Ρ‚ Π² любой ΠΊΠΎΠΎΡ€Π΄ΠΈΠ½Π°Ρ‚Π½ΠΎΠΉ систСмС x-y ΠΈ с ΠΊΠΎΠ½Π΅Ρ‡Π½ΠΎΠΉ Ρ‚ΠΎΡ‡ΠΊΠΎΠΉ Π² (a,b). ΠœΡ‹ Π³ΠΎΠ²ΠΎΡ€ΠΈΠΌ, Ρ‡Ρ‚ΠΎ Π²Π΅ΠΊΡ‚ΠΎΡ€ находится Π² стандартном ΠΏΠΎΠ»ΠΎΠΆΠ΅Π½ΠΈΠΈ ΠΈ ссылаСмся Π½Π° Π½Π΅Π³ΠΎ ΠΊΠ°ΠΊ Π½Π° радиус-Π²Π΅ΠΊΡ‚ΠΎΡ€. ΠžΠ±Ρ€Π°Ρ‚ΠΈΡ‚Π΅ Π²Π½ΠΈΠΌΠ°Π½ΠΈΠ΅, Ρ‡Ρ‚ΠΎ ΠΏΠ°Ρ€Π° Ρ‚ΠΎΡ‡Π΅ΠΊ опрСдСляСт этот Π²Π΅ΠΊΡ‚ΠΎΡ€. Π’Π°ΠΊΠΈΠΌ ΠΎΠ±Ρ€Π°Π·ΠΎΠΌ, ΠΌΡ‹ ΠΌΠΎΠΆΠ΅ΠΌ ΠΈΡΠΏΠΎΠ»ΡŒΠ·ΠΎΠ²Π°Ρ‚ΡŒ это для обозначСния Π²Π΅ΠΊΡ‚ΠΎΡ€Π°. Π§Ρ‚ΠΎΠ±Ρ‹ ΠΏΠΎΠ΄Ρ‡Π΅Ρ€ΠΊΠ½ΡƒΡ‚ΡŒ, Ρ‡Ρ‚ΠΎ ΠΌΡ‹ ΠΈΠΌΠ΅Π΅ΠΌ Π² Π²ΠΈΠ΄Ρƒ Π²Π΅ΠΊΡ‚ΠΎΡ€, ΠΈ, Ρ‡Ρ‚ΠΎΠ±Ρ‹ ΠΈΠ·Π±Π΅ΠΆΠ°Ρ‚ΡŒ ΠΏΡƒΡ‚Π°Π½ΠΈΡ†Ρ‹, ΠΊΠ°ΠΊ ΠΏΡ€Π°Π²ΠΈΠ»ΠΎ, ΠΏΠΈΡˆΡƒΡ‚:
v = .


ΠšΠΎΠΎΡ€Π΄ΠΈΠ½Π°Ρ‚Π° a Π΅ΡΡ‚ΡŒ скаляром Π³ΠΎΡ€ΠΈΠ·ΠΎΠ½Ρ‚Π°Π»ΡŒΠ½ΠΎΠΉ ΠΊΠΎΠΌΠΏΠΎΠ½Π΅Π½Ρ‚Ρ‹ Π²Π΅ΠΊΡ‚ΠΎΡ€Π°, ΠΈ ΠΊΠΎΠΎΡ€Π΄ΠΈΠ½Π°Ρ‚Π° b Π΅ΡΡ‚ΡŒ скаляром Π²Π΅Ρ€Ρ‚ΠΈΠΊΠ°Π»ΡŒΠ½ΠΎΠΉ ΠΊΠΎΠΌΠΏΠΎΠ½Π΅Π½Ρ‚Ρ‹ Π²Π΅ΠΊΡ‚ΠΎΡ€Π°. Под скаляром ΠΌΡ‹ ΠΏΠΎΠ΄Ρ€Π°Π·ΡƒΠΌΠ΅Π²Π°Π΅ΠΌ числСнноС количСство, Π° Π½Π΅ Π²Π΅ΠΊΡ‚ΠΎΡ€Π½ΡƒΡŽ Π²Π΅Π»ΠΈΡ‡ΠΈΠ½Ρƒ. Π’Π°ΠΊΠΈΠΌ ΠΎΠ±Ρ€Π°Π·ΠΎΠΌ, это рассматриваСтся ΠΊΠ°ΠΊ компонСнтная Ρ„ΠΎΡ€ΠΌΠ° v. ΠžΠ±Ρ€Π°Ρ‚ΠΈΡ‚Π΅ Π²Π½ΠΈΠΌΠ°Π½ΠΈΠ΅, Ρ‡Ρ‚ΠΎ a ΠΈ b НЕ Π²Π΅ΠΊΡ‚ΠΎΡ€Π° ΠΈ ΠΈΡ… Π½Π΅ Π½Π°Π΄ΠΎ ΠΏΡƒΡ‚Π°Ρ‚ΡŒ с ΠΎΠΏΡ€Π΅Π΄Π΅Π»Π΅Π½ΠΈΠ΅ΠΌ ΠΊΠΎΠΌΠΏΠΎΠ½Π΅Π½Ρ‚Π° Π²Π΅ΠΊΡ‚ΠΎΡ€Π°.

Π’Π΅ΠΏΠ΅Ρ€ΡŒ рассмотрим с A = (x1, y1) ΠΈ C = (x2, y2). Π”Π°Π²Π°ΠΉΡ‚Π΅ рассмотрим, ΠΊΠ°ΠΊ Π½Π°ΠΉΡ‚ΠΈ радиус Π²Π΅ΠΊΡ‚ΠΎΡ€, эквивалСнтный . Как Π’Ρ‹ Π²ΠΈΠ΄ΠΈΡ‚Π΅ Π½Π° рисункС Π²Π½ΠΈΠ·Ρƒ, Π½Π°Ρ‡Π°Π»ΡŒΠ½Π°Ρ Ρ‚ΠΎΡ‡ΠΊΠ° A ΠΏΠ΅Ρ€Π΅ΠΌΠ΅Ρ‰Π΅Π½Π° Π² Π½Π°Ρ‡Π°Π»ΠΎ ΠΊΠΎΠΎΡ€Π΄ΠΈΠ½Π°Ρ‚ (0, 0). ΠšΠΎΠΎΡ€Π΄ΠΈΠ½Π°Ρ‚Ρ‹ P находятся Π²Ρ‹Ρ‡ΠΈΡ‚Π°Π½ΠΈΠ΅ΠΌ ΠΊΠΎΠΎΡ€Π΄ΠΈΠ½Π°Ρ‚ A ΠΈΠ· ΠΊΠΎΠΎΡ€Π΄ΠΈΠ½Π°Ρ‚ C. Π’Π°ΠΊΠΈΠΌ ΠΎΠ±Ρ€Π°Π·ΠΎΠΌ, P = (x2 – x1, y2 – y1) ΠΈ радиус Π²Π΅ΠΊΡ‚ΠΎΡ€ Π΅ΡΡ‚ΡŒ .

МоТно ΠΏΠΎΠΊΠ°Π·Π°Ρ‚ΡŒ, Ρ‡Ρ‚ΠΎ ΠΈ ΠΈΠΌΠ΅ΡŽΡ‚ ΠΎΠ΄Π½Ρƒ ΠΈ Ρ‚Ρƒ ΠΆΠ΅ Π²Π΅Π»ΠΈΡ‡ΠΈΠ½Ρƒ ΠΈ Π½Π°ΠΏΡ€Π°Π²Π»Π΅Π½ΠΈΠ΅, ΠΈ поэтому эквивалСнтны. Π’Π°ΠΊΠΈΠΌ ΠΎΠ±Ρ€Π°Π·ΠΎΠΌ, = = 2 – x1, y2 – y1 >.

ΠšΠΎΠΌΠΏΠΎΠ½Π΅Π½Ρ‚Π½Π°Ρ Ρ„ΠΎΡ€ΠΌΠ° с A = (x1, y1) ΠΈ C = (x2, y2) Π΅ΡΡ‚ΡŒ
= 2 – x1, y2 – y1 >.

ΠŸΡ€ΠΈΠΌΠ΅Ρ€ 1 НайдитС ΠΊΠΎΠΌΠΏΠΎΠ½Π΅Π½Ρ‚Π½ΡƒΡŽ Ρ„ΠΎΡ€ΠΌΡƒ Ссли C = (- 4, – 3) ΠΈ F = (1, 5).

РСшСниС ΠœΡ‹ ΠΈΠΌΠ΅Π΅ΠΌ
= = .

ΠžΠ±Ρ€Π°Ρ‚ΠΈΡ‚Π΅ Π²Π½ΠΈΠΌΠ°Π½ΠΈΠ΅, Ρ‡Ρ‚ΠΎ Π²Π΅ΠΊΡ‚ΠΎΡ€ Π΅ΡΡ‚ΡŒ Ρ€Π°Π²Π½Ρ‹ΠΌ радиус-Π²Π΅ΠΊΡ‚ΠΎΡ€Ρƒ , ΠΊΠ°ΠΊ ΠΏΠΎΠΊΠ°Π·Π°Π½ΠΎ Π½Π° рисункС Π²Π²Π΅Ρ€Ρ…Ρƒ.

Π’Π΅ΠΏΠ΅Ρ€ΡŒ, ΠΊΠΎΠ³Π΄Π° ΠΌΡ‹ Π·Π½Π°Π΅ΠΌ, ΠΊΠ°ΠΊ Π·Π°ΠΏΠΈΡΠ°Ρ‚ΡŒ Π²Π΅ΠΊΡ‚ΠΎΡ€ Π² ΠΊΠΎΠΌΠΏΠΎΠ½Π΅Π½Ρ‚Π½ΠΎΠΉ Ρ„ΠΎΡ€ΠΌΠ΅, Π΄Π°Π²Π°ΠΉΡ‚Π΅ ΠΈΠ·Π»ΠΎΠΆΠΈΠΌ Π½Π΅ΠΊΠΎΡ‚ΠΎΡ€Ρ‹Π΅ опрСдСлСния.
Π”Π»ΠΈΠ½Ρƒ Π²Π΅ΠΊΡ‚ΠΎΡ€Π° v Π»Π΅Π³ΠΊΠΎ ΠΎΠΏΡ€Π΅Π΄Π΅Π»ΠΈΡ‚ΡŒ, ΠΊΠΎΠ³Π΄Π° извСстны ΠΊΠΎΠΌΠΏΠΎΠ½Π΅Π½Ρ‚Ρ‹ Π²Π΅ΠΊΡ‚ΠΎΡ€Π°. Для v = 1, v2 >, ΠΌΡ‹ ΠΈΠΌΠ΅Π΅ΠΌ
|v|2 = v21 + v22 Β Β Β Β Β Β Β Β  Π˜ΡΠΏΠΎΠ»ΡŒΠ·ΡƒΡ Ρ‚Π΅ΠΎΡ€Π΅ΠΌΡƒ ΠŸΠΈΡ„Π°Π³ΠΎΡ€Π°
|v| = √v21 + v22.

Π”Π»ΠΈΠ½Π°, ΠΈΠ»ΠΈ Π²Π΅Π»ΠΈΡ‡ΠΈΠ½Π° Π²Π΅Ρ‚ΠΊΡ‚ΠΎΡ€Π° v = 1, v2 > находится ΠΊΠ°ΠΊ |v| = √v21 + v22.

Π”Π²Π° Π²Π΅ΠΊΡ‚ΠΎΡ€Π° Ρ€Π°Π²Π½Ρ‹ ΠΈΠ»ΠΈ эквивалСнтны, Ссли ΠΎΠ½ΠΈ ΠΈΠΌΠ΅ΡŽΡ‚ ΠΎΠ΄Π½Ρƒ ΠΈ Ρ‚Ρƒ ΠΆΠ΅ Π²Π΅Π»ΠΈΡ‡ΠΈΠ½Ρƒ ΠΈ ΠΎΠ΄Π½ΠΎ ΠΈ Ρ‚ΠΎ ΠΆΠ΅ Π½Π°ΠΏΡ€Π°Π²Π»Π΅Π½ΠΈΠ΅.

ΠŸΡƒΡΡ‚ΡŒ u = 1, u2 > ΠΈ v = 1, v2 >. TΠΎΠ³Π΄Π°
1, u2 > = 1, v2 > Β Β Β Β Β Β Β Β  Ρ‚ΠΎΠ»ΡŒΠΊΠΎ Ссли u1 = v1 and u2 = v2.

ΠžΠΏΠ΅Ρ€Π°Ρ†ΠΈΠΈ с Π²Π΅ΠΊΡ‚ΠΎΡ€Π°ΠΌΠΈ

Π§Ρ‚ΠΎΠ±Ρ‹ ΡƒΠΌΠ½ΠΎΠΆΠΈΡ‚ΡŒ Π²Π΅ΠΊΡ‚ΠΎΡ€ V Π½Π° ΠΏΠΎΠ»ΠΎΠΆΠΈΡ‚Π΅Π»ΡŒΠ½ΠΎΠ΅ число, ΠΌΡ‹ ΡƒΠΌΠ½ΠΎΠΆΠ°Π΅ΠΌ Π΅Π³ΠΎ Π΄Π»ΠΈΠ½Ρƒ Π½Π° это число. Π•Π³ΠΎ Π½Π°ΠΏΡ€Π°Π²Π»Π΅Π½ΠΈΠ΅ остаСтся ΠΏΡ€Π΅ΠΆΠ½ΠΈΠΌ. Когда Π²Π΅ΠΊΡ‚ΠΎΡ€ V умноТаСтся Π½Π° 2, Π½Π°ΠΏΡ€ΠΈΠΌΠ΅Ρ€, Π΅Π³ΠΎ Π΄Π»ΠΈΠ½Π° увСличиваСтся Π² Π΄Π²Π° Ρ€Π°Π·Π°, Π½ΠΎ Π΅Π³ΠΎ Π½Π°ΠΏΡ€Π°Π²Π»Π΅Π½ΠΈΠ΅ Π½Π΅ измСняСтся. Когда Π²Π΅ΠΊΡ‚ΠΎΡ€ умноТаСтся Π½Π° 1,6, Π΅Π³ΠΎ Π΄Π»ΠΈΠ½Π° увСличиваСтся Π½Π° 60%, Π° Π½Π°ΠΏΡ€Π°Π²Π»Π΅Π½ΠΈΠ΅ остаСтся ΠΏΡ€Π΅ΠΆΠ½ΠΈΠΌ. Π§Ρ‚ΠΎΠ±Ρ‹ ΡƒΠΌΠ½ΠΎΠΆΠΈΡ‚ΡŒ Π²Π΅ΠΊΡ‚ΠΎΡ€ V Π½Π° ΠΎΡ‚Ρ€ΠΈΡ†Π°Ρ‚Π΅Π»ΡŒΠ½ΠΎΠ΅ Π΄Π΅ΠΉΡΡ‚Π²ΠΈΡ‚Π΅Π»ΡŒΠ½ΠΎΠ΅ число, ΡƒΠΌΠ½ΠΎΠΆΠ°Π΅ΠΌ Π΅Π³ΠΎ Π΄Π»ΠΈΠ½Ρƒ Π½Π° это число ΠΈ измСняСм Π½Π°ΠΏΡ€Π°Π²Π»Π΅Π½ΠΈΠ΅ Π½Π° ΠΏΡ€ΠΎΡ‚ΠΈΠ²ΠΎΠΏΠΎΠ»ΠΎΠΆΠ½ΠΎΠ΅. НапримСр, Когда Π²Π΅ΠΊΡ‚ΠΎΡ€ умноТаСтся Π½Π° (-2), Π΅Π³ΠΎ Π΄Π»ΠΈΠ½Π° увСличиваСтся Π² Π΄Π²Π° Ρ€Π°Π·Π° ΠΈ Π΅Π³ΠΎ Π½Π°ΠΏΡ€Π°Π²Π»Π΅Π½ΠΈΠ΅ измСняСтся Π½Π° ΠΏΡ€ΠΎΡ‚ΠΈΠ²ΠΎΠΏΠΎΠ»ΠΎΠΆΠ½ΠΎΠ΅. Π’Π°ΠΊ ΠΊΠ°ΠΊ Π΄Π΅ΠΉΡΡ‚Π²ΠΈΡ‚Π΅Π»ΡŒΠ½Ρ‹Π΅ числа Ρ€Π°Π±ΠΎΡ‚Π°ΡŽΡ‚ ΠΊΠ°ΠΊ скалярныС ΠΌΠ½ΠΎΠΆΠΈΡ‚Π΅Π»ΠΈ Π² ΡƒΠΌΠ½ΠΎΠΆΠ΅Π½ΠΈΠΈ Π²Π΅ΠΊΡ‚ΠΎΡ€ΠΎΠ², ΠΌΡ‹ Π½Π°Π·Ρ‹Π²Π°Π΅ΠΌ ΠΈΡ… скаляры ΠΈ ΠΏΡ€ΠΎΠΈΠ·Π²Π΅Π΄Π΅Π½ΠΈΠ΅ kv называСтся скалярныС ΠΊΡ€Π°Ρ‚Π½Ρ‹Π΅ v.

Для Π΄Π΅ΠΉΡΡ‚Π²ΠΈΡ‚Π΅Π»ΡŒΠ½ΠΎΠ³ΠΎ числа k ΠΈ Π²Π΅ΠΊΡ‚ΠΎΡ€Π° v = 1, v2 >, скалярноС ΠΏΡ€ΠΎΠΈΠ·Π²Π΅Π΄Π΅Π½ΠΈΠ΅ k ΠΈ v Π΅ΡΡ‚ΡŒ
kv = k.1, v2 > = 1, kv2 >.
Π’Π΅ΠΊΡ‚ΠΎΡ€ kv Π΅ΡΡ‚ΡŒ скалярным ΠΊΡ€Π°Ρ‚Π½Ρ‹ΠΌ Π²Π΅ΠΊΡ‚ΠΎΡ€Π° v.

ΠŸΡ€ΠΈΠΌΠ΅Ρ€ 2 ΠŸΡƒΡΡ‚ΡŒ u = ΠΈ w = . НайдитС – 7w, 3u ΠΈ – 1w.

РСшСниС
– 7w = – 7. = ,
3u = 3. = ,
– 1w = – 1. = .

Π’Π΅ΠΏΠ΅Ρ€ΡŒ ΠΌΡ‹ ΠΌΠΎΠΆΠ΅ΠΌ ΡΠ»ΠΎΠΆΠΈΡ‚ΡŒ Π΄Π²Π° Π²Π΅ΠΊΡ‚ΠΎΡ€Π°, ΠΈΡΠΏΠΎΠ»ΡŒΠ·ΡƒΡ ΠΊΠΎΠΌΠΏΠΎΠ½Π΅Π½Ρ‚Ρ‹. Π§Ρ‚ΠΎΠ±Ρ‹ ΡΠ»ΠΎΠΆΠΈΡ‚ΡŒ Π΄Π²Π° Π²Π΅ΠΊΡ‚ΠΎΡ€Π° Π² ΠΊΠΎΠΌΠΏΠΎΠ½Π΅Π½Ρ‚Π½ΠΎΠΉ Ρ„ΠΎΡ€ΠΌΠ΅, ΠΌΡ‹ складываСм ΡΠΎΠΎΡ‚Π²Π΅Ρ‚ΡΡ‚Π²ΡƒΡŽΡ‰ΠΈΠ΅ ΠΊΠΎΠΌΠΏΠΎΠ½Π΅Π½Ρ‚Ρ‹. ΠŸΡƒΡΡ‚ΡŒ u = 1, u2 > ΠΈ v = 1, v2 >. Π’ΠΎΠ³Π΄Π°
u + v = 1 + v1, u2 + v2 >

НапримСр, Ссли v = ΠΈ w = , Ρ‚ΠΎΠ³Π΄Π°
v + w = =

Если u = 1, u2 > ΠΈ v = 1, v2 >, Ρ‚ΠΎΠ³Π΄Π°
u + v = 1 + v1, u2 + v2 >.

ΠŸΠ΅Ρ€Π΅Π΄ Ρ‚Π΅ΠΌ, ΠΊΠ°ΠΊ ΠΌΡ‹ ΠΎΠΏΡ€Π΅Π΄Π΅Π»ΠΈΠΌ Π²Ρ‹Ρ‡ΠΈΡ‚Π°Π½ΠΈΠ΅ Π²Π΅ΠΊΡ‚ΠΎΡ€ΠΎΠ² Π½Π°ΠΌ Π½ΡƒΠΆΠ½ΠΎ Π΄Π°Ρ‚ΡŒ ΠΎΠΏΡ€Π΅Π΄Π΅Π»Π΅Π½ΠΈΠ΅ – v. ΠŸΡ€ΠΎΡ‚ΠΈΠ²ΠΎΠΏΠΎΠ»ΠΎΠΆΠ½Ρ‹ΠΉ Π²Π΅ΠΊΡ‚ΠΎΡ€Ρƒ v = 1, v2 >, ΠΈΠ·ΠΎΠ±Ρ€Π°ΠΆΠ΅Π½Π½ΠΎΠΌΡƒ Π²Π½ΠΈΠ·Ρƒ, Π΅ΡΡ‚ΡŒ
– v = (- 1).v = (- 1)1, v2 > = 1, – v2 >

Π’Ρ‹Ρ‡ΠΈΡ‚Π°Π½ΠΈΠ΅ Π²Π΅ΠΊΡ‚ΠΎΡ€ΠΎΠ², Ρ‚Π°ΠΊΠΎΠ΅ ΠΊΠ°ΠΊ u – v Π²ΠΎΠ²Π»Π΅ΠΊΠ°Π΅Ρ‚ Π²Ρ‹Ρ‡ΠΈΡ‚Π°Π½ΠΈΠ΅ ΡΠΎΠΎΡ‚Π²Π΅Ρ‚ΡΡ‚Π²ΡƒΡŽΡ‰ΠΈΡ… ΠΊΠΎΠΌΠΏΠΎΠ½Π΅Π½Ρ‚. ΠœΡ‹ ΠΏΠΎΠΊΠ°ΠΆΠ΅ΠΌ это прСдставлСниСм u – v ΠΊΠ°ΠΊ u + (- v). Если u = 1, u2 > ΠΈ v = 1, v2 >, Ρ‚ΠΎΠ³Π΄Π°
u – v = u + (- v) = 1, u2 > + 1, – v2 > = 1 + (- v1), u2 + (- v2) > = 1 – v1, u2 – v2 >

ΠœΡ‹ ΠΌΠΎΠΆΠ΅ΠΌ ΠΏΡ€ΠΎΠΈΠ»Π»ΡŽΡΡ‚Ρ€ΠΈΡ€ΠΎΠ²Π°Ρ‚ΡŒ Π²Ρ‹Ρ‡ΠΈΡ‚Π°Π½ΠΈΠ΅ Π²Π΅ΠΊΡ‚ΠΎΡ€ΠΎΠ² с ΠΏΠΎΠΌΠΎΡ‰ΡŒΡŽ ΠΏΠ°Ρ€Π°Π»Π»Π΅Π»ΠΎΠ³Ρ€Π°ΠΌΠΌΠ° , ΠΊΠ°ΠΊ ΠΌΡ‹ это Π΄Π΅Π»Π°Π»ΠΈ для слоТСния Π²Π΅ΠΊΡ‚ΠΎΡ€ΠΎΠ².

Π’Ρ‹Ρ‡ΠΈΡ‚Π°Π½ΠΈΠ΅ Π²Π΅ΠΊΡ‚ΠΎΡ€ΠΎΠ²

Если u = 1, u2 > ΠΈ v = 1, v2 >, Ρ‚ΠΎΠ³Π΄Π°
u – v = 1 – v1, u2 – v2 >.

Π˜Π½Ρ‚Π΅Ρ€Π΅ΡΠ½ΠΎ ΡΡ€Π°Π²Π½ΠΈΡ‚ΡŒ суммы Π΄Π²ΡƒΡ… Π²Π΅ΠΊΡ‚ΠΎΡ€ΠΎΠ² с Ρ€Π°Π·Π½ΠΈΡ†Π΅ΠΉ Ρ‚Π΅Ρ… ΠΆΠ΅ Π΄Π²ΡƒΡ… Π²Π΅ΠΊΡ‚ΠΎΡ€ΠΎΠ² Π² ΠΎΠ΄Π½ΠΎΠΌ ΠΏΠ°Ρ€Π°Π»Π»Π΅Π»ΠΎΠ³Ρ€Π°ΠΌΠΌΠ΅. Π’Π΅ΠΊΡ‚ΠΎΡ€Ρ‹ u + v ΠΈ u – v Π΅ΡΡ‚ΡŒ диагоналями ΠΏΠ°Ρ€Π°Π»Π»Π΅Π»ΠΎΠ³Ρ€Π°ΠΌΠΌΠ°.

ΠŸΡ€ΠΈΠΌΠ΅Ρ€ 3 Π‘Π΄Π΅Π»Π°ΠΉΡ‚Π΅ ΡΠ»Π΅Π΄ΡƒΡŽΡ‰ΠΈΠ΅ вычислСния, Π³Π΄Π΅ u = ΠΈ v = .
a) u + v
b) u – 6v
c)3u + 4v
d)|5v – 2u|

РСшСниС
a) u + v = + = = ;
b)u – 6v = – 6. = – = ;
c) 3u + 4v = 3. + 4. = + = ;
d) |5v – 2u| = |5. – 2.| = | – | = || = √(- 29)2 + 212 = √1282 β‰ˆ 35,8

ΠŸΡ€Π΅ΠΆΠ΄Π΅ Ρ‡Π΅ΠΌ ΡΡ„ΠΎΡ€ΠΌΡƒΠ»ΠΈΡ€ΠΎΠ²Π°Ρ‚ΡŒ свойства Π²Π΅ΠΊΡ‚ΠΎΡ€Π½ΠΎΠ³ΠΎ слоТСния ΠΈ умноТСния, ΠΌΡ‹ Π΄ΠΎΠ»ΠΆΠ½Ρ‹ Π΄Π°Ρ‚ΡŒ ΠΎΠΏΡ€Π΅Π΄Π΅Π»Π΅Π½ΠΈΠ΅ Π΅Ρ‰Π΅ ΠΎΠ΄Π½ΠΎΠΌΡƒ ΡΠΏΠ΅Ρ†ΠΈΠ°Π»ΡŒΠ½ΠΎΠΌΡƒ Π²Π΅ΠΊΡ‚ΠΎΡ€Ρƒ – Π½ΡƒΠ»Π΅Π²ΠΎΠΌΡƒ Π²Π΅ΠΊΡ‚ΠΎΡ€Ρƒ. Π’Π΅ΠΊΡ‚ΠΎΡ€, Ρ‡ΡŒΡ Π½Π°Ρ‡Π°Π»ΡŒΠ½Π°Ρ Ρ‚ΠΎΡ‡ΠΊΠ° совпадаСт с ΠΊΠΎΠ½Π΅Ρ‡Π½ΠΎΠΉ Ρ‚ΠΎΡ‡ΠΊΠΎΠΉ, называСтся Π½ΡƒΠ»Π΅Π²Ρ‹ΠΌ Π²Π΅ΠΊΡ‚ΠΎΡ€ΠΎΠΌ, обозначаСтся O, ΠΈΠ»ΠΈ . Π•Π³ΠΎ Π²Π΅Π»ΠΈΡ‡ΠΈΠ½Π° Ρ€Π°Π²Π½Π° 0. Π’ слоТСнии Π²Π΅ΠΊΡ‚ΠΎΡ€ΠΎΠ²:
v + O = v. Β Β Β Β Β Β Β Β  1, v2 > + = 1, v2 >
ΠžΠΏΠ΅Ρ€Π°Ρ†ΠΈΠΈ Π½Π°Π΄ Π²Π΅ΠΊΡ‚ΠΎΡ€Π°ΠΌΠΈ ΠΎΠ±Π»Π°Π΄Π°ΡŽΡ‚ Ρ‚Π΅ ΠΆΠ΅ самыми свойствами, Ρ‡Ρ‚ΠΎ ΠΈ ΠΎΠΏΠ΅Ρ€Π°Ρ†ΠΈΠΈ Π½Π°Π΄ вСщСствСнными числами.

Бвойства Π²Π΅ΠΊΡ‚ΠΎΡ€Π½ΠΎΠ³ΠΎ слоТСния ΠΈ умноТСния

Для всСх Π²Π΅ΠΊΡ‚ΠΎΡ€ΠΎΠ² u, v, ΠΈ w, ΠΈ для всСх скаляров b ΠΈ c:
1. u + v = v + u.
2. u + (v + w) = (u + v) + w.
3. v + O = v.
4 1.v = v; Β Β Β Β Β Β Β Β  0.v = O.
5. v + (- v) = O.
6. b(cv) = (bc)v.
7. (b + c)v = bv + cv.
8. b(u + v) = bu + bv.

ΠžΡ€Ρ‚Ρ‹

Π’Π΅ΠΊΡ‚ΠΎΡ€ Π²Π΅Π»ΠΈΡ‡ΠΈΠ½ΠΎΠΉ, ΠΈΠ»ΠΈ Π΄Π»ΠΈΠ½ΠΎΠΉ 1 называСтся ΠΎΡ€Ρ‚. Π’Π΅ΠΊΡ‚ΠΎΡ€ v = Π΅ΡΡ‚ΡŒ ΠΎΡ€Ρ‚, ΠΏΠΎΡ‚ΠΎΠΌΡƒ Ρ‡Ρ‚ΠΎ
|v| = || = √(- 3/5)2 + (4/5)2 = √9/25 + 16/25 = √25/25 = √1 = 1.

ΠŸΡ€ΠΈΠΌΠ΅Ρ€ 4 НайдитС ΠΎΡ€Ρ‚, ΠΊΠΎΡ‚ΠΎΡ€Ρ‹ΠΉ ΠΈΠΌΠ΅Π΅Ρ‚ Ρ‚ΠΎ ΠΆΠ΅ самоС Π½Π°ΠΏΡ€Π°Π²Π»Π΅Π½ΠΈΠ΅, Ρ‡Ρ‚ΠΎ ΠΈ Π²Π΅ΠΊΡ‚ΠΎΡ€ w = .

РСшСниС НайдСм сначала Π΄Π»ΠΈΠ½Ρƒ w:
|w| = √(- 3)2 + 52 = √34. Π’Π°ΠΊΠΈΠΌ ΠΎΠ±Ρ€Π°Π·ΠΎΠΌ, ΠΌΡ‹ ΠΈΡ‰Π΅ΠΌ Π²Π΅ΠΊΡ‚ΠΎΡ€, с Π΄Π»ΠΈΠ½ΠΎΠΉ 1/√34 ΠΎΡ‚ w ΠΈ с Ρ‚Π°ΠΊΠΈΠΌ ΠΆΠ΅ самым Π½Π°ΠΏΡ€Π°Π²Π»Π΅Π½ΠΈΠ΅ΠΌ, Ρ‡Ρ‚ΠΎ ΠΈ Π²Π΅ΠΊΡ‚ΠΎΡ€ w. Π­Ρ‚ΠΎΡ‚ Π²Π΅ΠΊΡ‚ΠΎΡ€ Π΅ΡΡ‚ΡŒ
u = w/√34 = /√34 = 34, 5/√34 >.
Π’Π΅ΠΊΡ‚ΠΎΡ€ u Π΅ΡΡ‚ΡŒ ΠΎΡ€Ρ‚, ΠΏΠΎΡ‚ΠΎΠΌΡƒ Ρ‡Ρ‚ΠΎ
|u| = |w/√34| = = √34/34 = √1 = 1.

Если v Π΅ΡΡ‚ΡŒ Π²Π΅ΠΊΡ‚ΠΎΡ€ ΠΈ v β‰  O, Ρ‚ΠΎΠ³Π΄Π°
(1/|v|)β€’ v, Β Β Β Β Β Β Β Β  or Β Β Β Β Β Β Β Β  v/|v|,
Π΅ΡΡ‚ΡŒ ΠΎΡ€Ρ‚ Π² Π½Π°ΠΏΡ€Π°Π²Π»Π΅Π½ΠΈΠΈ v.

Π₯отя ΠΎΡ€Ρ‚Ρ‹ ΠΌΠΎΠ³ΡƒΡ‚ ΠΈΠΌΠ΅Ρ‚ΡŒ любоС Π½Π°ΠΏΡ€Π°Π²Π»Π΅Π½ΠΈΠ΅, ΠΎΡ€Ρ‚Ρ‹, ΠΏΠ°Ρ€Π°Π»Π»Π΅Π»ΡŒΠ½Ρ‹Π΅ осям x ΠΈ y особСнно ΠΏΠΎΠ»Π΅Π·Π½Ρ‹. Они ΠΎΠΏΡ€Π΅Π΄Π΅Π»ΡΡŽΡ‚ΡΡ ΠΊΠ°ΠΊ
i = Β Β Β Β Β Β Β Β  and Β Β Β Β Β Β Β Β  j = .

Π›ΡŽΠ±ΠΎΠΉ Π²Π΅ΠΊΡ‚ΠΎΡ€ ΠΌΠΎΠΆΠ΅Ρ‚ Π±Ρ‹Ρ‚ΡŒ Π²Ρ‹Ρ€Π°ΠΆΠ΅Π½ ΠΊΠ°ΠΊ линСйная комбинация ΠΎΡ€Ρ‚Π° i ΠΈ j. НапримСр, ΠΏΡƒΡΡ‚ΡŒ v = 1, v2 >. TΠΎΠ³Π΄Π°
v = 1, v2 > = 1, 0 > + 2 > = v1 + v2 = v1i + v2j.

ΠŸΡ€ΠΈΠΌΠ΅Ρ€ 5 Π’Ρ‹Ρ€Π°Π·ΠΈΡ‚Π΅ Π²Π΅ΠΊΡ‚ΠΎΡ€ r = ΠΊΠ°ΠΊ Π»ΠΈΠ½Π΅ΠΉΠ½ΡƒΡŽ ΠΊΠΎΠΌΠ±ΠΈΠ½Π°Ρ†ΠΈΡŽ i ΠΈ j.

РСшСниС
r = = 2i + (- 6)j = 2i – 6j.

ΠŸΡ€ΠΈΠΌΠ΅Ρ€ 6 Π—Π°ΠΏΠΈΡˆΠΈΡ‚Π΅ Π²Π΅ΠΊΡ‚ΠΎΡ€ q = – i + 7j Π² ΠΊΠΎΠΌΠΏΠΎΠ½Π΅Π½Ρ‚Π½ΠΎΠΉ Ρ„ΠΎΡ€ΠΌΠ΅.

РСшСниСq = – i + 7j = -1i + 7j =

Π’Π΅ΠΊΡ‚ΠΎΡ€Π½Ρ‹Π΅ ΠΎΠΏΠ΅Ρ€Π°Ρ†ΠΈΠΈ ΠΌΠΎΠ³ΡƒΡ‚ Π±Ρ‹Ρ‚ΡŒ Ρ‚Π°ΠΊΠΆΠ΅ Π²Ρ‹ΠΏΠΎΠ»Π½Π΅Π½Ρ‹, ΠΊΠΎΠ³Π΄Π° Π²Π΅ΠΊΡ‚ΠΎΡ€Ρ‹ записаны ΠΊΠ°ΠΊ Π»ΠΈΠ½Π΅ΠΉΠ½Ρ‹Π΅ i ΠΈ j.

ΠŸΡ€ΠΈΠΌΠ΅Ρ€ 7 Если a = 5i – 2j ΠΈ b = -i + 8j, Π½Π°ΠΉΠ΄ΠΈΡ‚Π΅ 3a – b.

РСшСниС
3a – b = 3(5i – 2j) – (- i + 8j) = 15i – 6j + i – 8j = 16i – 14j.

Π£Π³Π»Ρ‹ ΠΎΠ±Π·ΠΎΡ€Π°

ΠšΠΎΠ½Π΅Ρ‡Π½Π°Ρ Ρ‚ΠΎΡ‡ΠΊΠ° P ΠΎΡ€Ρ‚Ρ‹ Π² стандартной ΠΏΠΎΠ·ΠΈΡ†ΠΈΠΈ Π΅ΡΡ‚ΡŒ Ρ‚ΠΎΡ‡ΠΊΠΎΠΉ Π½Π° Π΅Π΄ΠΈΠ½ΠΈΡ‡Π½ΠΎΠΉ окруТности, ΠΎΠΏΡ€Π΅Π΄Π΅Π»Π΅Π½Π½ΠΎΠΉ (cosΞΈ, sinΞΈ). Π’Π°ΠΊΠΈΠΌ ΠΎΠ±Ρ€Π°Π·ΠΎΠΌ, ΠΎΡ€Ρ‚ ΠΌΠΎΠΆΠ΅Ρ‚ Π±Ρ‹Ρ‚ΡŒ Π²Ρ‹Ρ€Π°ΠΆΠ΅Π½ Π² ΠΊΠΎΠΌΠΏΠΎΠ½Π΅Π½Ρ‚Π½ΠΎΠΉ Ρ„ΠΎΡ€ΠΌΠ΅,
u = ,
ΠΈΠ»ΠΈ ΠΊΠ°ΠΊ линСйная комбинация ΠΎΡ€Ρ‚ i ΠΈ j,
u = (cosΞΈ)i + (sinΞΈ)j,
Π³Π΄Π΅ ΠΊΠΎΠΌΠΏΠΎΠ½Π΅Π½Ρ‚Ρ‹ u Π΅ΡΡ‚ΡŒ функциями ΡƒΠ³Π»Π° ΠΎΠ±Π·ΠΎΡ€Π° ΞΈ измСряСмого ΠΏΡ€ΠΎΡ‚ΠΈΠ² часовой стрСлки ΠΎΡ‚ оси x ΠΊ этому Π²Π΅ΠΊΡ‚ΠΎΡ€Ρƒ. Π’Π°ΠΊ ΠΊΠ°ΠΊ ΞΈ измСняСтся ΠΎΡ‚ 0 Π΄ΠΎ 2Ο€, Ρ‚ΠΎΡ‡ΠΊΠ° P отслСТиваСт ΠΊΡ€ΡƒΠ³ x2 + y2 = 1. Π­Ρ‚ΠΎ ΠΎΡ…Π²Π°Ρ‚Ρ‹Π²Π°Π΅Ρ‚ всС Π²ΠΎΠ·ΠΌΠΎΠΆΠ½Ρ‹Π΅ направлСния ΠΎΡ€Ρ‚ΠΎΠ² ΠΈ Ρ‚ΠΎΠ³Π΄Π° ΡƒΡ€Π°Π²Π½Π΅Π½ΠΈΠ΅ u = (cosΞΈ)i + (sinΞΈ)j описываСт ΠΊΠ°ΠΆΠ΄Ρ‹ΠΉ Π²ΠΎΠ·ΠΌΠΎΠΆΠ½Ρ‹ΠΉ ΠΎΡ€Ρ‚ Π½Π° плоскости.

ΠŸΡ€ΠΈΠΌΠ΅Ρ€ 8 ВычислитС ΠΈ сдСлайтС эскиз ΠΎΡ€Ρ‚Π° u = (cosΞΈ)i + (sinΞΈ)j для ΞΈ = 2Ο€/3. Π˜Π·ΠΎΠ±Ρ€Π°Π·ΠΈΡ‚Π΅ Π΅Π΄ΠΈΠ½ΠΈΡ‡Π½ΡƒΡŽ ΠΎΠΊΡ€ΡƒΠΆΠ½ΠΎΡΡ‚ΡŒ Π½Π° эскизС.

РСшСниС
u = (cos(2Ο€/3))i + (sin(2Ο€/3))j = (- 1/2)i + (√3/2)j

ΠŸΡƒΡΡ‚ΡŒ v = 1, v2 > с ΡƒΠ³Π»ΠΎΠΌ ΠΎΠ±Π·ΠΎΡ€Π° ΞΈ. Π˜ΡΠΏΠΎΠ»ΡŒΠ·ΡƒΡ ΠΎΠΏΡ€Π΅Π΄Π΅Π»Π΅Π½ΠΈΠ΅ Ρ„ΡƒΠ½ΠΊΡ†ΠΈΠΈ тангСнса, ΠΌΡ‹ ΠΌΠΎΠΆΠ΅ΠΌ ΠΎΠΏΡ€Π΅Π΄Π΅Π»ΠΈΡ‚ΡŒ ΡƒΠ³ΠΎΠ» ΠΎΠ±Π·ΠΎΡ€Π° ΠΈΡ… ΠΊΠΎΠΌΠΏΠΎΠ½Π΅Π½Ρ‚ v:

ΠŸΡ€ΠΈΠΌΠ΅Ρ€ 9 ΠžΠΏΡ€Π΅Π΄Π΅Π»ΠΈΡ‚Π΅ ΡƒΠ³ΠΎΠ» ΠΎΠ±Π·ΠΎΡ€Π° ΞΈ Π²Π΅ΠΊΡ‚ΠΎΡ€Π° w = – 4i – 3j.

РСшСниС ΠœΡ‹ Π·Π½Π°Π΅ΠΌ, Ρ‡Ρ‚ΠΎ
w = – 4i – 3j = .
Π’Π°ΠΊΠΈΠΌ ΠΎΠ±Ρ€Π°Π·ΠΎΠΌ, ΠΈΠΌΠ΅Π΅ΠΌ
tanΞΈ = (- 3)/(- 4) = 3/4 Β Β Β Β Β Β Β Β  ΠΈ ΞΈ = tan– 1(3/4).
Π’Π°ΠΊ ΠΊΠ°ΠΊ w находится Π² Ρ‚Ρ€Π΅Ρ‚ΡŒΠ΅ΠΌ ΠΊΠ²Π°Π΄Ρ€Π°Π½Ρ‚Π΅, ΠΌΡ‹ Π·Π½Π°Π΅ΠΌ, Ρ‡Ρ‚ΠΎ ΞΈ Π΅ΡΡ‚ΡŒ ΡƒΠ³Π»ΠΎΠΌ Ρ‚Ρ€Π΅Ρ‚ΡŒΠ΅Π³ΠΎ ΠΊΠ²Π°Π΄Ρ€Π°Π½Ρ‚Π°. Π‘ΠΎΠΎΡ‚Π²Π΅Ρ‚ΡΡ‚Π²ΡƒΡŽΡ‰ΠΈΠΉ ΡƒΠ³ΠΎΠ» Π΅ΡΡ‚ΡŒ
tan– 1(3/4) β‰ˆ 37Β°, Β Β Β Β Β Β Β Β  ΠΈ Β Β Β Β Β Β Β Β  ΞΈ β‰ˆ 180Β° + 37Β°, ΠΈΠ»ΠΈ 217Β°.

Π­Ρ‚ΠΎ ΡƒΠ΄ΠΎΠ±Π½ΠΎ для Ρ€Π°Π±ΠΎΡ‚Ρ‹ с ΠΏΡ€ΠΈΠΊΠ»Π°Π΄Π½Ρ‹ΠΌΠΈ Π·Π°Π΄Π°Ρ‡Π°ΠΌΠΈ, Π° Π² ΠΏΠΎΡΠ»Π΅Π΄ΡƒΡŽΡ‰ΠΈΡ… курсах, Ρ‡Ρ‚ΠΎΠ±Ρ‹ ΠΈΠΌΠ΅Ρ‚ΡŒ способ Π²Ρ‹Ρ€Π°Π·ΠΈΡ‚ΡŒ Π²Π΅ΠΊΡ‚ΠΎΡ€ Ρ‚Π°ΠΊ, Ρ‡Ρ‚ΠΎΠ±Ρ‹ Π΅Π³ΠΎ Π²Π΅Π»ΠΈΡ‡ΠΈΠ½Π° ΠΈ Π½Π°ΠΏΡ€Π°Π²Π»Π΅Π½ΠΈΠ΅ ΠΌΠΎΠ³Π»ΠΈ Π±Ρ‹Ρ‚ΡŒ Π»Π΅Π³ΠΊΠΎ ΠΎΠΏΡ€Π΅Π΄Π΅Π»Π΅Π½Ρ‹ ΠΈΠ»ΠΈ ΠΏΡ€ΠΎΡ‡ΠΈΡ‚Π°Π½Ρ‹. ΠŸΡƒΡΡ‚ΡŒ v это Π²Π΅ΠΊΡ‚ΠΎΡ€. Π’ΠΎΠ³Π΄Π° v/|v| Π΅ΡΡ‚ΡŒ ΠΎΡ€Ρ‚ Π² Ρ‚ΠΎΠΌ ΠΆΠ΅ самом Π½Π°ΠΏΡ€Π°Π²Π»Π΅Π½ΠΈΠΈ, Ρ‡Ρ‚ΠΎ ΠΈ v. Π’Π°ΠΊΠΈΠΌ ΠΎΠ±Ρ€Π°Π·ΠΎΠΌ, ΠΌΡ‹ ΠΈΠΌΠ΅Π΅ΠΌ
v/|v| = (cosΞΈ)i + (sinΞΈ)j
v = |v|[(cosθ)i + (sinθ)j]              УмноТая на |v|
v = |v|(cosΞΈ)i + |v|(sinΞΈ)j.

Π£Π³Π»Ρ‹ ΠΌΠ΅ΠΆΠ΄Ρƒ Π²Π΅ΠΊΡ‚ΠΎΡ€Π°ΠΌΠΈ

Когда Π²Π΅ΠΊΡ‚ΠΎΡ€ умноТаСтся Π½Π° скаляр, Ρ€Π΅Π·ΡƒΠ»ΡŒΡ‚Π°Ρ‚ΠΎΠΌ Π΅ΡΡ‚ΡŒ Π²Π΅ΠΊΡ‚ΠΎΡ€. Когда ΡΠΊΠ»Π°Π΄Ρ‹Π²Π°ΡŽΡ‚ΡΡ Π΄Π²Π° Π²Π΅ΠΊΡ‚ΠΎΡ€Π°, Ρ€Π΅Π·ΡƒΠ»ΡŒΡ‚Π°Ρ‚ΠΎΠΌ Ρ‚Π°ΠΊΠΆΠ΅ Π΅ΡΡ‚ΡŒ Π²Π΅ΠΊΡ‚ΠΎΡ€. Π’Π°ΠΊΠΈΠΌ ΠΎΠ±Ρ€Π°Π·ΠΎΠΌ, ΠΌΡ‹ ΠΌΠΎΠ³Π»ΠΈ Π±Ρ‹ ΠΎΠΆΠΈΠ΄Π°Ρ‚ΡŒ, Ρ‡Ρ‚ΠΎ ΠΏΡ€ΠΎΠΈΠ·Π²Π΅Π΄Π΅Π½ΠΈΠ΅ Π΄Π²ΡƒΡ… Π²Π΅ΠΊΡ‚ΠΎΡ€ΠΎΠ² Π΅ΡΡ‚ΡŒ Π²Π΅ΠΊΡ‚ΠΎΡ€, Π½ΠΎ это Π½Π΅ Ρ‚Π°ΠΊ. БкалярноС ΠΏΡ€ΠΎΠΈΠ·Π²Π΅Π΄Π΅Π½ΠΈΠ΅ Π΄Π²ΡƒΡ… Π²Π΅ΠΊΡ‚ΠΎΡ€ΠΎΠ² Π΅ΡΡ‚ΡŒ Π΄Π΅ΠΉΡΡ‚Π²ΠΈΡ‚Π΅Π»ΡŒΠ½ΠΎΠ΅ число ΠΈΠ»ΠΈ скаляр. Π­Ρ‚ΠΎΡ‚ Ρ€Π΅Π·ΡƒΠ»ΡŒΡ‚Π°Ρ‚ ΠΏΠΎΠ»Π΅Π·Π΅Π½ Π² Π½Π°Ρ…ΠΎΠΆΠ΄Π΅Π½ΠΈΠΈ ΡƒΠ³Π»Π° ΠΌΠ΅ΠΆΠ΄Ρƒ двумя Π²Π΅ΠΊΡ‚ΠΎΡ€Π°ΠΌΠΈ ΠΈ Π² ΠΎΠΏΡ€Π΅Π΄Π΅Π»Π΅Π½ΠΈΠΈ, ΡΠ²Π»ΡΡŽΡ‚ΡΡ Π»ΠΈ Π΄Π²Π° Π²Π΅ΠΊΡ‚ΠΎΡ€Π° пСрпСндикулярными.

БкалярноС ΠΏΡ€ΠΎΠΈΠ·Π²Π΅Π΄Π΅Π½ΠΈΠ΅ Π΄Π²ΡƒΡ… Π²Π΅ΠΊΡ‚ΠΎΡ€ΠΎΠ² u = 1, u2 > ΠΈ v = 1, v2 > is
u β€’ v = u1.v1 + u2.v2
(ΠžΠ±Ρ€Π°Ρ‚ΠΈΡ‚Π΅ Π²Π½ΠΈΠΌΠ°Π½ΠΈΠ΅, Ρ‡Ρ‚ΠΎ u1v1 + u2v2 Π΅ΡΡ‚ΡŒ скаляром, Π° Π½Π΅ Π²Π΅ΠΊΡ‚ΠΎΡ€ΠΎΠΌ.)

ΠŸΡ€ΠΈΠΌΠ΅Ρ€ 10НайдитС скалярноС ΠΏΡ€ΠΎΠΈΠ·Π²Π΅Π΄Π΅Π½ΠΈΠ΅, ΠΊΠΎΠ³Π΄Π°
u = , v = ΠΈ w = .
a)u β€’ w
b)w β€’ v

РСшСниС
a) u β€’ w = 2(- 3) + (- 5)1 = – 6 – 5 = – 11;
b) w β€’ v = (- 3)0 + 1(4) = 0 + 4 = 4.

БкалярноС ΠΏΡ€ΠΎΠΈΠ·Π²Π΅Π΄Π΅Π½ΠΈΠ΅ ΠΌΠΎΠΆΠ΅Ρ‚ Π±Ρ‹Ρ‚ΡŒ использовано для нахоТдСния ΡƒΠ³Π»Π° ΠΌΠ΅ΠΆΠ΄Ρƒ двумя Π²Π΅ΠΊΡ‚ΠΎΡ€Π°ΠΌΠΈ. Π£Π³ΠΎΠ» ΠΌΠ΅ΠΆΠ΄Ρƒ двумя Π²Π΅ΠΊΡ‚ΠΎΡ€Π°ΠΌΠΈ это самый малСнький ΠΏΠΎΠ»ΠΎΠΆΠΈΡ‚Π΅Π»ΡŒΠ½Ρ‹ΠΉ ΡƒΠ³ΠΎΠ», ΠΎΠ±Ρ€Π°Π·ΠΎΠ²Π°Π½Π½Ρ‹ΠΉ двумя Π½Π°ΠΏΡ€Π°Π²Π»Π΅Π½Π½Ρ‹ΠΌΠΈ ΠΎΡ‚Ρ€Π΅Π·ΠΊΠ°ΠΌΠΈ. Π’Π°ΠΊΠΈΠΌ ΠΎΠ±Ρ€Π°Π·ΠΎΠΌ, ΞΈ ΠΌΠ΅ΠΆΠ΄Ρƒ u ΠΈ v это Ρ‚ΠΎΡ‚ ΠΆΠ΅ самый ΡƒΠ³ΠΎΠ», Ρ‡Ρ‚ΠΎ ΠΈ ΠΌΠ΅ΠΆΠ΄Ρƒ v ΠΈ u, ΠΈ 0 ≀ ΞΈ ≀ Ο€.

Если ΞΈ Π΅ΡΡ‚ΡŒ ΡƒΠ³Π»ΠΎΠΌ ΠΌΠ΅ΠΆΠ΄Ρƒ двумя Π½Π΅Π½ΡƒΠ»Π΅Π²Ρ‹ΠΌΠΈ Π²Π΅ΠΊΡ‚ΠΎΡ€Π°ΠΌΠΈ u ΠΈ v, Ρ‚ΠΎΠ³Π΄Π°
cosΞΈ = (u β€’ v)/|u||v|.

ΠŸΡ€ΠΈΠΌΠ΅Ρ€ 11НайдитС ΡƒΠ³ΠΎΠ» ΠΌΠ΅ΠΆΠ΄Ρƒ u = ΠΈ v = .

РСшСниС НачнСм с нахоТдСния u β€’ v, |u|, ΠΈ |v|:
u β€’ v = 3(- 4) + 7(2) = 2,
|u| = √32 + 72 = √58, and
|v| = √(- 4)2 + 22 = √20.
TΠΎΠ³Π΄Π°
cosΞ± = (u β€’ v)/|u||v| = 2/√58.√20
Ξ± = cos– 1(2/√58.√20)
Ξ± β‰ˆ 86,6Β°.

РавновСсиС сил

Когда нСсколько сил Π΄Π΅ΠΉΡΡ‚Π²ΡƒΡŽΡ‚ Π½Π° ΠΎΠ΄Π½Ρƒ ΠΈ Ρ‚Ρƒ ΠΆΠ΅ Ρ‚ΠΎΡ‡ΠΊΡƒ Π½Π° ΠΎΠ±ΡŠΠ΅ΠΊΡ‚Π΅, ΠΈΡ… вСкторная сумма Π΄ΠΎΠ»ΠΆΠ½Π° Π±Ρ‹Ρ‚ΡŒ Ρ€Π°Π²Π½Π° нуля, для Ρ‚ΠΎΠ³ΠΎ, Ρ‡Ρ‚ΠΎΠ±Ρ‹ Π±Ρ‹Π» баланс. Когда Π΅ΡΡ‚ΡŒ баланс сил, Ρ‚ΠΎ ΠΎΠ±ΡŠΠ΅ΠΊΡ‚ являСтся стационарным ΠΈΠ»ΠΈ двиТСтся ΠΏΠΎ прямой Π»ΠΈΠ½ΠΈΠΈ, Π±Π΅Π· ускорСния. Π’ΠΎΡ‚ Ρ„Π°ΠΊΡ‚, Ρ‡Ρ‚ΠΎ вСкторная сумма Π΄ΠΎΠ»ΠΆΠ½Π° Π±Ρ‹Ρ‚ΡŒ Ρ€Π°Π²Π½Π° Π½ΡƒΠ»ΡŽ Π²Ρ‹Π²ΠΎΠ΄Π° для получСния баланса, ΠΈ Π½Π°ΠΎΠ±ΠΎΡ€ΠΎΡ‚, позволяСт Ρ€Π΅ΡˆΠ°Ρ‚ΡŒ Π½Π°ΠΌ ΠΌΠ½ΠΎΠ³ΠΈΠ΅ ΠΏΡ€ΠΈΠΊΠ»Π°Π΄Π½Ρ‹Π΅ Π·Π°Π΄Π°Ρ‡ΠΈ с участиСм сил.

ΠŸΡ€ΠΈΠΌΠ΅Ρ€ 12 ПодвСсной Π±Π»ΠΎΠΊ 350- Ρ„ΡƒΠ½Ρ‚ΠΎΠ²Ρ‹ΠΉ Π±Π»ΠΎΠΊ подвСшСн с ΠΏΠΎΠΌΠΎΡ‰ΡŒΡŽ Π΄Π²ΡƒΡ… ΠΊΠ°Π±Π΅Π»Π΅ΠΉ. ΠΎΡΡ‚Π°Π»ΠΎΡΡŒ. Π’ Ρ‚ΠΎΡ‡ΠΊΠ΅ А Π΅ΡΡ‚ΡŒ Ρ‚Ρ€ΠΈ силы, Π΄Π΅ΠΉΡΡ‚Π²ΡƒΡŽΡ‰ΠΈΠ΅ Ρ‚Π°ΠΊ: W Π±Π»ΠΎΠΊ тянСт Π²Π½ΠΈΠ·, Π° R ΠΈ S (Π΄Π²Π° кабСля) тянут Π²Π²Π΅Ρ€Ρ… ΠΈ Π½Π°Ρ€ΡƒΠΆΡƒ. НайдитС Π½Π°Π³Ρ€ΡƒΠ·ΠΊΡƒ ΠΊΠ°ΠΆΠ΄ΠΎΠ³ΠΎ кабСля.

РСшСниС НарисуСм Π΄ΠΈΠ°Π³Ρ€Π°ΠΌΠΌΡƒ с Π½Π°Ρ‡Π°Π»ΡŒΠ½Ρ‹ΠΌΠΈ Ρ‚ΠΎΡ‡ΠΊΠ°ΠΌΠΈ ΠΊΠ°ΠΆΠ΄ΠΎΠ³ΠΎ Π²Π΅ΠΊΡ‚ΠΎΡ€Π° Π² Π½Π°Ρ‡Π°Π»Π΅ ΠΊΠΎΠΎΠΎΡ€Π΄ΠΈΠ½Π°Ρ‚. Для баланса, сумма Π²Π΅ΠΊΡ‚ΠΎΡ€ΠΎΠ² Π΄ΠΎΠ»ΠΆΠ½Π° Π±Ρ‹Ρ‚ΡŒ Ρ€Π°Π²Π½Π° О:

R + S + W = О.
ΠœΡ‹ ΠΌΠΎΠΆΠ΅ΠΌ Π²Ρ‹Ρ€Π°Π·ΠΈΡ‚ΡŒ ΠΊΠ°ΠΆΠ΄Ρ‹ΠΉ Π²Π΅ΠΊΡ‚ΠΎΡ€ Ρ‡Π΅Ρ€Π΅Π· Π΅Π³ΠΎ Π²Π΅Π»ΠΈΡ‡ΠΈΠ½Ρƒ ΠΈ ΡƒΠ³ΠΎΠ» ΠΎΠ±Π·ΠΎΡ€Π° :
R = |R|[(cos125Β°)i + (sin125Β°)j],
S = |S|[(cos37Β°)i + (sin37Β°)j], ΠΈ
W = |W|[(cos270Β°)i + (sin270Β°)j]
= 350(cos270Β°)i + 350(sin270Β°)j
= -350j Β Β Β Β Β Β Β Β  cos270Β° = 0; sin270Β° = – 1.
ЗамСняя R, S, ΠΈ W in R + S + W + O, ΠΌΡ‹ ΠΈΠΌΠ΅Π΅ΠΌ
[|R|(cos125Β°) + |S|(cos37Β°)]i + [|R|(sin125Β°) + |S|(sin37Β°) – 350]j = 0i + 0j.
Π­Ρ‚ΠΎ Π΄Π°Π΅Ρ‚ Π½Π°ΠΌ систСму ΡƒΡ€Π°Π²Π½Π΅Π½ΠΈΠΉ:
|R|(cos125Β°) + |S|(cos37Β°) = 0,
|R|(sin125Β°) + |S|(sin37Β°) – 350 = 0.
РСшая эту систСму, ΠΌΡ‹ ΠΏΠΎΠ»ΡƒΡ‡Π°Π΅ΠΌ
|R| β‰ˆ 280 ΠΈ |S| β‰ˆ 201.
Π’Π°ΠΊΠΈΠΌ ΠΎΠ±Ρ€Π°Π·ΠΎΠΌ, Π½Π°Π³Ρ€ΡƒΠ·ΠΊΠ° Π½Π° ΠΊΠ°Π±Π΅Π»ΠΈ 280 Ρ„ΡƒΠ½Ρ‚ΠΎΠ² ΠΈ 201 Ρ„ΡƒΠ½Ρ‚.

ΠœΠΎΠΌΠ΅Π½Ρ‚ Π²Π΅ΠΊΡ‚ΠΎΡ€Π° силы – ΠΎΠΏΡ€Π΅Π΄Π΅Π»Π΅Π½ΠΈΠ΅ ΠΈ свойства

ΠœΠΎΠΌΠ΅Π½Ρ‚ Π²Π΅ΠΊΡ‚ΠΎΡ€Π° силы β€” Π’Π²Π΅Π΄Π΅Π½ΠΈΠ΅

ЀизичСская Π²Π΅Π»ΠΈΡ‡ΠΈΠ½Π° Β«ΠœΠΎΠΌΠ΅Π½Ρ‚ Π²Π΅ΠΊΡ‚ΠΎΡ€Π° силы» ΠΈΠΌΠ΅Π΅Ρ‚ прямоС ΠΎΡ‚Π½ΠΎΡˆΠ΅Π½ΠΈΠ΅ ΠΊ Π²Ρ€Π°Ρ‰Π°Ρ‚Π΅Π»ΡŒΠ½ΠΎΠΌΡƒ двиТСнию ΠΈ Π²Ρ…ΠΎΠ΄ΠΈΡ‚ Π² состав ΠΎΠ΄Π½ΠΎΠ³ΠΎ Π²Π°ΠΆΠ½ΠΎΠ³ΠΎΒ ΡΠΎΠΎΡ‚Π½ΠΎΡˆΠ΅Π½ΠΈΡ, Π½Π°Π·Ρ‹Π²Π°Π΅ΠΌΠΎΠ³ΠΎ ΡƒΡ€Π°Π²Π½Π΅Π½ΠΈΠ΅ΠΌ ΠΌΠΎΠΌΠ΅Π½Ρ‚ΠΎΠ². Но Π΄Π°Π²Π°ΠΉΡ‚Π΅ Ρ€Π°Π·Π±ΠΈΡ€Π°Ρ‚ΡŒΡΡ ΠΏΠΎ порядку. Для Π½Π°Ρ‡Π°Π»Π° Π½Π°ΠΌ Π½Π΅ΠΎΠ±Ρ…ΠΎΠ΄ΠΈΠΌΠΎ провСсти ряд построСний, Π±Π΅Π· ΠΊΠΎΡ‚ΠΎΡ€Ρ‹Ρ… ΠΎΠΏΡ€Π΅Π΄Π΅Π»Π΅Π½ΠΈΠ΅ ΠΌΠΎΠΌΠ΅Π½Ρ‚Π° Π²Π΅ΠΊΡ‚ΠΎΡ€Π° силы Π±ΡƒΠ΄Π΅Ρ‚ нСясным.ΠŸΡƒΡΡ‚ΡŒ сущСствуСт нСкоторая Ρ‚ΠΎΡ‡ΠΊΠ° О. ΠžΡ‚Π½ΠΎΡΠΈΡ‚Π΅Π»ΡŒΠ½ΠΎ этой Ρ‚ΠΎΡ‡ΠΊΠΈ, Π½Π°Π·Ρ‹Π²Π°Π΅ΠΌΠΎΠΉ Π½Π°Ρ‡Π°Π»ΠΎΠΌ ΠΈΠ»ΠΈ полюсом, ΠΌΡ‹ Π±ΡƒΠ΄Π΅ΠΌ Ρ€Π°ΡΡΠΌΠ°Ρ‚Ρ€ΠΈΠ²Π°Ρ‚ΡŒ (Π° ΠΏΡ€Π°Π²ΠΈΠ»ΡŒΠ½Π΅Π΅ Π±ΡƒΠ΄Π΅Ρ‚ ΡΠΊΠ°Π·Π°Ρ‚ΡŒ Π½Π°Ρ…ΠΎΠ΄ΠΈΡ‚ΡŒ ΠΈΠ»ΠΈ ΠΎΠΏΡ€Π΅Π΄Π΅Π»ΡΡ‚ΡŒ) ΠΌΠΎΠΌΠ΅Π½Ρ‚ Π²Π΅ΠΊΡ‚ΠΎΡ€Π° силы (ΠΌΠΎΠΌΠ΅Π½Ρ‚ΠΎΠΌ силы ), Π° Ρ‚Π°ΠΊ ΠΆΠ΅ ΠΌΠΎΠΌΠ΅Π½Ρ‚ ΠΈΠΌΠΏΡƒΠ»ΡŒΡΠ° (ΠΌΠΎΠΌΠ΅Π½Ρ‚ ΠΈΠΌΠΏΡƒΠ»ΡŒΡΠ° ).

ΠŸΠΎΡΡ‚Ρ€ΠΎΠΈΠΌ ΠΈΠ· ΠΎΠ±ΠΎΠ·Π½Π°Ρ‡Π΅Π½Π½ΠΎΠ³ΠΎ Π½Π°ΠΌΠΈ полюса (Ρ‚ΠΎΡ‡ΠΊΠΈ О) радиус Π²Π΅ΠΊΡ‚ΠΎΡ€ ΠΊ Ρ‚ΠΎΡ‡ΠΊΠ΅ прилоТСния силы . ΠžΠ±Ρ€Π°Ρ‚ΠΈΡ‚Π΅ Π²Π½ΠΈΠΌΠ°Π½ΠΈΠ΅ Π½Π° рисунок ΠΏΡ€ΠΈΠ²Π΅Π΄Π΅Π½Π½Ρ‹ΠΉ Π²Ρ‹ΡˆΠ΅ β€” ΠΎΠ½ ΠΈΠ»Π»ΡŽΡΡ‚Ρ€ΠΈΡ€ΡƒΠ΅Ρ‚ всС наши рассуТдСния.

ΠœΠΎΠΌΠ΅Π½Ρ‚ Π²Π΅ΠΊΡ‚ΠΎΡ€Π° силы β€” ΠžΠΏΡ€Π΅Π΄Π΅Π»Π΅Π½ΠΈΠ΅

Π’Ρ‹ΠΏΠΎΠ»Π½ΠΈΠ² всС Π²Ρ‹ΡˆΠ΅ΠΏΠ΅Ρ€Π΅Ρ‡ΠΈΡΠ»Π΅Π½Π½ΠΎΠ΅, ΠΌΡ‹ ΠΌΠΎΠΆΠ΅ΠΌ ΠΏΡ€ΠΈΡΡ‚ΡƒΠΏΠΈΡ‚ΡŒ ΠΊ Π½Π°Ρ…ΠΎΠΆΠ΄Π΅Π½ΠΈΡŽ ΠΌΠΎΠΌΠ΅Π½Ρ‚Π° Π²Π΅ΠΊΡ‚ΠΎΡ€Π° силы (ΠΌΠΎΠΌΠ΅Π½Ρ‚Π° ). Π˜Ρ‚Π°ΠΊ, ΠΌΠΎΠΌΠ΅Π½Ρ‚ Π²Π΅ΠΊΡ‚ΠΎΡ€Π° силы это Π²Π΅ΠΊΡ‚ΠΎΡ€, ΠΏΠΎΠ»ΡƒΡ‡Π°Π΅ΠΌΡ‹ΠΉ ΠΏΡ€ΠΈ Π²Π΅ΠΊΡ‚ΠΎΡ€Π½ΠΎΠΌ ΠΏΠ΅Ρ€Π΅ΠΌΠ½ΠΎΠΆΠ΅Π½ΠΈΠΈ ΠΈ . ΠžΠ±ΠΎΠ·Π½Π°Ρ‡Π°Ρ‚ΡŒ ΠΌΠΎΠΌΠ΅Π½Ρ‚ силы ΠΌΡ‹ Π±ΡƒΠ΄Π΅ΠΌ Ρ‡Π΅Ρ€Π΅Π· . НиТС ΠΏΡ€ΠΈΠ²Π΅Π΄Π΅Π½Π° Ρ„ΠΎΡ€ΠΌΡƒΠ»Π°, ΡΠΎΠΎΡ‚Π²Π΅Ρ‚ΡΡ‚Π²ΡƒΡŽΡ‰Π°Ρ ΠΏΡ€ΠΈΠ²Π΅Π΄Π΅Π½Π½ΠΎΠΌΡƒ ΠΎΠΏΡ€Π΅Π΄Π΅Π»Π΅Π½ΠΈΡŽ.

Как Π²ΠΈΠ΄Π½ΠΎ ΠΈΠ· Ρ„ΠΎΡ€ΠΌΡƒΠ»Ρ‹, Π½Π°ΠΏΡ€Π°Π²Π»Π΅Π½ΠΈΠ΅ Π²Π΅ΠΊΡ‚ΠΎΡ€Π° зависит ΠΎΡ‚ полоТСния Π²Ρ‹Π±Ρ€Π°Π½Π½ΠΎΠ³ΠΎ полюса (ΠΌΠΎΠΆΠ΅Ρ‚ Π±Ρ‹Ρ‚ΡŒ ΠΈΠ·ΠΌΠ΅Π½Π΅Π½ΠΎ Π½Π°ΠΏΡ€Π°Π²Π»Π΅Π½ΠΈΠ΅ Π²Π΅ΠΊΡ‚ΠΎΡ€Π° ) ΠΈ ΠΎΡ‚ направлСния Π²Π΅ΠΊΡ‚ΠΎΡ€Π° силы .

ΠœΠΎΠΌΠ΅Π½Ρ‚Π° Π²Π΅ΠΊΡ‚ΠΎΡ€Π° силы β€” Бвойства

  1. ΠœΠΎΠΌΠ΅Π½Ρ‚ Π²Π΅ΠΊΡ‚ΠΎΡ€Π° силы Π½Π΅ измСняСтся ΠΏΡ€ΠΈ пСрСносС Ρ‚ΠΎΡ‡ΠΊΠΈ ΠΏΡ€ΠΈΠ»ΠΎΠΆΠ΅Π½ΠΈΠ΅ силы вдоль Π»ΠΈΠ½ΠΈΠΈ дСйствия этой силы.
  2. ΠœΠΎΠΌΠ΅Π½Ρ‚ Π²Π΅ΠΊΡ‚ΠΎΡ€Π° силы ΠΎΡ‚Π½ΠΎΡΠΈΡ‚Π΅Π»ΡŒΠ½ΠΎ Π²Ρ‹Π±Ρ€Π°Π½Π½ΠΎΠ³ΠΎ полюса для Π½Π΅ΡΠΊΠΎΠ»ΡŒΠΊΠΈΡ… сил Ρ€Π°Π²Π΅Π½ гСомСтричСской суммС ΠΌΠΎΠΌΠ΅Π½Ρ‚ΠΎΠ² сил, рассчитанных для ΠΊΠ°ΠΆΠ΄ΠΎΠΉ силы ΠΏΠΎ ΠΎΡ‚Π΄Π΅Π»ΡŒΠ½ΠΎΡΡ‚ΠΈ.

Π”ΠΎΠΊΠ°ΠΆΠ΅ΠΌ ΡΠΏΡ€Π°Π²Π΅Π΄Π»ΠΈΠ²ΠΎΡΡ‚ΡŒ ΠΏΠ΅Ρ€Π²ΠΎΠ³ΠΎ ΠΏΡƒΠ½ΠΊΡ‚Π°. Π”Π»ΠΈΠ½Π° Π²Π΅ΠΊΡ‚ΠΎΡ€Π° , ΠΏΠΎΠ»ΡƒΡ‡Π΅Π½Π½ΠΎΠ³ΠΎ Π½Π°ΠΌΠΈ, Ρ€Π°Π²Π½Π° ΠΏΠ»ΠΎΡ‰Π°Π΄ΠΈ ΠΏΠ°Ρ€Π°Π»Π»Π΅Π»ΠΎΠ³Ρ€Π°ΠΌΠΌΠ° OABC (ΡˆΠΊΠΎΠ»ΡŒΠ½Ρ‹ΠΉ курс ΠΌΠ°Ρ‚Π΅ΠΌΠ°Ρ‚ΠΈΠΊΠΈ). Если ΠΌΡ‹ смСстим Π²Π΅ΠΊΡ‚ΠΎΡ€ силы вдоль Π»ΠΈΠ½ΠΈΠΈ Π΅Π΅ дСйствия (смотри рисунок Π² Π²Ρ‹ΡˆΠ΅), Ρ‚ΠΎ ΠΌΡ‹ ΠΏΠΎΠ»ΡƒΡ‡ΠΈΠΌ ΠΏΠ°Ρ€Π°Π»Π»Π΅Π»ΠΎΠ³Ρ€Π°ΠΌΠΌ ΠžΠβ€™Bβ€˜C, ΠΏΠ»ΠΎΡ‰Π°Π΄ΡŒ ΠΊΠΎΡ‚ΠΎΡ€ΠΎΠ³ΠΎ Ρ€Π°Π²Π½Π° ΠΏΠ»ΠΎΡ‰Π°Π΄ΠΈ ΠΏΠ΅Ρ€Π²ΠΎΠ³ΠΎ ΠΏΠ°Ρ€Π°Π»Π»Π΅Π»ΠΎΠ³Ρ€Π°ΠΌΠΌΠ°. А Π΄ΠΎΡ‡ΠΈΡ‚Π°Π² ΠΏΡ€Π°Π²ΠΈΠ»Π° Π²Π΅ΠΊΡ‚ΠΎΡ€Π½ΠΎΠ³ΠΎ умноТСния Π΄ΠΎ ΠΊΠΎΠ½Ρ†Π°, Π²Ρ‹ ΠΏΠΎΠΉΠΌΠ΅Ρ‚Π΅, Ρ‡Ρ‚ΠΎ ΠΈ Π½Π°ΠΏΡ€Π°Π²Π»Π΅Π½ΠΈΠ΅ Π²Π΅ΠΊΡ‚ΠΎΡ€Π° ΠΎΡΡ‚Π°Π»ΠΎΡΡŒ ΠΏΡ€Π΅ΠΆΠ½ΠΈΠΌ.
Π‘ΠΏΡ€Π°Π²Π΅Π΄Π»ΠΈΠ²ΠΎΡΡ‚ΡŒ Π²Ρ‚ΠΎΡ€ΠΎΠ³ΠΎ ΠΏΡƒΠ½ΠΊΡ‚Π° ΠΌΠΎΠΆΠ½ΠΎ Π΄ΠΎΠΊΠ°Π·Π°Ρ‚ΡŒ вспомнив Π΅Ρ‰Π΅ ΠΎΠ΄Π½ΠΎ свойство Π²Π΅ΠΊΡ‚ΠΎΡ€Π½ΠΎΠ³ΠΎ умноТСния β€” . Π—Π°ΠΌΠ΅Π½ΠΈΠ² Π²Π΅ΠΊΡ‚ΠΎΡ€Π½Ρ‹Π΅ произвСдСния ΠΈΡ… значСниями, ΠΌΡ‹ ΠΏΠΎΠ»ΡƒΡ‡ΠΈΠΌ матСматичСскоС Π²Ρ‹Ρ€Π°ΠΆΠ΅Π½ΠΈΠ΅ для Π²Ρ‚ΠΎΡ€ΠΎΠ³ΠΎ свойства ΠΌΠΎΠΌΠ΅Π½Ρ‚Π° Π²Π΅ΠΊΡ‚ΠΎΡ€Π° сил.

Post Views: 11Β 067

ΠŸΠΎΡ…ΠΎΠΆΠ΅Π΅

КомплСксная ΠΏΠ»ΠΎΡΠΊΠΎΡΡ‚ΡŒ | ΠœΠ°Ρ‚Π΅ΠΌΠ°Ρ‚ΠΈΠΊΠ°, которая ΠΌΠ½Π΅ нравится

Рассмотрим ΠΊΠΎΠΎΡ€Π΄ΠΈΠ½Π°Ρ‚Π½ΡƒΡŽ ΠΏΠ»ΠΎΡΠΊΠΎΡΡ‚ΡŒ ΠΈ поставим Π² соотвСтствиС ΠΊΠ°ΠΆΠ΄ΠΎΠΌΡƒ комплСксному числу Ρ‚ΠΎΡ‡ΠΊΡƒ с ΠΊΠΎΠΎΡ€Π΄ΠΈΠ½Π°Ρ‚Π°ΠΌΠΈ . Π’ΠΎΠ³Π΄Π° устанавливаСтся Π²Π·Π°ΠΈΠΌΠ½ΠΎ ΠΎΠ΄Π½ΠΎΠ·Π½Π°Ρ‡Π½ΠΎΠ΅ соотвСтствиС ΠΌΠ΅ΠΆΠ΄Ρƒ ΠΏΠΎΠ»Π΅ΠΌ ΠΈ мноТСством Ρ‚ΠΎΡ‡Π΅ΠΊ ΠΊΠΎΠΎΡ€Π΄ΠΈΠ½Π°Ρ‚Π½ΠΎΠΉ плоскости. ΠšΠΎΠΎΡ€Π΄ΠΈΠ½Π°Ρ‚Π½ΡƒΡŽ ΠΏΠ»ΠΎΡΠΊΠΎΡΡ‚ΡŒ Π² этом случаС Π±ΡƒΠ΄Π΅ΠΌ Π½Π°Π·Ρ‹Π²Π°Ρ‚ΡŒ комплСксной ΠΏΠ»ΠΎΡΠΊΠΎΡΡ‚ΡŒΡŽ. Ось абсцисс Π±ΡƒΠ΄Π΅ΠΌ Π½Π°Π·Ρ‹Π²Π°Ρ‚ΡŒ вСщСствСнной осью, Π° ось ΠΎΡ€Π΄ΠΈΠ½Π°Ρ‚ β€” ΠΌΠ½ΠΈΠΌΠΎΠΉ осью.

Π‘ ΠΊΠ°ΠΆΠ΄ΠΎΠΉ Ρ‚ΠΎΡ‡ΠΊΠΎΠΉ комплСксной плоскости ΠΌΠΎΠΆΠ½ΠΎ ΡΠ²ΡΠ·Π°Ρ‚ΡŒ Π²Π΅ΠΊΡ‚ΠΎΡ€, ΠΈΠ΄ΡƒΡ‰ΠΈΠΉ ΠΈΠ· нуля Π² эту Ρ‚ΠΎΡ‡ΠΊΡƒ (радиус-Π²Π΅ΠΊΡ‚ΠΎΡ€). ΠšΠΎΠΎΡ€Π΄ΠΈΠ½Π°Ρ‚Ρ‹ этого Π²Π΅ΠΊΡ‚ΠΎΡ€Π° β€” вСщСствСнная ΠΈ мнимая части Π΅Π³ΠΎ ΠΊΠΎΠ½Ρ†Π°.

Β  Β 

Радиус-Π²Π΅ΠΊΡ‚ΠΎΡ€ числа Ρ€Π°Π²Π΅Π½ суммС радиус-Π²Π΅ΠΊΡ‚ΠΎΡ€ΠΎΠ² чисСл ΠΈ .

Аналогично с Π²Ρ‹Ρ‡ΠΈΡ‚Π°Π½ΠΈΠ΅ΠΌ.

ΠŸΠΎΠ»ΡΡ€Π½Π°Ρ систСма ΠΊΠΎΠΎΡ€Π΄ΠΈΠ½Π°Ρ‚

Π’Ρ‹Π±Π΅Ρ€Π΅ΠΌ Π² плоскости Ρ‚ΠΎΡ‡ΠΊΡƒ (полюс) ΠΈ Π»ΡƒΡ‡ с Π½Π°Ρ‡Π°Π»ΠΎΠΌ Π² Ρ‚ΠΎΡ‡ΠΊΠ΅ (ΠΏΠΎΠ»ΡΡ€Π½ΡƒΡŽ ΠΏΠΎΠ»ΡƒΠΎΡΡŒ). Π’ΠΎΠ³Π΄Π° ΠΏΠΎΠ»ΠΎΠΆΠ΅Π½ΠΈΠ΅ любой Ρ‚ΠΎΡ‡ΠΊΠΈ плоскости , ΠΎΡ‚Π»ΠΈΡ‡Π½ΠΎΠΉ ΠΎΡ‚ Ρ‚ΠΎΡ‡ΠΊΠΈ , ΠΎΠ΄Π½ΠΎΠ·Π½Π°Ρ‡Π½ΠΎ характСризуСтся двумя числами: Π΄Π»ΠΈΠ½ΠΎΠΉ ΠΎΡ‚Ρ€Π΅Π·ΠΊΠ° (полярный радиус Ρ‚ΠΎΡ‡ΠΊΠΈ ) ΠΈ Π²Π΅Π»ΠΈΡ‡ΠΈΠ½ΠΎΠΉ ΡƒΠ³Π»Π°, ΠΎΠ±Ρ€Π°Π·ΠΎΠ²Π°Π½Π½ΠΎΠ³ΠΎ Π»ΡƒΡ‡ΠΎΠΌ с полярной ΠΏΠΎΠ»ΡƒΠΎΡΡŒΡŽ (полярный ΡƒΠ³ΠΎΠ» Ρ‚ΠΎΡ‡ΠΊΠΈ ). ΠŸΡ€ΠΈ этом полярный ΡƒΠ³ΠΎΠ» отсчитываСтся Π² Π½Π΅ΠΊΠΎΡ‚ΠΎΡ€ΠΎΠΌ фиксированном Π½Π°ΠΏΡ€Π°Π²Π»Π΅Π½ΠΈΠΈ, ΠΎΠ΄Π½ΠΎΠΌ ΠΈ Ρ‚ΠΎΠΌ ΠΆΠ΅ для всСх Ρ‚ΠΎΡ‡Π΅ΠΊ.

ΠŸΠΎΠ»ΡΡ€Π½Ρ‹ΠΉ ΡƒΠ³ΠΎΠ» ΠΎΠΏΡ€Π΅Π΄Π΅Π»Π΅Π½ с Ρ‚ΠΎΡ‡Π½ΠΎΡΡ‚ΡŒΡŽ Π΄ΠΎ . ΠŸΠΎΠ»ΡΡ€Π½Ρ‹ΠΉ радиус Ρ‚ΠΎΡ‡ΠΊΠΈ считаСтся Ρ€Π°Π²Π½Ρ‹ΠΌ Π½ΡƒΠ»ΡŽ, Π° полярного ΡƒΠ³Π»Π° Ρƒ Π½Π΅Π΅ Π½Π΅Ρ‚.

ΠœΠΎΠ΄ΡƒΠ»ΡŒ ΠΈ Π°Ρ€Π³ΡƒΠΌΠ΅Π½Ρ‚ комплСксного числа

ΠœΠΎΠ΄ΡƒΠ»Π΅ΠΌ комплСксного числа называСтся расстояниС ΠΎΡ‚ Π΄ΠΎ нуля Π½Π° комплСксной плоскости.

ΠžΠ±ΠΎΠ·Π½Π°Ρ‡Π΅Π½ΠΈΠ΅. . (ΠœΠΎΠ΄ΡƒΠ»ΡŒ β€” Π΄Π»ΠΈΠ½Π° радиус-Π²Π΅ΠΊΡ‚ΠΎΡ€Π°.) Если , это ΠΎΠΏΡ€Π΅Π΄Π΅Π»Π΅Π½ΠΈΠ΅ Π½Π΅ ΠΏΡ€ΠΎΡ‚ΠΈΠ²ΠΎΡ€Π΅Ρ‡ΠΈΡ‚ ΠΎΠΏΡ€Π΅Π΄Π΅Π»Π΅Π½ΠΈΡŽ модуля вСщСствСнного числа.

ΠŸΡƒΡΡ‚ΡŒ ΠΈ β€” комплСксныС числа. β€” Π΄Π»ΠΈΠ½Π° радиус-Π²Π΅ΠΊΡ‚ΠΎΡ€Π° числа β€” Π΄Π»ΠΈΠ½Π° Π²Π΅ΠΊΡ‚ΠΎΡ€Π° β€” расстояниС ΠΌΠ΅ΠΆΠ΄Ρƒ ΠΈ Π½Π° комплСксной плоскости.

ΠŸΡƒΡΡ‚ΡŒ . Π’ΠΎΠ³Π΄Π° Π‘Π»Π΅Π΄ΠΎΠ²Π°Ρ‚Π΅Π»ΡŒΠ½ΠΎ, .

Π’Π΅ΠΎΡ€Π΅ΠΌΠ°. .

Π”ΠΎΠΊΠ°Π·Π°Ρ‚Π΅Π»ΡŒΡΡ‚Π²ΠΎ.

Β  Β 

Π’Π΅ΠΎΡ€Π΅ΠΌΠ°.

Β  Β 

Π”ΠΎΠΊΠ°Π·Π°Ρ‚Π΅Π»ΡŒΡΡ‚Π²ΠΎ Π°Π½Π°Π»ΠΎΠ³ΠΈΡ‡Π½ΠΎ.

Π’Π΅ΠΎΡ€Π΅ΠΌΠ°.

Β  Β 

Π”ΠΎΠΊΠ°Π·Π°Ρ‚Π΅Π»ΡŒΡΡ‚Π²ΠΎ. Π‘Π»Π΅Π΄ΡƒΠ΅Ρ‚ ΠΈΠ· опрСдСлСния модуля ΠΈ гСомСтричСского смысла слоТСния ΠΈ вычитания комплСксных чисСл.

ΠžΠΏΡ€Π΅Π΄Π΅Π»Π΅Π½ΠΈΠ΅. ΠŸΡƒΡΡ‚ΡŒ β€” комплСксноС число, Π½Π΅ Ρ€Π°Π²Π½ΠΎΠ΅ Π½ΡƒΠ»ΡŽ, ΠΏΡƒΡΡ‚ΡŒ , Ρ‚ΠΎΠ³Π΄Π°

Β  Β 

Π‘Π»Π΅Π΄ΠΎΠ²Π°Ρ‚Π΅Π»ΡŒΠ½ΠΎ, Ρ‚ΠΎΡ‡ΠΊΠ° Π»Π΅ΠΆΠΈΡ‚ Π½Π° Π΅Π΄ΠΈΠ½ΠΈΡ‡Π½ΠΎΠΉ окруТности с Ρ†Π΅Π½Ρ‚Ρ€ΠΎΠΌ Π² Π½ΡƒΠ»Π΅.

БущСствуСт вСщСствСнноС число Ρ‚Π°ΠΊΠΎΠ΅, Ρ‡Ρ‚ΠΎ

Β  Β 

Π’Π°ΠΊΠΈΡ… чисСл бСсконСчно ΠΌΠ½ΠΎΠ³ΠΎ, ΠΈ Π»ΡŽΠ±Ρ‹Π΅ Π΄Π²Π° ΠΈΠ· Π½ΠΈΡ… Ρ€Π°Π·Π»ΠΈΡ‡Π°ΡŽΡ‚ΡΡ Π½Π° , Π³Π΄Π΅ β€” ΠΏΡ€ΠΎΠΈΠ·Π²ΠΎΠ»ΡŒΠ½ΠΎΠ΅ Ρ†Π΅Π»ΠΎΠ΅ число. Π›ΡŽΠ±ΠΎΠ΅ Ρ‚Π°ΠΊΠΎΠ΅ число называСтся Π°Ρ€Π³ΡƒΠΌΠ΅Π½Ρ‚ΠΎΠΌ числа .

ΠžΠ±ΠΎΠ·Π½Π°Ρ‡Π΅Π½ΠΈΠ΅. .

Β  Β 

Π˜Ρ‚Π°ΠΊ, любоС комплСксноС число ΠΌΠΎΠΆΠ½ΠΎ ΠΏΡ€Π΅Π΄ΡΡ‚Π°Π²ΠΈΡ‚ΡŒ Π² Π²ΠΈΠ΄Π΅

Β  Β 

Π³Π΄Π΅ .

Π’Π°ΠΊΠΎΠ΅ прСдставлСниС комплСксного числа называСтся Π΅Π³ΠΎ тригономСтричСской Ρ„ΠΎΡ€ΠΌΠΎΠΉ.

Π€ΠΎΡ€ΠΌΡƒΠ»Π° ΠœΡƒΠ°Π²Ρ€Π°

Абрахам Π΄Π΅ ΠœΡƒΠ°Π²Ρ€ (1667–1754) Π±Ρ‹Π» Π²Ρ‹Ρ…ΠΎΠ΄Ρ†Π΅ΠΌ ΠΈΠ· Π€Ρ€Π°Π½Ρ†ΠΈΠΈ, ΠΏΡ€ΠΎΠΆΠΈΠ» Π΄ΠΎΠ»Π³ΡƒΡŽ Тизнь Π² Англии ΠΈ ΡƒΠΌΠ΅Ρ€ Π² Π›ΠΎΠ½Π΄ΠΎΠ½Π΅. ΠžΠ΄Π½Π°ΠΆΠ΄Ρ‹, Π½Π΅Π·Π°Π΄ΠΎΠ»Π³ΠΎ Π΄ΠΎ смСрти, ΠœΡƒΠ°Π²Ρ€ заявил, Ρ‡Ρ‚ΠΎ Π΅ΠΌΡƒ Π½Π΅ΠΎΠ±Ρ…ΠΎΠ΄ΠΈΠΌΠΎ Π΅ΠΆΠ΅Π΄Π½Π΅Π²Π½ΠΎ ΡƒΠ²Π΅Π»ΠΈΡ‡ΠΈΠ²Π°Ρ‚ΡŒ врСмя сна Π½Π° 10–15 ΠΌΠΈΠ½ΡƒΡ‚. Достигнув, Ρ‚Π°ΠΊΠΈΠΌ ΠΎΠ±Ρ€Π°Π·ΠΎΠΌ, Π² суммС ΠΏΡ€ΠΎΠ΄ΠΎΠ»ΠΆΠΈΡ‚Π΅Π»ΡŒΠ½ΠΎΡΡ‚ΡŒ сна Π±ΠΎΠ»Π΅Π΅ 23 часов, ΠΎΠ½ Π½Π° ΡΠ»Π΅Π΄ΡƒΡŽΡ‰ΠΈΠ΅ сутки проспал всС 24 часа ΠΈ ΡƒΠΌΠ΅Ρ€ Π²ΠΎ снС.

Π‘ ΠΈΠΌΠ΅Π½Π΅ΠΌ ΠœΡƒΠ°Π²Ρ€Π° связаны ΠΏΡ€Π°Π²ΠΈΠ»Π° возвСдСния Π² -ю ΡΡ‚Π΅ΠΏΠ΅Π½ΡŒ ΠΈ извлСчСния корня -ΠΉ стСпСни для комплСксных чисСл. ΠœΡƒΠ°Π²Ρ€ ΠΌΠ½ΠΎΠ³ΠΎ занимался исслСдованиСм рядов ΠΈ Π΄ΠΎΠΊΠ°Π·Π°Π» частный случай ΠΏΡ€Π΅Π΄Π΅Π»ΡŒΠ½ΠΎΠΉ Ρ‚Π΅ΠΎΡ€Π΅ΠΌΡ‹ Π² Ρ‚Π΅ΠΎΡ€ΠΈΠΈ вСроятностСй.

Π›Π΅ΠΌΠΌΠ°. ΠŸΡƒΡΡ‚ΡŒ , Π³Π΄Π΅ . Π’ΠΎΠ³Π΄Π° .

Π”ΠΎΠΊΠ°Π·Π°Ρ‚Π΅Π»ΡŒΡΡ‚Π²ΠΎ.

Β  Β 

ΠŸΡƒΡΡ‚ΡŒ . Π’ΠΎΠ³Π΄Π° . Но

Β  Β 

Π‘Π»Π΅Π΄ΠΎΠ²Π°Ρ‚Π΅Π»ΡŒΠ½ΠΎ, числам ΠΈ соотвСтствуСт ΠΎΠ΄Π½Π° ΠΈ Ρ‚Π° ΠΆΠ΅ Ρ‚ΠΎΡ‡ΠΊΠ° числовой окруТности. Π—Π½Π°Ρ‡ΠΈΡ‚, . Π—Π½Π°Ρ‡ΠΈΡ‚, β€” ΠΎΠ΄Π½ΠΎ ΠΈΠ· Π·Π½Π°Ρ‡Π΅Π½ΠΈΠΉ .

Π’Π΅ΠΎΡ€Π΅ΠΌΠ°. ΠŸΡƒΡΡ‚ΡŒ . Π’ΠΎΠ³Π΄Π° .

Π”ΠΎΠΊΠ°Π·Π°Ρ‚Π΅Π»ΡŒΡΡ‚Π²ΠΎ. ΠŸΡ€Π΅Π΄ΡΡ‚Π°Π²ΠΈΠΌ ΠΈ Π² тригономСтричСской Ρ„ΠΎΡ€ΠΌΠ΅:

Β  Β 

Π’ΠΎΠ³Π΄Π°

Β  Β 

Π’Π°ΠΊ ΠΊΠ°ΠΊ , Ρ‚ΠΎ ΠΌΠΎΠΆΠ½ΠΎ ΠΏΡ€ΠΈΠΌΠ΅Π½ΠΈΡ‚ΡŒ Π»Π΅ΠΌΠΌΡƒ. β€” ΠΎΠ΄Π½ΠΎ ΠΈΠ· Π·Π½Π°Ρ‡Π΅Π½ΠΈΠΉ .

БлСдствиС.

Β  Β 

β€” Ρ„ΠΎΡ€ΠΌΡƒΠ»Π° ΠœΡƒΠ°Π²Ρ€Π°.

БлСдствиС. Π’ условиях Ρ‚Π΅ΠΎΡ€Π΅ΠΌΡ‹ .

Π”ΠΎΠΊΠ°Π·Π°Ρ‚Π΅Π»ΡŒΡΡ‚Π²ΠΎ.

Β  Β 

ΠŸΡƒΡΡ‚ΡŒ . По Ρ‚Π΅ΠΎΡ€Π΅ΠΌΠ΅

Β  Β 

БлСдствиС. Π€ΠΎΡ€ΠΌΡƒΠ»Π° ΠœΡƒΠ°Π²Ρ€Π° Π²Π΅Ρ€Π½Π° для любого Ρ†Π΅Π»ΠΎΠ³ΠΎ показатСля (цСлая ΠΎΡ‚Ρ€ΠΈΡ†Π°Ρ‚Π΅Π»ΡŒΠ½Π°Ρ ΠΈ нулСвая ΡΡ‚Π΅ΠΏΠ΅Π½ΡŒ для комплСксных чисСл вводится Ρ‚Π°ΠΊ ΠΆΠ΅, ΠΊΠ°ΠΊ ΠΈ для вСщСствСнных).

Π”ΠΎΠΊΠ°Π·Π°Ρ‚Π΅Π»ΡŒΡΡ‚Π²ΠΎ. ΠŸΡƒΡΡ‚ΡŒ .

Β  Β 

БлСдствиС.

Β  Β 

Π”ΠΎΠΊΠ°Π·Π°Ρ‚Π΅Π»ΡŒΡΡ‚Π²ΠΎ.

Β  Β 

Π—Π°Π΄Π°Ρ‡ΠΈ.

1. Π˜Π·ΠΎΠ±Ρ€Π°Π·ΠΈΡ‚Π΅ Π½Π° комплСксной плоскости числа

1)

2) ;

3) .

2. НайдитС ΠΌΠΎΠ΄ΡƒΠ»ΡŒ ΠΈ Π°Ρ€Π³ΡƒΠΌΠ΅Π½Ρ‚ чисСл

1) ;

2) ;

3) ;

4) .

3. Π˜Π·ΠΎΠ±Ρ€Π°Π·ΠΈΡ‚Π΅ Π½Π° комплСксной плоскости мноТСство Ρ‚ΠΎΡ‡Π΅ΠΊ z, Π·Π°Π΄Π°Π²Π°Π΅ΠΌΡ‹Ρ… условиями

1) ;

2) ;

3) ;

4) ;

5) ;

6) ;

7) ΠΈ .

4. НайдитС мноТСство Π·Π½Π°Ρ‡Π΅Π½ΠΈΠΉ Ρ„ΡƒΠ½ΠΊΡ†ΠΈΠΉ с ΠΎΠ±Π»Π°ΡΡ‚ΡŒΡŽ опрСдСлСния :

1) , ;

2) , .

5. Π˜ΡΠΏΠΎΠ»ΡŒΠ·ΡƒΠΉΡ‚Π΅ Ρ‚Ρ€ΠΈΠ³ΠΎΠ½ΠΎΠΌΠ΅Ρ‚Ρ€ΠΈΡ‡Π΅ΡΠΊΡƒΡŽ Ρ„ΠΎΡ€ΠΌΡƒ ΠΈ Ρ„ΠΎΡ€ΠΌΡƒΠ»Ρƒ ΠœΡƒΠ°Π²Ρ€Π° для вычислСния Π·Π½Π°Ρ‡Π΅Π½ΠΈΠΉ:

1) ;

2) ;

3) ;

4) .

13.3: Π”Π»ΠΈΠ½Π° Π΄ΡƒΠ³ΠΈ ΠΈ ΠΊΡ€ΠΈΠ²ΠΈΠ·Π½Π°

Π’ этом Ρ€Π°Π·Π΄Π΅Π»Π΅ ΠΌΡ‹ ΠΈΠ·ΡƒΡ‡Π°Π΅ΠΌ Ρ„ΠΎΡ€ΠΌΡƒΠ»Ρ‹, относящиСся ΠΊ ΠΊΡ€ΠΈΠ²Ρ‹ΠΌ ΠΊΠ°ΠΊ Π² Π΄Π²ΡƒΡ…, Ρ‚Π°ΠΊ ΠΈ Π² Ρ‚Ρ€Π΅Ρ… измСрСниях, ΠΈ Π²ΠΈΠ΄ΠΈΠΌ, ΠΊΠ°ΠΊ ΠΎΠ½ΠΈ связаны с Ρ€Π°Π·Π»ΠΈΡ‡Π½Ρ‹ΠΌΠΈ свойствами ΠΎΠ΄Π½ΠΎΠΉ ΠΈ Ρ‚ΠΎΠΉ ΠΆΠ΅ ΠΊΡ€ΠΈΠ²ΠΎΠΉ. НапримСр, ΠΏΡ€Π΅Π΄ΠΏΠΎΠ»ΠΎΠΆΠΈΠΌ, Ρ‡Ρ‚ΠΎ Π²Π΅ΠΊΡ‚ΠΎΡ€-функция описываСт Π΄Π²ΠΈΠΆΠ΅Π½ΠΈΠ΅ частицы Π² пространствС. ΠœΡ‹ Ρ…ΠΎΡ‚Π΅Π»ΠΈ Π±Ρ‹ ΠΎΠΏΡ€Π΅Π΄Π΅Π»ΠΈΡ‚ΡŒ, ΠΊΠ°ΠΊΠΎΠ΅ расстояниС ΠΏΡ€ΠΎΡˆΠ»Π° частица Π·Π° Π·Π°Π΄Π°Π½Π½Ρ‹ΠΉ ΠΈΠ½Ρ‚Π΅Ρ€Π²Π°Π» Π²Ρ€Π΅ΠΌΠ΅Π½ΠΈ, ΠΊΠΎΡ‚ΠΎΡ€Ρ‹ΠΉ ΠΌΠΎΠΆΠ½ΠΎ ΠΎΠΏΠΈΡΠ°Ρ‚ΡŒ Π΄Π»ΠΈΠ½ΠΎΠΉ Π΄ΡƒΠ³ΠΈ ΠΏΡƒΡ‚ΠΈ, ΠΏΠΎ ΠΊΠΎΡ‚ΠΎΡ€ΠΎΠΌΡƒ ΠΎΠ½Π° слСдуСт. Или ΠΏΡ€Π΅Π΄ΠΏΠΎΠ»ΠΎΠΆΠΈΠΌ, Ρ‡Ρ‚ΠΎ Π²Π΅ΠΊΡ‚ΠΎΡ€-функция описываСт Π΄ΠΎΡ€ΠΎΠ³Ρƒ, ΠΊΠΎΡ‚ΠΎΡ€ΡƒΡŽ ΠΌΡ‹ строим, ΠΈ ΠΌΡ‹ Ρ…ΠΎΡ‚ΠΈΠΌ ΠΎΠΏΡ€Π΅Π΄Π΅Π»ΠΈΡ‚ΡŒ, насколько ΠΊΡ€ΡƒΡ‚ΠΎ изгибаСтся Π΄ΠΎΡ€ΠΎΠ³Π° Π² Π΄Π°Π½Π½ΠΎΠΉ Ρ‚ΠΎΡ‡ΠΊΠ΅.{b}_{a} \|\vecs rβ€²(t)\|dt . \label{Arc3D} \end{align}\]

Π­Ρ‚ΠΈ Π΄Π²Π΅ Ρ„ΠΎΡ€ΠΌΡƒΠ»Ρ‹ ΠΎΡ‡Π΅Π½ΡŒ ΠΏΠΎΡ…ΠΎΠΆΠΈ; ΠΎΠ½ΠΈ ΠΎΡ‚Π»ΠΈΡ‡Π°ΡŽΡ‚ΡΡ Ρ‚ΠΎΠ»ΡŒΠΊΠΎ Ρ‚Π΅ΠΌ, Ρ‡Ρ‚ΠΎ пространствСнная кривая ΠΈΠΌΠ΅Π΅Ρ‚ Ρ‚Ρ€ΠΈ ΡΠΎΡΡ‚Π°Π²Π»ΡΡŽΡ‰ΠΈΠ΅ Ρ„ΡƒΠ½ΠΊΡ†ΠΈΠΈ вмСсто Π΄Π²ΡƒΡ…. ΠžΠ±Ρ€Π°Ρ‚ΠΈΡ‚Π΅ Π²Π½ΠΈΠΌΠ°Π½ΠΈΠ΅, Ρ‡Ρ‚ΠΎ Ρ„ΠΎΡ€ΠΌΡƒΠ»Ρ‹ ΠΎΠΏΡ€Π΅Π΄Π΅Π»Π΅Π½Ρ‹ для Π³Π»Π°Π΄ΠΊΠΈΡ… ΠΊΡ€ΠΈΠ²Ρ‹Ρ…: ΠΊΡ€ΠΈΠ²Ρ‹Ρ…, Π³Π΄Π΅ Π²Π΅ΠΊΡ‚ΠΎΡ€-функция \(\vecs r(t)\) Π΄ΠΈΡ„Ρ„Π΅Ρ€Π΅Π½Ρ†ΠΈΡ€ΡƒΠ΅ΠΌΠ° с Π½Π΅Π½ΡƒΠ»Π΅Π²ΠΎΠΉ ΠΏΡ€ΠΎΠΈΠ·Π²ΠΎΠ΄Π½ΠΎΠΉ. УсловиС гладкости Π³Π°Ρ€Π°Π½Ρ‚ΠΈΡ€ΡƒΠ΅Ρ‚, Ρ‡Ρ‚ΠΎ кривая Π½Π΅ ΠΈΠΌΠ΅Π΅Ρ‚ Ρ‚ΠΎΡ‡Π΅ΠΊ Π²ΠΎΠ·Π²Ρ€Π°Ρ‚Π° (ΠΈΠ»ΠΈ ΡƒΠ³Π»ΠΎΠ²), ΠΊΠΎΡ‚ΠΎΡ€Ρ‹Π΅ ΠΌΠΎΠ³Π»ΠΈ Π±Ρ‹ ΡΠ΄Π΅Π»Π°Ρ‚ΡŒ Ρ„ΠΎΡ€ΠΌΡƒΠ»Ρƒ ΠΏΡ€ΠΎΠ±Π»Π΅ΠΌΠ°Ρ‚ΠΈΡ‡Π½ΠΎΠΉ.

ΠŸΡ€ΠΈΠΌΠ΅Ρ€ \(\PageIndex{1}\): ΠΎΠΏΡ€Π΅Π΄Π΅Π»Π΅Π½ΠΈΠ΅ Π΄Π»ΠΈΠ½Ρ‹ Π΄ΡƒΠ³ΠΈ

ВычислитС Π΄Π»ΠΈΠ½Ρƒ Π΄ΡƒΠ³ΠΈ для ΠΊΠ°ΠΆΠ΄ΠΎΠΉ ΠΈΠ· ΡΠ»Π΅Π΄ΡƒΡŽΡ‰ΠΈΡ… Π²Π΅ΠΊΡ‚ΠΎΡ€-Ρ„ΡƒΠ½ΠΊΡ†ΠΈΠΉ:

  1. \(\vecs r(t)=(3tβˆ’2) \,\hat{\mathbf{i}}+(4t+5) \,\hat{\mathbf{j}},\quad 1≀t ≀5\)
  2. \(\vecs r(t)=⟨t\cos t,t\sin t,2t⟩,0≀t≀2 \pi \)

Раствор

  1. Π˜ΡΠΏΠΎΠ»ΡŒΠ·ΡƒΡ ΡƒΡ€Π°Π²Π½Π΅Π½ΠΈΠ΅ \ref{Arc2D}, \(\vecs rβ€²(t)=3 \,\hat{\mathbf{i}}+4 \,\hat{\mathbf{j}}\), поэтому

    \[\begin{align*} s &=\int^{b}_{a} \|\vecs rβ€²(t)\|dt \\[4pt] &=\int^{5}_{1 } \sqrt{3^2 + 4^2} dt \\[4pt] &=\int^{5}_{1} 5 dt = 5t\big|^{5}_{1} = 20. {3/2})β‰ˆ37,785\) Π΅Π΄ΠΈΠ½ΠΈΡ†

    ВСрнСмся ΠΊ спирали, прСдставлСнной Ρ€Π°Π½Π΅Π΅ Π² этой Π³Π»Π°Π²Π΅. Π’Π΅ΠΊΡ‚ΠΎΡ€-Ρ„ΡƒΠ½ΠΊΡ†ΠΈΡŽ, ΠΎΠΏΠΈΡΡ‹Π²Π°ΡŽΡ‰ΡƒΡŽ ΡΠΏΠΈΡ€Π°Π»ΡŒ, ΠΌΠΎΠΆΠ½ΠΎ Π·Π°ΠΏΠΈΡΠ°Ρ‚ΡŒ Π² Π²ΠΈΠ΄Π΅

    \[\vecs r(t)=R \cos \left(\dfrac{2Ο€Nt}{h}\right) \,\hat{\mathbf{i}} +R \sin \left(\dfrac{2Ο€Nt} {h}\right) \,\hat{\mathbf{j}}+t \,\hat{\mathbf{k}},0≀t≀h, \nonumber\]

    , Π³Π΄Π΅ \(R\) прСдставляСт собой радиус спирали, \(h\) прСдставляСт собой высоту (расстояниС ΠΌΠ΅ΠΆΠ΄Ρƒ двумя ΠΏΠΎΡΠ»Π΅Π΄ΠΎΠ²Π°Ρ‚Π΅Π»ΡŒΠ½Ρ‹ΠΌΠΈ Π²ΠΈΡ‚ΠΊΠ°ΠΌΠΈ), Π° ΡΠΏΠΈΡ€Π°Π»ΡŒ Π·Π°Π²Π΅Ρ€ΡˆΠ°Π΅Ρ‚ \(N\) Π²ΠΈΡ‚ΠΊΠΎΠ².2}.\end{Π²Ρ‹Ρ€Π°Π²Π½ΠΈΠ²Π°Π½ΠΈΠ΅*}\]

    Π­Ρ‚ΠΎ Π΄Π°Π΅Ρ‚ Ρ„ΠΎΡ€ΠΌΡƒΠ»Ρƒ для Π΄Π»ΠΈΠ½Ρ‹ ΠΏΡ€ΠΎΠ²ΠΎΠ»ΠΎΠΊΠΈ, Π½Π΅ΠΎΠ±Ρ…ΠΎΠ΄ΠΈΠΌΠΎΠΉ для формирования спирали с \(N\) Π²ΠΈΡ‚ΠΊΠ°ΠΌΠΈ, которая ΠΈΠΌΠ΅Π΅Ρ‚ радиус \(R\) ΠΈ высоту \(h\).

    ΠŸΠ°Ρ€Π°ΠΌΠ΅Ρ‚Ρ€ΠΈΠ·Π°Ρ†ΠΈΡ Π΄Π»ΠΈΠ½Ρ‹ Π΄ΡƒΠ³ΠΈ

    Π’Π΅ΠΏΠ΅Ρ€ΡŒ Ρƒ нас Π΅ΡΡ‚ΡŒ Ρ„ΠΎΡ€ΠΌΡƒΠ»Π° для Π΄Π»ΠΈΠ½Ρ‹ Π΄ΡƒΠ³ΠΈ ΠΊΡ€ΠΈΠ²ΠΎΠΉ, Π·Π°Π΄Π°Π½Π½ΠΎΠΉ Π²Π΅ΠΊΡ‚ΠΎΡ€-Ρ„ΡƒΠ½ΠΊΡ†ΠΈΠ΅ΠΉ. Π”Π°Π²Π°ΠΉΡ‚Π΅ сдСлаСм Π΅Ρ‰Π΅ ΠΎΠ΄ΠΈΠ½ шаг Π²ΠΏΠ΅Ρ€Π΅Π΄ ΠΈ рассмотрим, Ρ‡Ρ‚ΠΎ Ρ‚Π°ΠΊΠΎΠ΅ функция Π΄Π»ΠΈΠ½Ρ‹ Π΄ΡƒΠ³ΠΈ .

    Если вСкторнозначная функция прСдставляСт ΠΏΠΎΠ»ΠΎΠΆΠ΅Π½ΠΈΠ΅ частицы Π² пространствС ΠΊΠ°ΠΊ Ρ„ΡƒΠ½ΠΊΡ†ΠΈΡŽ Π²Ρ€Π΅ΠΌΠ΅Π½ΠΈ, Ρ‚ΠΎ функция Π΄Π»ΠΈΠ½Ρ‹ Π΄ΡƒΠ³ΠΈ измСряСт, ΠΊΠ°ΠΊ Π΄Π°Π»Π΅ΠΊΠΎ эта частица ΠΏΡ€ΠΎΡ…ΠΎΠ΄ΠΈΡ‚ ΠΊΠ°ΠΊ Ρ„ΡƒΠ½ΠΊΡ†ΠΈΡŽ Π²Ρ€Π΅ΠΌΠ΅Π½ΠΈ. {t}_{ a} β€–\vecs rβ€²(u)β€–du \bigg] \\[4pt] &=\|\vecs rβ€²(t)\|.{t}_{a} β€–\vecs rβ€²(u)β€–\,du\]

    ΠšΡ€ΠΎΠΌΠ΅ Ρ‚ΠΎΠ³ΠΎ,

    \[\dfrac{ds}{dt}=β€–\vecs rβ€²(t)β€–>0. \Π½ΠΎΠΌΠ΅Ρ€\]

    Если \(β€–\vecs rβ€²(t)β€–=1\) для всСх \(tβ‰₯a\), Ρ‚ΠΎ ΠΏΠ°Ρ€Π°ΠΌΠ΅Ρ‚Ρ€ \(t\) прСдставляСт собой Π΄Π»ΠΈΠ½Ρƒ Π΄ΡƒΠ³ΠΈ ΠΎΡ‚ Π½Π°Ρ‡Π°Π»ΡŒΠ½ΠΎΠΉ Ρ‚ΠΎΡ‡ΠΊΠΈ Π² \(t=a\ ).

    ΠŸΠΎΠ»Π΅Π·Π½Ρ‹ΠΌ ΠΏΡ€ΠΈΠΌΠ΅Π½Π΅Π½ΠΈΠ΅ΠΌ этой Ρ‚Π΅ΠΎΡ€Π΅ΠΌΡ‹ являСтся Π½Π°Ρ…ΠΎΠΆΠ΄Π΅Π½ΠΈΠ΅ Π°Π»ΡŒΡ‚Π΅Ρ€Π½Π°Ρ‚ΠΈΠ²Π½ΠΎΠΉ ΠΏΠ°Ρ€Π°ΠΌΠ΅Ρ‚Ρ€ΠΈΠ·Π°Ρ†ΠΈΠΈ Π΄Π°Π½Π½ΠΎΠΉ ΠΊΡ€ΠΈΠ²ΠΎΠΉ, Π½Π°Π·Ρ‹Π²Π°Π΅ΠΌΠΎΠΉ ΠΏΠ°Ρ€Π°ΠΌΠ΅Ρ‚Ρ€ΠΈΠ·Π°Ρ†ΠΈΠ΅ΠΉ Π΄Π»ΠΈΠ½Ρ‹ Π΄ΡƒΠ³ΠΈ . Напомним, Ρ‡Ρ‚ΠΎ Π»ΡŽΠ±ΡƒΡŽ Π²Π΅ΠΊΡ‚ΠΎΡ€-Ρ„ΡƒΠ½ΠΊΡ†ΠΈΡŽ ΠΌΠΎΠΆΠ½ΠΎ ΠΏΠ΅Ρ€Π΅ΠΏΠ°Ρ€Π°ΠΌΠ΅Ρ‚Ρ€ΠΈΡ€ΠΎΠ²Π°Ρ‚ΡŒ Π·Π°ΠΌΠ΅Π½ΠΎΠΉ ΠΏΠ΅Ρ€Π΅ΠΌΠ΅Π½Π½Ρ‹Ρ….НапримСр, Ссли Ρƒ нас Π΅ΡΡ‚ΡŒ функция \(\vecs r(t)=⟨3 \cos t,3 \sin t⟩,0≀t≀2Ο€\), которая ΠΏΠ°Ρ€Π°ΠΌΠ΅Ρ‚Ρ€ΠΈΠ·ΡƒΠ΅Ρ‚ ΠΎΠΊΡ€ΡƒΠΆΠ½ΠΎΡΡ‚ΡŒ радиуса 3, ΠΌΡ‹ ΠΌΠΎΠΆΠ΅ΠΌ ΠΈΠ·ΠΌΠ΅Π½ΠΈΡ‚ΡŒ ΠΏΠ°Ρ€Π°ΠΌΠ΅Ρ‚Ρ€ с \(t\) Π² \(4t\), ΠΏΠΎΠ»ΡƒΡ‡ΠΈΠ² Π½ΠΎΠ²ΡƒΡŽ ΠΏΠ°Ρ€Π°ΠΌΠ΅Ρ‚Ρ€ΠΈΠ·Π°Ρ†ΠΈΡŽ \(\vecs r(t)=⟨3 \cos 4t,3 \sin 4t⟩\). Новая парамСтризация ΠΏΠΎ-ΠΏΡ€Π΅ΠΆΠ½Π΅ΠΌΡƒ опрСдСляСт ΠΎΠΊΡ€ΡƒΠΆΠ½ΠΎΡΡ‚ΡŒ радиуса 3, Π½ΠΎ Ρ‚Π΅ΠΏΠ΅Ρ€ΡŒ Π½Π°ΠΌ Π½ΡƒΠΆΠ½ΠΎ ΠΈΡΠΏΠΎΠ»ΡŒΠ·ΠΎΠ²Π°Ρ‚ΡŒ Ρ‚ΠΎΠ»ΡŒΠΊΠΎ значСния \(0≀t≀π/2\), Ρ‡Ρ‚ΠΎΠ±Ρ‹ ΠΏΡ€ΠΎΠΉΡ‚ΠΈ ΠΊΡ€ΡƒΠ³ ΠΎΠ΄ΠΈΠ½ Ρ€Π°Π·.

    ΠŸΡ€Π΅Π΄ΠΏΠΎΠ»ΠΎΠΆΠΈΠΌ, Ρ‡Ρ‚ΠΎ ΠΌΡ‹ нашли Ρ„ΡƒΠ½ΠΊΡ†ΠΈΡŽ Π΄Π»ΠΈΠ½Ρ‹ Π΄ΡƒΠ³ΠΈ \(s(t)\) ΠΈ ΠΌΠΎΠΆΠ΅ΠΌ Ρ€Π΅ΡˆΠΈΡ‚ΡŒ эту Ρ„ΡƒΠ½ΠΊΡ†ΠΈΡŽ для \(t\) ΠΊΠ°ΠΊ Ρ„ΡƒΠ½ΠΊΡ†ΠΈΡŽ ΠΎΡ‚ \(s\) . Π—Π°Ρ‚Π΅ΠΌ ΠΌΡ‹ ΠΌΠΎΠΆΠ΅ΠΌ ΠΏΠ΅Ρ€Π΅ΠΏΠ°Ρ€Π°ΠΌΠ΅Ρ‚Ρ€ΠΈΡ€ΠΎΠ²Π°Ρ‚ΡŒ ΠΈΡΡ…ΠΎΠ΄Π½ΡƒΡŽ Ρ„ΡƒΠ½ΠΊΡ†ΠΈΡŽ \(\vecs r(t)\), подставив Π²Ρ‹Ρ€Π°ΠΆΠ΅Π½ΠΈΠ΅ для \(t\) ΠΎΠ±Ρ€Π°Ρ‚Π½ΠΎ Π² \(\vecs r(t)\). Π’Π΅ΠΏΠ΅Ρ€ΡŒ Π²Π΅ΠΊΡ‚ΠΎΡ€-функция записываСтся Π² Ρ‚Π΅Ρ€ΠΌΠΈΠ½Π°Ρ… ΠΏΠ°Ρ€Π°ΠΌΠ΅Ρ‚Ρ€Π° \(s\) . ΠŸΠΎΡΠΊΠΎΠ»ΡŒΠΊΡƒ пСрСмСнная \(s\) прСдставляСт Π΄Π»ΠΈΠ½Ρƒ Π΄ΡƒΠ³ΠΈ, ΠΌΡ‹ Π½Π°Π·Ρ‹Π²Π°Π΅ΠΌ это ΠΏΠ°Ρ€Π°ΠΌΠ΅Ρ‚Ρ€ΠΈΠ·Π°Ρ†ΠΈΠ΅ΠΉ Π΄Π»ΠΈΠ½Ρ‹ Π΄ΡƒΠ³ΠΈ исходной Ρ„ΡƒΠ½ΠΊΡ†ΠΈΠΈ \(\vecs r(t)\). Одним ΠΈΠ· прСимущСств нахоТдСния ΠΏΠ°Ρ€Π°ΠΌΠ΅Ρ‚Ρ€ΠΈΠ·Π°Ρ†ΠΈΠΈ Π΄Π»ΠΈΠ½Ρ‹ Π΄ΡƒΠ³ΠΈ являСтся Ρ‚ΠΎ, Ρ‡Ρ‚ΠΎ расстояниС, ΠΏΡ€ΠΎΠΉΠ΄Π΅Π½Π½ΠΎΠ΅ вдоль ΠΊΡ€ΠΈΠ²ΠΎΠΉ, начиная с \(s=0\), Ρ‚Π΅ΠΏΠ΅Ρ€ΡŒ Ρ€Π°Π²Π½ΠΎ ΠΏΠ°Ρ€Π°ΠΌΠ΅Ρ‚Ρ€Ρƒ \(s\).ΠŸΠ°Ρ€Π°ΠΌΠ΅Ρ‚Ρ€ΠΈΠ·Π°Ρ†ΠΈΡ Π΄Π»ΠΈΠ½Ρ‹ Π΄ΡƒΠ³ΠΈ Ρ‚Π°ΠΊΠΆΠ΅ появляСтся Π² контСкстС ΠΊΡ€ΠΈΠ²ΠΈΠ·Π½Ρ‹ (ΠΊΠΎΡ‚ΠΎΡ€ΡƒΡŽ ΠΌΡ‹ рассмотрим ΠΏΠΎΠ·ΠΆΠ΅ Π² этом Ρ€Π°Π·Π΄Π΅Π»Π΅) ΠΈ Π»ΠΈΠ½Π΅ΠΉΠ½Ρ‹Ρ… ΠΈΠ½Ρ‚Π΅Π³Ρ€Π°Π»ΠΎΠ².

    ΠŸΡ€ΠΈΠΌΠ΅Ρ€ \(\PageIndex{2}\): поиск ΠΏΠ°Ρ€Π°ΠΌΠ΅Ρ‚Ρ€ΠΈΠ·Π°Ρ†ΠΈΠΈ Π΄Π»ΠΈΠ½Ρ‹ Π΄ΡƒΠ³ΠΈ

    НайдитС ΠΏΠ°Ρ€Π°ΠΌΠ΅Ρ‚Ρ€ΠΈΠ·Π°Ρ†ΠΈΡŽ Π΄Π»ΠΈΠ½Ρ‹ Π΄ΡƒΠ³ΠΈ для ΠΊΠ°ΠΆΠ΄ΠΎΠΉ ΠΈΠ· ΡΠ»Π΅Π΄ΡƒΡŽΡ‰ΠΈΡ… ΠΊΡ€ΠΈΠ²Ρ‹Ρ…:

    1. \(\vecs r(t)=4 \cos t \,\hat{\mathbf{i}}+ 4 \sin t \,\hat{\mathbf{j}},\quad tβ‰₯0\)
    2. \(\vecs r(t)=⟨t+3,2tβˆ’4,2t⟩,\quad tβ‰₯3\)

    Раствор

    1. Π‘Π½Π°Ρ‡Π°Π»Π° ΠΌΡ‹ Π½Π°Ρ…ΠΎΠ΄ΠΈΠΌ Ρ„ΡƒΠ½ΠΊΡ†ΠΈΡŽ Π΄Π»ΠΈΠ½Ρ‹ Π΄ΡƒΠ³ΠΈ, ΠΈΡΠΏΠΎΠ»ΡŒΠ·ΡƒΡ ΡƒΡ€Π°Π²Π½Π΅Π½ΠΈΠ΅ \ref{arclength3}:

      \[\begin{align*} s(t) &= \int_a^t β€–\vecs rβ€²(u)β€– \,du \\[4pt] &= \int_0^t β€–βŸ¨βˆ’4 \sin u, 4 \cos uβŸ©β€– \,du \\[4pt] &= \int_0^t \sqrt{(βˆ’4 \sin u)^2+(4 \cos u)^2} \,du \\[4pt] &= \int_0^t \sqrt{16 \sin ^2 u+16 \cos ^2 u} \,du \\[4pt] &= \int_0^t 4\,du = 4t, \end{align*} \]

    2. , ΠΊΠΎΡ‚ΠΎΡ€Ρ‹ΠΉ Π΄Π°Π΅Ρ‚ связь ΠΌΠ΅ΠΆΠ΄Ρƒ Π΄Π»ΠΈΠ½ΠΎΠΉ Π΄ΡƒΠ³ΠΈ \(s\) ΠΈ ΠΏΠ°Ρ€Π°ΠΌΠ΅Ρ‚Ρ€ΠΎΠΌ \(t\) ΠΊΠ°ΠΊ \(s=4t;\), Ρ‚ΠΎ Π΅ΡΡ‚ΡŒ \(t=s/4\). Π”Π°Π»Π΅Π΅ замСняСм ΠΏΠ΅Ρ€Π΅ΠΌΠ΅Π½Π½ΡƒΡŽ \(t\) Π² исходной Ρ„ΡƒΠ½ΠΊΡ†ΠΈΠΈ \(\vecs r(t)=4 \cos t \,\hat{\mathbf{i}}+4 \sin t \,\hat{\mathbf {j}}\) с Π²Ρ‹Ρ€Π°ΠΆΠ΅Π½ΠΈΠ΅ΠΌ \(s/4\), Ρ‡Ρ‚ΠΎΠ±Ρ‹ ΠΏΠΎΠ»ΡƒΡ‡ΠΈΡ‚ΡŒ

      \[\vecs r(s)=4 \cos\left(\frac{s}{4}\right) \,\hat{\mathbf{i}} + 4 \sin\left(\frac{s} {4}\справа) \,\шляпа{\mathbf{j}}. \Π½ΠΎΠΌΠ΅Ρ€\]

      Π­Ρ‚ΠΎ парамСтризация Π΄Π»ΠΈΠ½Ρ‹ Π΄ΡƒΠ³ΠΈ \(\vecs r(t)\). ΠŸΠΎΡΠΊΠΎΠ»ΡŒΠΊΡƒ исходноС ΠΎΠ³Ρ€Π°Π½ΠΈΡ‡Π΅Π½ΠΈΠ΅ Π½Π° \(t\) Π±Ρ‹Π»ΠΎ Π·Π°Π΄Π°Π½ΠΎ ΠΊΠ°ΠΊ \(tβ‰₯0\), ΠΎΠ³Ρ€Π°Π½ΠΈΡ‡Π΅Π½ΠΈΠ΅ Π½Π° s становится \(s/4β‰₯0\) ΠΈΠ»ΠΈ \(sβ‰₯0\).t 3 \,du \\[4pt] &= 3t – 9. \end{align*}\]

      Π‘Π»Π΅Π΄ΠΎΠ²Π°Ρ‚Π΅Π»ΡŒΠ½ΠΎ, связь ΠΌΠ΅ΠΆΠ΄Ρƒ Π΄Π»ΠΈΠ½ΠΎΠΉ Π΄ΡƒΠ³ΠΈ \(s\) ΠΈ ΠΏΠ°Ρ€Π°ΠΌΠ΅Ρ‚Ρ€ΠΎΠΌ \(t\) Ρ€Π°Π²Π½Π° \(s=3tβˆ’9\), поэтому \(t= \frac{s}{3}+3\). ΠŸΠΎΠ΄ΡΡ‚Π°Π½ΠΎΠ²ΠΊΠ° этого Π² ΠΈΡΡ…ΠΎΠ΄Π½ΡƒΡŽ Ρ„ΡƒΠ½ΠΊΡ†ΠΈΡŽ \(\vecs r(t)=⟨t+3,2tβˆ’4,2t⟩ \) Π΄Π°Π΅Ρ‚

      \[\vecs r(s)=⟨\left(\frac{s}{3}+3\right)+3,\,2\left(\frac{s}{3}+3\right)βˆ’ 4,\,2\Π²Π»Π΅Π²ΠΎ(\frac{s}{3}+3\Π²ΠΏΡ€Π°Π²ΠΎ)⟩=⟨\frac{s}{3}+6, \frac{2s}{3}+2,\frac{2s {3}+6⟩.\Π½ΠΎΠΌΠ΅Ρ€\]

      Π­Ρ‚ΠΎ парамСтризация Π΄Π»ΠΈΠ½Ρ‹ Π΄ΡƒΠ³ΠΈ \(\vecs r(t)\).Π˜ΡΡ…ΠΎΠ΄Π½ΠΎΠ΅ ΠΎΠ³Ρ€Π°Π½ΠΈΡ‡Π΅Π½ΠΈΠ΅ Π½Π° ΠΏΠ°Ρ€Π°ΠΌΠ΅Ρ‚Ρ€ \(t\) Π±Ρ‹Π»ΠΎ \(tβ‰₯3\), поэтому ΠΎΠ³Ρ€Π°Π½ΠΈΡ‡Π΅Π½ΠΈΠ΅ Π½Π° \(s\) Ρ€Π°Π²Π½ΠΎ \((s/3)+3β‰₯3\) ΠΈΠ»ΠΈ \(sβ‰₯0 \).

    Π£ΠΏΡ€Π°ΠΆΠ½Π΅Π½ΠΈΠ΅ \(\PageIndex{2}\)

    НайдитС Ρ„ΡƒΠ½ΠΊΡ†ΠΈΡŽ Π΄Π»ΠΈΠ½Ρ‹ Π΄ΡƒΠ³ΠΈ для спирали

    \[\vecs r(t)=⟨3 \cos t, 3 \sin t,4t⟩,\quad tβ‰₯0. \Π½ΠΎΠΌΠ΅Ρ€\]

    Π—Π°Ρ‚Π΅ΠΌ ΠΈΡΠΏΠΎΠ»ΡŒΠ·ΡƒΠΉΡ‚Π΅ связь ΠΌΠ΅ΠΆΠ΄Ρƒ Π΄Π»ΠΈΠ½ΠΎΠΉ Π΄ΡƒΠ³ΠΈ ΠΈ ΠΏΠ°Ρ€Π°ΠΌΠ΅Ρ‚Ρ€ΠΎΠΌ \(t\), Ρ‡Ρ‚ΠΎΠ±Ρ‹ Π½Π°ΠΉΡ‚ΠΈ ΠΏΠ°Ρ€Π°ΠΌΠ΅Ρ‚Ρ€ΠΈΠ·Π°Ρ†ΠΈΡŽ Π΄Π»ΠΈΠ½Ρ‹ Π΄ΡƒΠ³ΠΈ для \(\vecs r(t)\).

    Подсказка

    НачнитС с нахоТдСния Ρ„ΡƒΠ½ΠΊΡ†ΠΈΠΈ Π΄Π»ΠΈΠ½Ρ‹ Π΄ΡƒΠ³ΠΈ.

    ΠžΡ‚Π²Π΅Ρ‚ΠΈΡ‚ΡŒ

    \(s=5t\) ΠΈΠ»ΠΈ \(t=s/5\). ΠŸΠΎΠ΄ΡΡ‚Π°Π½ΠΎΠ²ΠΊΠ° этого Π² \(\vecs r(t)=⟨3 \cos t,3 \sin t,4t⟩\) Π΄Π°Π΅Ρ‚

    \[\vecs r(s)=⟨3 \cos\left(\frac{s}{5}\right),3 \sin\left(\frac{s}{5}\right),\frac{ 4s}{5}⟩,\quad sβ‰₯0 \nonumber\]

      ΠšΡ€ΠΈΠ²ΠΈΠ·Π½Π°

      ΠšΡ€ΠΈΠ²ΠΈΠ·Π½Π° являСтся Π²Π°ΠΆΠ½ΠΎΠΉ Ρ‚Π΅ΠΌΠΎΠΉ, связанной с Π΄Π»ΠΈΠ½ΠΎΠΉ Π΄ΡƒΠ³ΠΈ. ΠŸΠΎΠ½ΡΡ‚ΠΈΠ΅ ΠΊΡ€ΠΈΠ²ΠΈΠ·Π½Ρ‹ позволяСт ΠΈΠ·ΠΌΠ΅Ρ€ΠΈΡ‚ΡŒ, насколько Ρ€Π΅Π·ΠΊΠΎ ΠΏΠΎΠ²ΠΎΡ€Π°Ρ‡ΠΈΠ²Π°Π΅Ρ‚ гладкая кривая.ΠžΠΊΡ€ΡƒΠΆΠ½ΠΎΡΡ‚ΡŒ ΠΈΠΌΠ΅Π΅Ρ‚ ΠΏΠΎΡΡ‚ΠΎΡΠ½Π½ΡƒΡŽ ΠΊΡ€ΠΈΠ²ΠΈΠ·Π½Ρƒ. Π§Π΅ΠΌ мСньшС радиус окруТности, Ρ‚Π΅ΠΌ большС ΠΊΡ€ΠΈΠ²ΠΈΠ·Π½Π°.

      ΠŸΡ€Π΅Π΄ΡΡ‚Π°Π²ΡŒΡ‚Π΅, Ρ‡Ρ‚ΠΎ Π²Ρ‹ Π΅Π΄Π΅Ρ‚Π΅ ΠΏΠΎ Π΄ΠΎΡ€ΠΎΠ³Π΅. ΠŸΡ€Π΅Π΄ΠΏΠΎΠ»ΠΎΠΆΠΈΠΌ, Ρ‡Ρ‚ΠΎ Π΄ΠΎΡ€ΠΎΠ³Π° Π»Π΅ΠΆΠΈΡ‚ Π½Π° Π΄ΡƒΠ³Π΅ большого ΠΊΡ€ΡƒΠ³Π°. Π’ этом случаС Π²Π°ΠΌ Π΅Π΄Π²Π° Π»ΠΈ придСтся ΠΊΡ€ΡƒΡ‚ΠΈΡ‚ΡŒ Ρ€ΡƒΠ»ΡŒ, Ρ‡Ρ‚ΠΎΠ±Ρ‹ ΠΎΡΡ‚Π°Π²Π°Ρ‚ΡŒΡΡ Π½Π° Π΄ΠΎΡ€ΠΎΠ³Π΅. Π’Π΅ΠΏΠ΅Ρ€ΡŒ ΠΏΡ€Π΅Π΄ΠΏΠΎΠ»ΠΎΠΆΠΈΠΌ, Ρ‡Ρ‚ΠΎ радиус мСньшС. Π’ этом случаС Π²Π°ΠΌ Π½ΡƒΠΆΠ½ΠΎ Π±ΡƒΠ΄Π΅Ρ‚ ΠΏΠΎΠ²ΠΎΡ€Π°Ρ‡ΠΈΠ²Π°Ρ‚ΡŒ Π±ΠΎΠ»Π΅Π΅ Ρ€Π΅Π·ΠΊΠΎ, Ρ‡Ρ‚ΠΎΠ±Ρ‹ ΠΎΡΡ‚Π°Ρ‚ΡŒΡΡ Π½Π° Π΄ΠΎΡ€ΠΎΠ³Π΅. Π’ случаС ΠΊΡ€ΠΈΠ²ΠΎΠΉ, ΠΎΡ‚Π»ΠΈΡ‡Π½ΠΎΠΉ ΠΎΡ‚ окруТности, часто ΠΏΠΎΠ»Π΅Π·Π½ΠΎ сначала Π²ΠΏΠΈΡΠ°Ρ‚ΡŒ ΠΎΠΊΡ€ΡƒΠΆΠ½ΠΎΡΡ‚ΡŒ Π² ΠΊΡ€ΠΈΠ²ΡƒΡŽ Π² Π΄Π°Π½Π½ΠΎΠΉ Ρ‚ΠΎΡ‡ΠΊΠ΅ Ρ‚Π°ΠΊ, Ρ‡Ρ‚ΠΎΠ±Ρ‹ ΠΎΠ½Π° касалась ΠΊΡ€ΠΈΠ²ΠΎΠΉ Π² этой Ρ‚ΠΎΡ‡ΠΊΠ΅ ΠΈ Β«ΠΎΠ±Π½ΠΈΠΌΠ°Π»Π°Β» ΠΊΡ€ΠΈΠ²ΡƒΡŽ ΠΊΠ°ΠΊ ΠΌΠΎΠΆΠ½ΠΎ тСснСС Π² Π·Π°Π΄Π°Π½Π½ΠΎΠΉ Ρ‚ΠΎΡ‡ΠΊΠ΅. окрСстности Ρ‚ΠΎΡ‡ΠΊΠΈ (рис. \(\PageIndex{1}\)).Π—Π°Ρ‚Π΅ΠΌ ΠΊΡ€ΠΈΠ²ΠΈΠ·Π½Π° Π³Ρ€Π°Ρ„ΠΈΠΊΠ° Π² этой Ρ‚ΠΎΡ‡ΠΊΠ΅ опрСдСляСтся ΠΊΠ°ΠΊ такая ΠΆΠ΅, ΠΊΠ°ΠΊ ΠΊΡ€ΠΈΠ²ΠΈΠ·Π½Π° вписанной окруТности.

      Рисунок \(\PageIndex{1}\): Π“Ρ€Π°Ρ„ΠΈΠΊ прСдставляСт ΠΊΡ€ΠΈΠ²ΠΈΠ·Π½Ρƒ Ρ„ΡƒΠ½ΠΊΡ†ΠΈΠΈ \(y=f(x).\) Π§Π΅ΠΌ ΠΊΡ€ΡƒΡ‡Π΅ ΠΏΠΎΠ²ΠΎΡ€ΠΎΡ‚ Π½Π° Π³Ρ€Π°Ρ„ΠΈΠΊΠ΅, Ρ‚Π΅ΠΌ большС ΠΊΡ€ΠΈΠ²ΠΈΠ·Π½Π° ΠΈ мСньшС радиус вписанный ΠΊΡ€ΡƒΠ³.

      ΠžΠΏΡ€Π΅Π΄Π΅Π»Π΅Π½ΠΈΠ΅: ΠΊΡ€ΠΈΠ²ΠΈΠ·Π½Π°

      ΠŸΡƒΡΡ‚ΡŒ \(C\) β€” гладкая кривая Π½Π° плоскости ΠΈΠ»ΠΈ Π² пространствС, заданная Ρ„ΠΎΡ€ΠΌΡƒΠ»ΠΎΠΉ \(\vecs r(s)\), Π³Π΄Π΅ \(s\) β€” ΠΏΠ°Ρ€Π°ΠΌΠ΅Ρ‚Ρ€ Π΄Π»ΠΈΠ½Ρ‹ Π΄ΡƒΠ³ΠΈ. ΠšΡ€ΠΈΠ²ΠΈΠ·Π½Π° \(ΞΊ\) Π² Ρ‚ΠΎΡ‡ΠΊΠ΅ \(s\) Ρ€Π°Π²Π½Π°

      \[ΞΊ =\bigg{\|}\dfrac{d\vecs{T}}{ds}\bigg{\|}=β€–\vecs Tβ€²(s)β€–. \]

      ΠŸΠΎΡΠΌΠΎΡ‚Ρ€ΠΈΡ‚Π΅ это Π²ΠΈΠ΄Π΅ΠΎ, Ρ‡Ρ‚ΠΎΠ±Ρ‹ ΡƒΠ·Π½Π°Ρ‚ΡŒ большС ΠΎ ΠΊΡ€ΠΈΠ²ΠΈΠ·Π½Π΅ пространствСнной ΠΊΡ€ΠΈΠ²ΠΎΠΉ.

      Π€ΠΎΡ€ΠΌΡƒΠ»Π° опрСдСлСния ΠΊΡ€ΠΈΠ²ΠΈΠ·Π½Ρ‹ Π½Π΅ ΠΎΡ‡Π΅Π½ΡŒ ΠΏΠΎΠ»Π΅Π·Π½Π° с Ρ‚ΠΎΡ‡ΠΊΠΈ зрСния расчСта. Π’ частности, Π½Π°ΠΏΠΎΠΌΠ½ΠΈΠΌ, Ρ‡Ρ‚ΠΎ \(\vecs T(t)\) прСдставляСт собой Π΅Π΄ΠΈΠ½ΠΈΡ‡Π½Ρ‹ΠΉ ΠΊΠ°ΡΠ°Ρ‚Π΅Π»ΡŒΠ½Ρ‹ΠΉ Π²Π΅ΠΊΡ‚ΠΎΡ€ ΠΊ Π·Π°Π΄Π°Π½Π½ΠΎΠΉ Π²Π΅ΠΊΡ‚ΠΎΡ€-Ρ„ΡƒΠ½ΠΊΡ†ΠΈΠΈ \(\vecs r(t)\), Π° Ρ„ΠΎΡ€ΠΌΡƒΠ»Π° для \(\vecs T(t) \) это

      \[\vecs T(t)=\frac{\vecs rβ€²(t)}{βˆ₯\vecs rβ€²(t)βˆ₯}.\]

      Π§Ρ‚ΠΎΠ±Ρ‹ ΠΈΡΠΏΠΎΠ»ΡŒΠ·ΠΎΠ²Π°Ρ‚ΡŒ Ρ„ΠΎΡ€ΠΌΡƒΠ»Ρƒ ΠΊΡ€ΠΈΠ²ΠΈΠ·Π½Ρ‹, сначала Π½Π΅ΠΎΠ±Ρ…ΠΎΠ΄ΠΈΠΌΠΎ Π²Ρ‹Ρ€Π°Π·ΠΈΡ‚ΡŒ \(\vecs r(t)\) Ρ‡Π΅Ρ€Π΅Π· ΠΏΠ°Ρ€Π°ΠΌΠ΅Ρ‚Ρ€ Π΄Π»ΠΈΠ½Ρ‹ Π΄ΡƒΠ³ΠΈ \(s\), Π·Π°Ρ‚Π΅ΠΌ Π½Π°ΠΉΡ‚ΠΈ Π΅Π΄ΠΈΠ½ΠΈΡ‡Π½Ρ‹ΠΉ ΠΊΠ°ΡΠ°Ρ‚Π΅Π»ΡŒΠ½Ρ‹ΠΉ Π²Π΅ΠΊΡ‚ΠΎΡ€ \(\vecs T( s)\) для Ρ„ΡƒΠ½ΠΊΡ†ΠΈΠΈ \(\vecs r(s)\), Ρ‚ΠΎ возьмСм ΠΏΡ€ΠΎΠΈΠ·Π²ΠΎΠ΄Π½ΡƒΡŽ ΠΎΡ‚ \(\vecs T(s)\) ΠΏΠΎ \(s\).Π­Ρ‚ΠΎ ΡƒΡ‚ΠΎΠΌΠΈΡ‚Π΅Π»ΡŒΠ½Ρ‹ΠΉ процСсс. К ΡΡ‡Π°ΡΡ‚ΡŒΡŽ, ΡΡƒΡ‰Π΅ΡΡ‚Π²ΡƒΡŽΡ‚ эквивалСнтныС Ρ„ΠΎΡ€ΠΌΡƒΠ»Ρ‹ для ΠΊΡ€ΠΈΠ²ΠΈΠ·Π½Ρ‹.

      Π’Π΅ΠΎΡ€Π΅ΠΌΠ°: Π°Π»ΡŒΡ‚Π΅Ρ€Π½Π°Ρ‚ΠΈΠ²Π½Ρ‹Π΅ Ρ„ΠΎΡ€ΠΌΡƒΠ»Ρ‹ ΠΊΡ€ΠΈΠ²ΠΈΠ·Π½Ρ‹

      Если \(C\) β€” гладкая кривая, заданная \(\vecs r(t)\), Ρ‚ΠΎ ΠΊΡ€ΠΈΠ²ΠΈΠ·Π½Π° \(ΞΊ\) ΠΊΡ€ΠΈΠ²ΠΎΠΉ \(C\) Π² Ρ‚ΠΎΡ‡ΠΊΠ΅ \(t\) опрСдСляСтся Π²Ρ‹Ρ€Π°ΠΆΠ΅Π½ΠΈΠ΅ΠΌ

      \[ΞΊ =\dfrac{β€–\vecs Tβ€²(t)β€–}{β€–\vecs rβ€²(t)β€–}. \ΠΌΠ΅Ρ‚ΠΊΠ°{EqK2} \]

      Если \(Π‘\) β€” трСхмСрная кривая, Ρ‚ΠΎ ΠΊΡ€ΠΈΠ²ΠΈΠ·Π½Π° ΠΌΠΎΠΆΠ΅Ρ‚ Π±Ρ‹Ρ‚ΡŒ Π·Π°Π΄Π°Π½Π° Ρ„ΠΎΡ€ΠΌΡƒΠ»ΠΎΠΉ

      \[ΞΊ =\dfrac{β€–\vecs rβ€²(t)Γ—\vecs rβ€²β€²(t)β€–}{β€–\vecs rβ€²(t)β€–^3}. {3/2}}.\ΠΌΠ΅Ρ‚ΠΊΠ°{EqK4}\]

      Π”ΠΎΠΊΠ°Π·Π°Ρ‚Π΅Π»ΡŒΡΡ‚Π²ΠΎ

      ΠŸΠ΅Ρ€Π²Π°Ρ Ρ„ΠΎΡ€ΠΌΡƒΠ»Π° слСдуСт нСпосрСдствСнно ΠΈΠ· Ρ†Π΅ΠΏΠ½ΠΎΠ³ΠΎ ΠΏΡ€Π°Π²ΠΈΠ»Π°:

      \[\dfrac{d\vecs{T}}{dt} = \dfrac{d\vecs{T}}{ds} \dfrac{ds}{dt}, \nonumber\]

      , Π³Π΄Π΅ \(s\) – Π΄Π»ΠΈΠ½Π° Π΄ΡƒΠ³ΠΈ вдоль ΠΊΡ€ΠΈΠ²ΠΎΠΉ \(C\). Π Π°Π·Π΄Π΅Π»ΠΈΠ² ΠΎΠ±Π΅ стороны Π½Π° \(ds/dt\) ΠΈ взяв Π²Π΅Π»ΠΈΡ‡ΠΈΠ½Ρƒ ΠΎΠ±Π΅ΠΈΡ… сторон, ΠΌΡ‹ ΠΏΠΎΠ»ΡƒΡ‡ΠΈΠΌ

      .

      \[\bigg{\|}\dfrac{d\vecs{T}}{ds}\bigg{\|}= \left\lVert\frac{\vecs T'(t)}{\dfrac{ds} {dt}}\right\rVert.\nonumber\]

      ΠŸΠΎΡΠΊΠΎΠ»ΡŒΠΊΡƒ \(ds/dt=β€–\vecs rβ€²(t)β€–\), это Π΄Π°Π΅Ρ‚ Ρ„ΠΎΡ€ΠΌΡƒΠ»Ρƒ ΠΊΡ€ΠΈΠ²ΠΈΠ·Π½Ρ‹ \(ΞΊ\) ΠΊΡ€ΠΈΠ²ΠΎΠΉ \(C\) Π² Ρ‚Π΅Ρ€ΠΌΠΈΠ½Π°Ρ… любой ΠΏΠ°Ρ€Π°ΠΌΠ΅Ρ‚Ρ€ΠΈΠ·Π°Ρ†ΠΈΠΈ \(C\ ):

      \[ΞΊ =\dfrac{β€–\vecs Tβ€²(t)β€–}{β€–\vecs rβ€²(t)β€–}.3}.\Π½ΠΎΠΌΠ΅Ρ€\]

      Π­Ρ‚ΠΎ Π΄ΠΎΠΊΠ°Π·Ρ‹Π²Π°Π΅Ρ‚ \(\ref{EqK3}\). Π§Ρ‚ΠΎΠ±Ρ‹ Π΄ΠΎΠΊΠ°Π·Π°Ρ‚ΡŒ \(\ref{EqK4}\), ΠΌΡ‹ Π½Π°Ρ‡Π½Π΅ΠΌ с прСдполоТСния, Ρ‡Ρ‚ΠΎ кривая \(C\) опрСдСляСтся Ρ„ΡƒΠ½ΠΊΡ†ΠΈΠ΅ΠΉ \(y=f(x)\). Π’ΠΎΠ³Π΄Π° ΠΌΡ‹ ΠΌΠΎΠΆΠ΅ΠΌ ΠΎΠΏΡ€Π΅Π΄Π΅Π»ΠΈΡ‚ΡŒ mathbf{k}}\). Π˜ΡΠΏΠΎΠ»ΡŒΠ·ΡƒΡ ΠΏΡ€Π΅Π΄Ρ‹Π΄ΡƒΡ‰ΡƒΡŽ Ρ„ΠΎΡ€ΠΌΡƒΠ»Ρƒ ΠΊΡ€ΠΈΠ²ΠΈΠ·Π½Ρ‹:

      \[\begin{align*} \vecs r'(t) &=\,\hat{\mathbf{i}}+f'(x)\,\hat{\mathbf{j}} \\[4pt ] \vecs rβ€³(t) &=fβ€³(x)\,\hat{\mathbf{j}} \\[4pt] \vecs rβ€²(t)Γ—\vecs rβ€³(t) &= \begin {vmatrix} \hat{\mathbf{i}} & \hat{\mathbf{j}} & \hat{\mathbf{k}} \\ 1 & f'(x) & 0 \\ 0 & f”( Ρ…) & 0 \end{vmatrix} = fβ€³(x)\,\hat{\mathbf{k}}. {3/2}}β‰ˆ0,0059\)

        ΠΠΎΡ€ΠΌΠ°Π»ΡŒΠ½Ρ‹Π΅ ΠΈ Π±ΠΈΠ½ΠΎΡ€ΠΌΠ°Π»ΡŒΠ½Ρ‹Π΅ Π²Π΅ΠΊΡ‚ΠΎΡ€Ρ‹

        ΠœΡ‹ Π²ΠΈΠ΄Π΅Π»ΠΈ, Ρ‡Ρ‚ΠΎ производная \(\vecs rβ€²(t)\) Π²Π΅ΠΊΡ‚ΠΎΡ€-Ρ„ΡƒΠ½ΠΊΡ†ΠΈΠΈ являСтся ΠΊΠ°ΡΠ°Ρ‚Π΅Π»ΡŒΠ½Ρ‹ΠΌ Π²Π΅ΠΊΡ‚ΠΎΡ€ΠΎΠΌ ΠΊ ΠΊΡ€ΠΈΠ²ΠΎΠΉ, Π·Π°Π΄Π°Π½Π½ΠΎΠΉ \(\vecs r(t)\), Π° Π΅Π΄ΠΈΠ½ΠΈΡ‡Π½Ρ‹ΠΉ ΠΊΠ°ΡΠ°Ρ‚Π΅Π»ΡŒΠ½Ρ‹ΠΉ Π²Π΅ΠΊΡ‚ΠΎΡ€ \ (\vecs T(t)\) ΠΌΠΎΠΆΠ½ΠΎ Ρ€Π°ΡΡΡ‡ΠΈΡ‚Π°Ρ‚ΡŒ, Ρ€Π°Π·Π΄Π΅Π»ΠΈΠ² \(\vecs rβ€²(t)\) Π½Π° Π΅Π³ΠΎ Π²Π΅Π»ΠΈΡ‡ΠΈΠ½Ρƒ. ΠŸΡ€ΠΈ ΠΈΠ·ΡƒΡ‡Π΅Π½ΠΈΠΈ двиТСния Π² Ρ‚Ρ€Π΅Ρ… измСрСниях для описания двиТСния частицы вдоль Ρ‚Ρ€Π°Π΅ΠΊΡ‚ΠΎΡ€ΠΈΠΈ Π² пространствС ΠΏΠΎΠ»Π΅Π·Π½Ρ‹ Π΄Π²Π° Π΄Ρ€ΡƒΠ³ΠΈΡ… Π²Π΅ΠΊΡ‚ΠΎΡ€Π°: Π²Π΅ΠΊΡ‚ΠΎΡ€ Π³Π»Π°Π²Π½ΠΎΠΉ Π΅Π΄ΠΈΠ½ΠΈΡ‡Π½ΠΎΠΉ Π½ΠΎΡ€ΠΌΠ°Π»ΠΈ ΠΈ Π±ΠΈΠ½ΠΎΡ€ΠΌΠ°Π»ΡŒΠ½Ρ‹ΠΉ Π²Π΅ΠΊΡ‚ΠΎΡ€ .

        ΠžΠΏΡ€Π΅Π΄Π΅Π»Π΅Π½ΠΈΠ΅: Π±ΠΈΠ½ΠΎΡ€ΠΌΠ°Π»ΡŒΠ½Ρ‹Π΅ Π²Π΅ΠΊΡ‚ΠΎΡ€Ρ‹

        ΠŸΡƒΡΡ‚ΡŒ \(C\) трСхмСрная гладкая кривая, прСдставлСнная \(\vecs r\) Π½Π° ΠΎΡ‚ΠΊΡ€Ρ‹Ρ‚ΠΎΠΌ ΠΈΠ½Ρ‚Π΅Ρ€Π²Π°Π»Π΅ \(I\) . Если \(\vecs Tβ€²(t)β‰ \vecs 0\), Ρ‚ΠΎ Π³Π»Π°Π²Π½Ρ‹ΠΉ Π΅Π΄ΠΈΠ½ΠΈΡ‡Π½Ρ‹ΠΉ Π²Π΅ΠΊΡ‚ΠΎΡ€ Π½ΠΎΡ€ΠΌΠ°Π»ΠΈ Π² Ρ‚ΠΎΡ‡ΠΊΠ΅ \(t\) опрСдСляСтся ΠΊΠ°ΠΊ

        \[\vecs N(t)=\dfrac{\vecs Tβ€²(t)}{β€–\vecs Tβ€²(t)β€–}. \ΠΌΠ΅Ρ‚ΠΊΠ°{EqNormal}\]

        Π’Π΅ΠΊΡ‚ΠΎΡ€ Π±ΠΈΠ½ΠΎΡ€ΠΌΠ°Π»Π΅ΠΉ Π² Ρ‚ΠΎΡ‡ΠΊΠ΅ \(t\) опрСдСляСтся ΠΊΠ°ΠΊ

        \[\vecs B(t)=\vecs T(t)Γ—\vecs N(t),\label{EqBinormal}\]

        , Π³Π΄Π΅ \(\vecs T(t)\) – Π΅Π΄ΠΈΠ½ΠΈΡ‡Π½Ρ‹ΠΉ ΠΊΠ°ΡΠ°Ρ‚Π΅Π»ΡŒΠ½Ρ‹ΠΉ Π²Π΅ΠΊΡ‚ΠΎΡ€.

        ΠžΠ±Ρ€Π°Ρ‚ΠΈΡ‚Π΅ Π²Π½ΠΈΠΌΠ°Π½ΠΈΠ΅, Ρ‡Ρ‚ΠΎ Π±ΠΈΠ½ΠΎΡ€ΠΌΠ°Π»ΠΈ ΠΏΠΎ ΠΎΠΏΡ€Π΅Π΄Π΅Π»Π΅Π½ΠΈΡŽ ΠΎΡ€Ρ‚ΠΎΠ³ΠΎΠ½Π°Π»ΡŒΠ½Ρ‹ ΠΊΠ°ΠΊ Π΅Π΄ΠΈΠ½ΠΈΡ‡Π½ΠΎΠΌΡƒ ΠΊΠ°ΡΠ°Ρ‚Π΅Π»ΡŒΠ½ΠΎΠΌΡƒ Π²Π΅ΠΊΡ‚ΠΎΡ€Ρƒ, Ρ‚Π°ΠΊ ΠΈ Π²Π΅ΠΊΡ‚ΠΎΡ€Ρƒ Π½ΠΎΡ€ΠΌΠ°Π»ΠΈ. ΠšΡ€ΠΎΠΌΠ΅ Ρ‚ΠΎΠ³ΠΎ, \(\vecs B(t)\) всСгда являСтся Π΅Π΄ΠΈΠ½ΠΈΡ‡Π½Ρ‹ΠΌ Π²Π΅ΠΊΡ‚ΠΎΡ€ΠΎΠΌ. Π­Ρ‚ΠΎ ΠΌΠΎΠΆΠ½ΠΎ ΠΏΠΎΠΊΠ°Π·Π°Ρ‚ΡŒ, ΠΈΡΠΏΠΎΠ»ΡŒΠ·ΡƒΡ Ρ„ΠΎΡ€ΠΌΡƒΠ»Ρƒ для Π²Π΅Π»ΠΈΡ‡ΠΈΠ½Ρ‹ Π²Π΅ΠΊΡ‚ΠΎΡ€Π½ΠΎΠ³ΠΎ произвСдСния.

        \[β€–\vecs B(t)β€–=β€–\vecs T(t)Γ—\vecs N(t)β€–=β€–\vecs T(t)β€–β€–\vecs N(t)β€– \sin \theta,\ ]

        , Π³Π΄Π΅ \(\theta\) β€” ΡƒΠ³ΠΎΠ» ΠΌΠ΅ΠΆΠ΄Ρƒ \(\vecs T(t)\) ΠΈ \(\vecs N(t)\). ΠŸΠΎΡΠΊΠΎΠ»ΡŒΠΊΡƒ \(\vecs N(t)\) являСтся ΠΏΡ€ΠΎΠΈΠ·Π²ΠΎΠ΄Π½ΠΎΠΉ Π΅Π΄ΠΈΠ½ΠΈΡ‡Π½ΠΎΠ³ΠΎ Π²Π΅ΠΊΡ‚ΠΎΡ€Π°, свойство (vii) ΠΏΡ€ΠΎΠΈΠ·Π²ΠΎΠ΄Π½ΠΎΠΉ Π²Π΅ΠΊΡ‚ΠΎΡ€-Ρ„ΡƒΠ½ΠΊΡ†ΠΈΠΈ Π³ΠΎΠ²ΠΎΡ€ΠΈΡ‚ Π½Π°ΠΌ, Ρ‡Ρ‚ΠΎ \(\vecs T(t)\) ΠΈ \(\vecs N(t)\) ΠΎΡ€Ρ‚ΠΎΠ³ΠΎΠ½Π°Π»ΡŒΠ½Ρ‹ Π΄Ρ€ΡƒΠ³ Π΄Ρ€ΡƒΠ³Ρƒ, поэтому \(\theta=Ο€/2\).ΠšΡ€ΠΎΠΌΠ΅ Ρ‚ΠΎΠ³ΠΎ, ΠΎΠ½ΠΈ ΠΎΠ±Π° ΡΠ²Π»ΡΡŽΡ‚ΡΡ Π΅Π΄ΠΈΠ½ΠΈΡ‡Π½Ρ‹ΠΌΠΈ Π²Π΅ΠΊΡ‚ΠΎΡ€Π°ΠΌΠΈ, поэтому ΠΈΡ… Π²Π΅Π»ΠΈΡ‡ΠΈΠ½Π° Ρ€Π°Π²Π½Π° 1. Π‘Π»Π΅Π΄ΠΎΠ²Π°Ρ‚Π΅Π»ΡŒΠ½ΠΎ, \(β€–\vecs T(t)β€–β€–\vecs N(t)β€– \sin \theta=(1)(1) \sin (Ο€/ 2)=1\) ΠΈ \(\vecs B(t)\) – Π΅Π΄ΠΈΠ½ΠΈΡ‡Π½Ρ‹ΠΉ Π²Π΅ΠΊΡ‚ΠΎΡ€.

        ВычислСниС Π³Π»Π°Π²Π½ΠΎΠ³ΠΎ Π΅Π΄ΠΈΠ½ΠΈΡ‡Π½ΠΎΠ³ΠΎ Π²Π΅ΠΊΡ‚ΠΎΡ€Π° Π½ΠΎΡ€ΠΌΠ°Π»ΠΈ ΠΌΠΎΠΆΠ΅Ρ‚ ΠΎΠΊΠ°Π·Π°Ρ‚ΡŒΡΡ слоТной Π·Π°Π΄Π°Ρ‡Π΅ΠΉ, ΠΏΠΎΡΠΊΠΎΠ»ΡŒΠΊΡƒ Π΅Π΄ΠΈΠ½ΠΈΡ‡Π½Ρ‹ΠΉ Π²Π΅ΠΊΡ‚ΠΎΡ€ ΠΊΠ°ΡΠ°Ρ‚Π΅Π»ΡŒΠ½ΠΎΠΉ Π²ΠΊΠ»ΡŽΡ‡Π°Π΅Ρ‚ частноС, Π° это частноС часто ΠΈΠΌΠ΅Π΅Ρ‚ ΠΊΠ²Π°Π΄Ρ€Π°Ρ‚Π½Ρ‹ΠΉ ΠΊΠΎΡ€Π΅Π½ΡŒ Π² Π·Π½Π°ΠΌΠ΅Π½Π°Ρ‚Π΅Π»Π΅. 2\,\hat{\mathbf{j}}βˆ’8t\,\hat{\mathbf {ΠΊ}}\)

      Раствор

      1. Π­Ρ‚Π° функция описываСт ΠΎΠΊΡ€ΡƒΠΆΠ½ΠΎΡΡ‚ΡŒ.2 t}} \\[4pt]
        &=βˆ’ \cos t\,\hat{\mathbf{i}}+ \sin t\,\hat{\mathbf{j}}. \ΠΊΠΎΠ½Π΅Ρ†{Π²Ρ‹Ρ€Π°Π²Π½ΠΈΠ²Π°Π½ΠΈΠ΅*}\]

        ΠžΠ±Ρ€Π°Ρ‚ΠΈΡ‚Π΅ Π²Π½ΠΈΠΌΠ°Π½ΠΈΠ΅, Ρ‡Ρ‚ΠΎ Π΅Π΄ΠΈΠ½ΠΈΡ‡Π½Ρ‹ΠΉ ΠΊΠ°ΡΠ°Ρ‚Π΅Π»ΡŒΠ½Ρ‹ΠΉ Π²Π΅ΠΊΡ‚ΠΎΡ€ ΠΈ Π³Π»Π°Π²Π½Ρ‹ΠΉ Π΅Π΄ΠΈΠ½ΠΈΡ‡Π½Ρ‹ΠΉ Π²Π΅ΠΊΡ‚ΠΎΡ€ Π½ΠΎΡ€ΠΌΠ°Π»ΠΈ ΠΎΡ€Ρ‚ΠΎΠ³ΠΎΠ½Π°Π»ΡŒΠ½Ρ‹ Π΄Ρ€ΡƒΠ³ Π΄Ρ€ΡƒΠ³Ρƒ для всСх Π·Π½Π°Ρ‡Π΅Π½ΠΈΠΉ \(t\):

        \[\begin{align*} \vecs T(t)Β·\vecs N(t) &=βŸ¨βˆ’ \sin t,βˆ’ \cos tβŸ©Β·βŸ¨βˆ’ \cos t, \sin t⟩ \\[4pt ] &= \sin t \cos tβˆ’\cos t \sin t \\[4pt] &=0. \ΠΊΠΎΠ½Π΅Ρ†{Π²Ρ‹Ρ€Π°Π²Π½ΠΈΠ²Π°Π½ΠΈΠ΅*}\]

        ΠšΡ€ΠΎΠΌΠ΅ Ρ‚ΠΎΠ³ΠΎ, Π³Π»Π°Π²Π½Ρ‹ΠΉ Π΅Π΄ΠΈΠ½ΠΈΡ‡Π½Ρ‹ΠΉ Π²Π΅ΠΊΡ‚ΠΎΡ€ Π½ΠΎΡ€ΠΌΠ°Π»ΠΈ ΡƒΠΊΠ°Π·Ρ‹Π²Π°Π΅Ρ‚ ΠΊ Ρ†Π΅Π½Ρ‚Ρ€Ρƒ окруТности ΠΈΠ· ΠΊΠ°ΠΆΠ΄ΠΎΠΉ Ρ‚ΠΎΡ‡ΠΊΠΈ окруТности.2βˆ’3t)\,\hat{\mathbf{i}}+(4t+1)\,\hat{\mathbf{j}}\) ΠΈ Π²Ρ‹Ρ‡ΠΈΡΠ»ΠΈΡ‚ΡŒ Π΅Π³ΠΎ ΠΏΡ€ΠΈ \(t=2\).

        Подсказка

        Π‘Π½Π°Ρ‡Π°Π»Π° Π½Π°ΠΉΠ΄ΠΈΡ‚Π΅ \(\vecs T(t)\), Π·Π°Ρ‚Π΅ΠΌ ΠΈΡΠΏΠΎΠ»ΡŒΠ·ΡƒΠΉΡ‚Π΅ \(\ref{EqNormal}\).

        ΠžΡ‚Π²Π΅Ρ‚ΠΈΡ‚ΡŒ

        \(\vecs N(2)=\dfrac{\sqrt{2}}{2}(\,\hat{\mathbf{i}}βˆ’\,\hat{\mathbf{j}})\)

        Для любой Π³Π»Π°Π΄ΠΊΠΎΠΉ ΠΊΡ€ΠΈΠ²ΠΎΠΉ Π² Ρ‚Ρ€Π΅Ρ… измСрСниях, Π·Π°Π΄Π°Π½Π½ΠΎΠΉ Π²Π΅ΠΊΡ‚ΠΎΡ€-Ρ„ΡƒΠ½ΠΊΡ†ΠΈΠ΅ΠΉ, Ρ‚Π΅ΠΏΠ΅Ρ€ΡŒ Ρƒ нас Π΅ΡΡ‚ΡŒ Ρ„ΠΎΡ€ΠΌΡƒΠ»Ρ‹ для Π΅Π΄ΠΈΠ½ΠΈΡ‡Π½ΠΎΠ³ΠΎ ΠΊΠ°ΡΠ°Ρ‚Π΅Π»ΡŒΠ½ΠΎΠ³ΠΎ Π²Π΅ΠΊΡ‚ΠΎΡ€Π° \(\vecs T\), Π΅Π΄ΠΈΠ½ΠΈΡ‡Π½ΠΎΠ³ΠΎ Π²Π΅ΠΊΡ‚ΠΎΡ€Π° Π½ΠΎΡ€ΠΌΠ°Π»ΠΈ \(\vecs N\) ΠΈ Π±ΠΈΠ½ΠΎΡ€ΠΌΠ°Π»ΠΈ Π²Π΅ΠΊΡ‚ΠΎΡ€ \(\vecs B\). Π•Π΄ΠΈΠ½ΠΈΡ‡Π½Ρ‹ΠΉ Π²Π΅ΠΊΡ‚ΠΎΡ€ Π½ΠΎΡ€ΠΌΠ°Π»ΠΈ ΠΈ Π²Π΅ΠΊΡ‚ΠΎΡ€ Π±ΠΈΠ½ΠΎΡ€ΠΌΠ°Π»ΠΈ ΠΎΠ±Ρ€Π°Π·ΡƒΡŽΡ‚ ΠΏΠ»ΠΎΡΠΊΠΎΡΡ‚ΡŒ, ΠΏΠ΅Ρ€ΠΏΠ΅Π½Π΄ΠΈΠΊΡƒΠ»ΡΡ€Π½ΡƒΡŽ ΠΊΡ€ΠΈΠ²ΠΎΠΉ Π² любой Ρ‚ΠΎΡ‡ΠΊΠ΅ ΠΊΡ€ΠΈΠ²ΠΎΠΉ, Π½Π°Π·Ρ‹Π²Π°Π΅ΠΌΡƒΡŽ ΠΏΠ»ΠΎΡΠΊΠΎΡΡ‚ΡŒΡŽ Π½ΠΎΡ€ΠΌΠ°Π»ΠΈ. ΠšΡ€ΠΎΠΌΠ΅ Ρ‚ΠΎΠ³ΠΎ, эти Ρ‚Ρ€ΠΈ Π²Π΅ΠΊΡ‚ΠΎΡ€Π° ΠΎΠ±Ρ€Π°Π·ΡƒΡŽΡ‚ систСму отсчСта Π² Ρ‚Ρ€Π΅Ρ…ΠΌΠ΅Ρ€Π½ΠΎΠΌ пространствС, Π½Π°Π·Ρ‹Π²Π°Π΅ΠΌΡƒΡŽ систСмой отсчСта Π€Ρ€Π΅Π½Π΅ (Ρ‚Π°ΠΊΠΆΠ΅ Π½Π°Π·Ρ‹Π²Π°Π΅ΠΌΠΎΠΉ систСмой отсчСта TNB ) (рис. \(\PageIndex{2}\)). НаконСц, ΠΏΠ»ΠΎΡΠΊΠΎΡΡ‚ΡŒ, опрСдСляСмая Π²Π΅ΠΊΡ‚ΠΎΡ€Π°ΠΌΠΈ \(\vecs T\) ΠΈ \(\vecs N\), ΠΎΠ±Ρ€Π°Π·ΡƒΠ΅Ρ‚ ΡΠΎΠΏΡ€ΠΈΠΊΠ°ΡΠ°ΡŽΡ‰ΡƒΡŽΡΡ ΠΏΠ»ΠΎΡΠΊΠΎΡΡ‚ΡŒ \(C\) Π² любой Ρ‚ΠΎΡ‡ΠΊΠ΅ \(P\) Π½Π° ΠΊΡ€ΠΈΠ²ΠΎΠΉ.

        Рисунок \(\PageIndex{2}\): На этом рисункС ΠΏΠΎΠΊΠ°Π·Π°Π½Π° систСма отсчСта Π€Ρ€Π΅Π½Π΅.Π’ ΠΊΠ°ΠΆΠ΄ΠΎΠΉ Ρ‚ΠΎΡ‡ΠΊΠ΅ \(P\) Π½Π° Ρ‚Ρ€Π΅Ρ…ΠΌΠ΅Ρ€Π½ΠΎΠΉ ΠΊΡ€ΠΈΠ²ΠΎΠΉ Π²Π΅ΠΊΡ‚ΠΎΡ€Π° Π΅Π΄ΠΈΠ½ΠΈΡ‡Π½ΠΎΠΉ ΠΊΠ°ΡΠ°Ρ‚Π΅Π»ΡŒΠ½ΠΎΠΉ, Π΅Π΄ΠΈΠ½ΠΈΡ‡Π½ΠΎΠΉ Π½ΠΎΡ€ΠΌΠ°Π»ΠΈ ΠΈ Π±ΠΈΠ½ΠΎΡ€ΠΌΠ°Π»ΠΈ ΠΎΠ±Ρ€Π°Π·ΡƒΡŽΡ‚ Ρ‚Ρ€Π΅Ρ…ΠΌΠ΅Ρ€Π½ΡƒΡŽ систСму отсчСта.

        ΠŸΡ€Π΅Π΄ΠΏΠΎΠ»ΠΎΠΆΠΈΠΌ, ΠΌΡ‹ ΠΎΠ±Ρ€Π°Π·ΡƒΠ΅ΠΌ ΠΎΠΊΡ€ΡƒΠΆΠ½ΠΎΡΡ‚ΡŒ Π² ΡΠΎΠΏΡ€ΠΈΠΊΠ°ΡΠ°ΡŽΡ‰Π΅ΠΉΡΡ плоскости \(C\) Π² Ρ‚ΠΎΡ‡ΠΊΠ΅ \(P\) Π½Π° ΠΊΡ€ΠΈΠ²ΠΎΠΉ. ΠŸΡ€Π΅Π΄ΠΏΠΎΠ»ΠΎΠΆΠΈΠΌ, Ρ‡Ρ‚ΠΎ ΠΎΠΊΡ€ΡƒΠΆΠ½ΠΎΡΡ‚ΡŒ ΠΈΠΌΠ΅Π΅Ρ‚ Ρ‚Ρƒ ΠΆΠ΅ ΠΊΡ€ΠΈΠ²ΠΈΠ·Π½Ρƒ, Ρ‡Ρ‚ΠΎ ΠΈ кривая Π² Ρ‚ΠΎΡ‡ΠΊΠ΅ \(P\), ΠΈ ΠΏΡƒΡΡ‚ΡŒ ΠΎΠΊΡ€ΡƒΠΆΠ½ΠΎΡΡ‚ΡŒ ΠΈΠΌΠ΅Π΅Ρ‚ радиус \(r\). Π’ΠΎΠ³Π΄Π° ΠΊΡ€ΠΈΠ²ΠΈΠ·Π½Π° ΠΊΡ€ΡƒΠ³Π° опрСдСляСтся ΠΊΠ°ΠΊ \(\frac{1}{r}\). ΠœΡ‹ Π½Π°Π·Ρ‹Π²Π°Π΅ΠΌ \(r\) радиусом ΠΊΡ€ΠΈΠ²ΠΈΠ·Π½Ρ‹ ΠΊΡ€ΠΈΠ²ΠΎΠΉ, ΠΈ ΠΎΠ½ Ρ€Π°Π²Π΅Π½ ΠΎΠ±Ρ€Π°Ρ‚Π½ΠΎΠΉ Π²Π΅Π»ΠΈΡ‡ΠΈΠ½Π΅ ΠΊΡ€ΠΈΠ²ΠΈΠ·Π½Ρ‹.Если эта ΠΎΠΊΡ€ΡƒΠΆΠ½ΠΎΡΡ‚ΡŒ Π»Π΅ΠΆΠΈΡ‚ Π½Π° Π²ΠΎΠ³Π½ΡƒΡ‚ΠΎΠΉ сторонС ΠΊΡ€ΠΈΠ²ΠΎΠΉ ΠΈ касаСтся ΠΊΡ€ΠΈΠ²ΠΎΠΉ Π² Ρ‚ΠΎΡ‡ΠΊΠ΅ \(P\), Ρ‚ΠΎ эта ΠΎΠΊΡ€ΡƒΠΆΠ½ΠΎΡΡ‚ΡŒ называСтся ΡΠΎΠΏΡ€ΠΈΠΊΠ°ΡΠ°ΡŽΡ‰Π΅ΠΉΡΡ ΠΎΠΊΡ€ΡƒΠΆΠ½ΠΎΡΡ‚ΡŒΡŽ ΠΊΡ€ΠΈΠ²ΠΎΠΉ \(C\) Π² \(P\), ΠΊΠ°ΠΊ ΠΏΠΎΠΊΠ°Π·Π°Π½ΠΎ Π½Π° рисункС \(\PageIndex{3}\).

        Рисунок \(\PageIndex{3}\): Π’ этой ΡΠΎΠΏΡ€ΠΈΠΊΠ°ΡΠ°ΡŽΡ‰Π΅ΠΉΡΡ окруТности ΠΎΠΊΡ€ΡƒΠΆΠ½ΠΎΡΡ‚ΡŒ касаСтся ΠΊΡ€ΠΈΠ²ΠΎΠΉ \(C\) Π² Ρ‚ΠΎΡ‡ΠΊΠ΅ \(P\) ΠΈ ΠΈΠΌΠ΅Π΅Ρ‚ Ρ‚Ρƒ ΠΆΠ΅ ΠΊΡ€ΠΈΠ²ΠΈΠ·Π½Ρƒ.

        Для получСния Π΄ΠΎΠΏΠΎΠ»Π½ΠΈΡ‚Π΅Π»ΡŒΠ½ΠΎΠΉ ΠΈΠ½Ρ„ΠΎΡ€ΠΌΠ°Ρ†ΠΈΠΈ ΠΎ ΡΠΎΠΏΡ€ΠΈΠΊΠ°ΡΠ°ΡŽΡ‰ΠΈΡ…ΡΡ окруТностях см. эту Π΄Π΅ΠΌΠΎΠ½ΡΡ‚Ρ€Π°Ρ†ΠΈΡŽ ΠΊΡ€ΠΈΠ²ΠΈΠ·Π½Ρ‹ ΠΈ кручСния, эту ΡΡ‚Π°Ρ‚ΡŒΡŽ ΠΎ ΡΠΎΠΏΡ€ΠΈΠΊΠ°ΡΠ°ΡŽΡ‰ΠΈΡ…ΡΡ окруТностях ΠΈ это обсуТдСниС Ρ„ΠΎΡ€ΠΌΡƒΠ» Π‘Π΅Ρ€Ρ€Π΅.{3/2}}.\]

        Π­Ρ‚ΠΎ Π΄Π°Π΅Ρ‚ \(ΞΊ=6\). Π‘Π»Π΅Π΄ΠΎΠ²Π°Ρ‚Π΅Π»ΡŒΠ½ΠΎ, радиус ΡΠΎΠΏΡ€ΠΈΠΊΠ°ΡΠ°ΡŽΡ‰Π΅ΠΉΡΡ окруТности опрСдСляСтся Π²Ρ‹Ρ€Π°ΠΆΠ΅Π½ΠΈΠ΅ΠΌ \(R=\frac{1}{ΞΊ}=\dfrac{1}{6}\). Π”Π°Π»Π΅Π΅ ΠΌΡ‹ вычисляСм ΠΊΠΎΠΎΡ€Π΄ΠΈΠ½Π°Ρ‚Ρ‹ Ρ†Π΅Π½Ρ‚Ρ€Π° ΠΊΡ€ΡƒΠ³Π°. Когда \(x=1\), Π½Π°ΠΊΠ»ΠΎΠ½ ΠΊΠ°ΡΠ°Ρ‚Π΅Π»ΡŒΠ½ΠΎΠΉ Ρ€Π°Π²Π΅Π½ Π½ΡƒΠ»ΡŽ. Π‘Π»Π΅Π΄ΠΎΠ²Π°Ρ‚Π΅Π»ΡŒΠ½ΠΎ, Ρ†Π΅Π½Ρ‚Ρ€ ΡΠΎΠΏΡ€ΠΈΠΊΠ°ΡΠ°ΡŽΡ‰Π΅ΠΉΡΡ окруТности находится прямо Π½Π°Π΄ Ρ‚ΠΎΡ‡ΠΊΠΎΠΉ Π½Π° Π³Ρ€Π°Ρ„ΠΈΠΊΠ΅ с ΠΊΠΎΠΎΡ€Π΄ΠΈΠ½Π°Ρ‚Π°ΠΌΠΈ \((1,βˆ’1)\). Π¦Π΅Π½Ρ‚Ρ€ располоТСн Π² Ρ‚ΠΎΡ‡ΠΊΠ΅ \((1,βˆ’\frac{5}{6})\). 2=\frac{1}{16}\).

        16.6 Π’Π΅ΠΊΡ‚ΠΎΡ€Π½Ρ‹Π΅ Ρ„ΡƒΠ½ΠΊΡ†ΠΈΠΈ для повСрхностСй

        ΠœΡ‹ ΠΌΠ½ΠΎΠ³ΠΎ Ρ€Π°Π±ΠΎΡ‚Π°Π»ΠΈ с Π²Π΅ΠΊΡ‚ΠΎΡ€Π½Ρ‹ΠΌΠΈ уравнСниями для ΠΊΡ€ΠΈΠ²Ρ‹Ρ…, ${\bf r}(t)=\langle x(t),y(t),z(t)\rangle$. Аналогичная Ρ‚Π΅Ρ…Π½ΠΈΠΊΠ° ΠΌΠΎΠΆΠ΅Ρ‚ ΠΈΡΠΏΠΎΠ»ΡŒΠ·ΠΎΠ²Π°Ρ‚ΡŒΡΡ для прСдставлСния повСрхностСй Π² Π±ΠΎΠ»Π΅Π΅ ΠΎΠ±Ρ‰Π΅ΠΌ Π²ΠΈΠ΄Π΅, Ρ‡Π΅ΠΌ уравнСния для повСрхностСй, ΠΊΠΎΡ‚ΠΎΡ€Ρ‹Π΅ ΠΌΡ‹ использовали Π΄ΠΎ сих ΠΏΠΎΡ€. Напомним, Ρ‡Ρ‚ΠΎ ΠΊΠΎΠ³Π΄Π° ΠΌΡ‹ ΠΈΡΠΏΠΎΠ»ΡŒΠ·ΡƒΠ΅ΠΌ ${\bf r}(t)$ для прСдставлСния ΠΊΡ€ΠΈΠ²ΠΎΠΉ, ΠΌΡ‹ прСдставляСм сСбС Π²Π΅ΠΊΡ‚ΠΎΡ€ ${\bf r}(t)$ хвостом Π² Π½Π°Ρ‡Π°Π»Π΅ ΠΊΠΎΠΎΡ€Π΄ΠΈΠ½Π°Ρ‚, Π° Π·Π°Ρ‚Π΅ΠΌ слСдуСм Π·Π° Π³ΠΎΠ»ΠΎΠ²ΠΎΠΉ стрСлки ΠΏΡ€ΠΈ ΠΈΠ·ΠΌΠ΅Π½Π΅Π½ΠΈΠΈ $t$.Π’Π΅ΠΊΡ‚ΠΎΡ€ «рисуСт» ΠΊΡ€ΠΈΠ²ΡƒΡŽ Ρ‡Π΅Ρ€Π΅Π· пространство ΠΏΡ€ΠΈ ΠΈΠ·ΠΌΠ΅Π½Π΅Π½ΠΈΠΈ $t$.

        ΠŸΡ€Π΅Π΄ΠΏΠΎΠ»ΠΎΠΆΠΈΠΌ, Ρ‡Ρ‚ΠΎ вмСсто этого Ρƒ нас Π΅ΡΡ‚ΡŒ вСкторная функция Π΄Π²ΡƒΡ… ΠΏΠ΅Ρ€Π΅ΠΌΠ΅Π½Π½Ρ‹Ρ…, $${\bf r}(u,v)=\langle x(u,v),y(u,v),z(u,v)\rangle.$$ Как ΠΈ $u$, ΠΈ $v$ ΠΌΠ΅Π½ΡΡŽΡ‚ΡΡ, ΠΌΡ‹ снова прСдставляСм сСбС Π²Π΅ΠΊΡ‚ΠΎΡ€ ${\bf r}(u,v)$ с хвостом Π² происхоТдСниС, ΠΈ Π΅Π³ΠΎ Π³ΠΎΠ»ΠΎΠ²Π° Π²Ρ‹ΠΌΠ΅Ρ‚Π°Π΅Ρ‚ ΠΏΠΎΠ²Π΅Ρ€Ρ…Π½ΠΎΡΡ‚ΡŒ Π² космосС. ΠŸΠΎΠ»Π΅Π·Π½Ρ‹ΠΉ Π°Π½Π°Π»ΠΎΠ³ΠΎΠΌ являСтся тСхнология Π­Π›Π’-видСоэкранов, Π² ΠΊΠΎΡ‚ΠΎΡ€ΠΎΠΉ элСктрон ΠΏΡƒΡˆΠΊΠ° стрСляСт элСктронами Π² сторону экрана. пистолСт Π½Π°ΠΏΡ€Π°Π²Π»Π΅Π½ΠΈΠ΅ пСрСмСщаСтся ΠΏΠΎ Π³ΠΎΡ€ΠΈΠ·ΠΎΠ½Ρ‚Π°Π»ΠΈ ΠΈ Π²Π΅Ρ€Ρ‚ΠΈΠΊΠ°Π»ΠΈ, Ρ‡Ρ‚ΠΎΠ±Ρ‹ Β«Ρ€Π°ΡΠΊΡ€Π°ΡΠΈΡ‚ΡŒΒ» экран с ΠΆΠ΅Π»Π°Π΅ΠΌΡ‹ΠΌ ΠΈΠ·ΠΎΠ±Ρ€Π°ΠΆΠ΅Π½ΠΈΠ΅ΠΌ.На ΠΏΡ€Π°ΠΊΡ‚ΠΈΠΊΠ΅ ΠΏΡƒΡˆΠΊΠ° пСрСмСщаСтся Π³ΠΎΡ€ΠΈΠ·ΠΎΠ½Ρ‚Π°Π»ΡŒΠ½ΠΎ Ρ‡Π΅Ρ€Π΅Π· всю строку, Π·Π°Ρ‚Π΅ΠΌ пСрСмСщаСтся ΠΏΠΎ Π²Π΅Ρ€Ρ‚ΠΈΠΊΠ°Π»ΠΈ Π½Π° ΡΠ»Π΅Π΄ΡƒΡŽΡ‰ΡƒΡŽ строку ΠΈ повторяСт ΠΎΠΏΠ΅Ρ€Π°Ρ†ΠΈΡŽ. Π’ΠΎΡ‡Π½ΠΎ Ρ‚Π°ΠΊ ΠΆΠ΅ ΠΌΠΎΠΆΠ΅Ρ‚ Π±Ρ‹Ρ‚ΡŒ ΠΏΠΎΠ»Π΅Π·Π½ΠΎ ΠΏΡ€Π΅Π΄ΡΡ‚Π°Π²ΠΈΡ‚ΡŒ фиксируя Π·Π½Π°Ρ‡Π΅Π½ΠΈΠ΅ $v$ ΠΈ позволяя ${\bf r}(u,v)$ Π·Π°ΠΌΠ΅Ρ‚Π°Ρ‚ΡŒ ΠΊΡ€ΠΈΠ²ΡƒΡŽ ΠΊΠ°ΠΊ $u$ мСняСтся. Π’ΠΎΠ³Π΄Π° $v$ ΠΌΠΎΠΆΠ΅Ρ‚ Π½Π΅ΠΌΠ½ΠΎΠ³ΠΎ ΠΈΠ·ΠΌΠ΅Π½ΠΈΡ‚ΡŒΡΡ, ΠΈ ${\bf r}(u,v)$ Π·Π°ΠΌΠ΅Ρ‚Π°Π΅Ρ‚ новая кривая ΠΎΡ‡Π΅Π½ΡŒ Π±Π»ΠΈΠ·ΠΊΠ° ΠΊ ΠΏΠ΅Ρ€Π²ΠΎΠΉ. ΠŸΠΎΠ»ΠΎΠΆΠΈΡ‚Π΅ достаточно этих ΠΊΡ€ΠΈΠ²Ρ‹Ρ… вмСстС, ΠΈ ΠΎΠ½ΠΈ ΠΎΠ±Ρ€Π°Π·ΡƒΡŽΡ‚ ΠΏΠΎΠ²Π΅Ρ€Ρ…Π½ΠΎΡΡ‚ΡŒ.

        ΠŸΡ€ΠΈΠΌΠ΅Ρ€ 16.6.1. Рассмотрим Ρ„ΡƒΠ½ΠΊΡ†ΠΈΡŽ ${\bf r}(u,v)=\langle v\cos u,v\sin u, v\rangle$.Для фиксированного значСния $v$, ΠΊΠΎΠ³Π΄Π° $u$ измСняСтся ΠΎΡ‚ 0 Π΄ΠΎ $2\pi$, это описываСт ΠΎΠΊΡ€ΡƒΠΆΠ½ΠΎΡΡ‚ΡŒ радиуса $v$ Π½Π° высотС $v$ Π½Π°Π΄ ΠŸΠ»ΠΎΡΠΊΠΎΡΡ‚ΡŒ $x$-$y$. Π‘Π»ΠΎΠΆΠΈΡ‚Π΅ ΠΌΠ½ΠΎΠ³ΠΎ-ΠΌΠ½ΠΎΠ³ΠΎ всСго этого вмСстС, ΠΈ ΠΎΠ½ΠΈ ΠΎΠ±Ρ€Π°Π·ΡƒΡŽΡ‚ конус, ΠΊΠ°ΠΊ Π½Π° рисункС 16.6.1. Π’ качСствС Π°Π»ΡŒΡ‚Π΅Ρ€Π½Π°Ρ‚ΠΈΠ²Ρ‹ ΠΌΡ‹ ΠΌΠΎΠΆΠ΅ΠΌ Π·Π°Ρ„ΠΈΠΊΡΠΈΡ€ΠΎΠ²Π°Ρ‚ΡŒ $u$, ΠΈ ΠΏΠΎΡΠΊΠΎΠ»ΡŒΠΊΡƒ $v$ Π²Π°Ρ€ΡŒΠΈΡ€ΡƒΠ΅Ρ‚ΡΡ ΠΎΡ‚ $0$ Π΄ΠΎ бСсконСчности, ${\bf r}(u,v)$ отслСТиваСт Π²Π½Π΅ Π»ΠΈΠ½ΠΈΠΈ; ΠΏΡ€ΠΈΠΌΠ΅Ρ€Ρ‹ этих Π»ΠΈΠ½ΠΈΠΉ ΠΌΠΎΠΆΠ½ΠΎ ΡƒΠ²ΠΈΠ΄Π΅Ρ‚ΡŒ Π² стСнкС конуса ΠΈΠ»ΠΈ Π² ΠΎΠ΄ΠΈΠ½ΠΎΡ‡ΠΊΡƒ Π½Π° Ρ‚Ρ€Π΅Ρ‚ΡŒΠ΅ΠΌ Π³Ρ€Π°Ρ„ΠΈΠΊΠ΅ рисунка. $\ΠΊΠ²Π°Π΄Ρ€Π°Ρ‚$

        Рис. 16.6.1. Врассировка повСрхности.

        ΠŸΡ€ΠΈΠΌΠ΅Ρ€ 16.6.2. ΠŸΡƒΡΡ‚ΡŒ ${\bf r}=\langle v\cos u, v\sin u, u\rangle$. Если $v$ постоянной, Ρ€Π΅Π·ΡƒΠ»ΡŒΡ‚ΠΈΡ€ΡƒΡŽΡ‰Π°Ρ кривая прСдставляСт собой ΡΠΏΠΈΡ€Π°Π»ΡŒ (ΠΊΠ°ΠΊ Π² рисунок 13.1.1). Если $u$ постоянна, Ρ€Π΅Π·ΡƒΠ»ΡŒΡ‚ΠΈΡ€ΡƒΡŽΡ‰Π°Ρ кривая прСдставляСт собой ΠΏΡ€ΡΠΌΡƒΡŽ линию Π½Π° высотС $u$ Π² Π½Π°ΠΏΡ€Π°Π²Π»Π΅Π½ΠΈΠΈ $u$ Ρ€Π°Π΄ΠΈΠ°Π½ ΠΎΡ‚ ΠΏΠΎΠ»ΠΎΠΆΠΈΡ‚Π΅Π»ΡŒΠ½Π°Ρ ось $x$. ΠžΠ±Ρ€Π°Ρ‚ΠΈΡ‚Π΅ Π²Π½ΠΈΠΌΠ°Π½ΠΈΠ΅ Π½Π° рисунок 16.6.2, ΠΊΠ°ΠΊ спирали ΠΈ Π»ΠΈΠ½ΠΈΠΈ ΠΎΠΊΡ€Π°ΡˆΠΈΠ²Π°ΡŽΡ‚ ΠΎΠ΄Π½Ρƒ ΠΈ Ρ‚Ρƒ ΠΆΠ΅ ΠΏΠΎΠ²Π΅Ρ€Ρ…Π½ΠΎΡΡ‚ΡŒ ΠΏΠΎ-Ρ€Π°Π·Π½ΠΎΠΌΡƒ. $\ΠΊΠ²Π°Π΄Ρ€Π°Ρ‚$

        Рисунок 16.6.2. Врассировка повСрхности.

        Π­Ρ‚ΠΎΡ‚ ΠΌΠ΅Ρ‚ΠΎΠ΄ позволяСт Π½Π°ΠΌ ΠΏΡ€Π΅Π΄ΡΡ‚Π°Π²Π»ΡΡ‚ΡŒ Π½Π°ΠΌΠ½ΠΎΠ³ΠΎ большС повСрхностСй, Ρ‡Π΅ΠΌ Ρ€Π°Π½Π΅Π΅.

        ΠŸΡ€ΠΈΠΌΠ΅Ρ€ 16.6.3. ΠšΡ€ΠΈΠ²Π°Ρ, заданная Π²Ρ‹Ρ€Π°ΠΆΠ΅Π½ΠΈΠ΅ΠΌ $${\bf r}=\langle (2+\cos(3u/2))\cos u, (2+\cos(3u/2))\sin u, \sin(3u/2)\rangle$$ называСтся ΡƒΠ·Π΅Π»-трилистник. Напомним, Ρ‡Ρ‚ΠΎ ΠΈΠ· Π²Π΅ΠΊΡ‚ΠΎΡ€Π½ΠΎΠ³ΠΎ уравнСния ΠΊΡ€ΠΈΠ²ΠΎΠΉ ΠΌΡ‹ ΠΌΠΎΠΆΠ΅ΠΌ Π²Ρ‹Ρ‡ΠΈΡΠ»ΠΈΡ‚ΡŒ Π΅Π΄ΠΈΠ½ΠΈΡ‡Π½Ρ‹ΠΉ тангСнс $\bf T$, Сдиничная Π½ΠΎΡ€ΠΌΠ°Π»ΡŒ $\bf N$ ΠΈ Π±ΠΈΠ½ΠΎΡ€ΠΌΠ°Π»ΡŒΠ½Ρ‹ΠΉ Π²Π΅ΠΊΡ‚ΠΎΡ€ ${\bf B}={\bf T}\times{\bf N}$; Ρ‚Ρ‹ моТСшь Ρ…ΠΎΡ‚ΠΈΡ‚Π΅ ΠΏΡ€ΠΎΡΠΌΠΎΡ‚Ρ€Π΅Ρ‚ΡŒ Ρ€Π°Π·Π΄Π΅Π» 13. 3. Π±ΠΈΠ½ΠΎΡ€ΠΌΠ°Π»ΡŒ являСтся пСрпСндикулярно ΠΎΠ±ΠΎΠΈΠΌ $\bf T$ ΠΈ $\bf N$; ΠΎΠ΄ΠΈΠ½ ΠΈΠ· способов ΠΈΠ½Ρ‚Π΅Ρ€ΠΏΡ€Π΅Ρ‚ΠΈΡ€ΠΎΠ²Π°Ρ‚ΡŒ это состоит Π² Ρ‚ΠΎΠΌ, Ρ‡Ρ‚ΠΎ ${\bf N}$ ΠΈ ${\bf B}$ ΠΎΠΏΡ€Π΅Π΄Π΅Π»ΡΡŽΡ‚ ΠΏΠ»ΠΎΡΠΊΠΎΡΡ‚ΡŒ, ΠΏΠ΅Ρ€ΠΏΠ΅Π½Π΄ΠΈΠΊΡƒΠ»ΡΡ€Π½ΡƒΡŽ $\bf T$, Ρ‚. Π΅. пСрпСндикулярно ΠΊΡ€ΠΈΠ²ΠΎΠΉ; ΠΏΠΎΡΠΊΠΎΠ»ΡŒΠΊΡƒ ${\bf N}$ ΠΈ ${\bf B}$ пСрпСндикулярны Π΄Ρ€ΡƒΠ³ Π΄Ρ€ΡƒΠ³Ρƒ, ΠΎΠ½ΠΈ ΠΌΠΎΠ³ΡƒΡ‚ Ρ„ΡƒΠ½ΠΊΡ†ΠΈΠΎΠ½ΠΈΡ€ΠΎΠ²Π°Ρ‚ΡŒ ΠΊΠ°ΠΊ $\bf i$ ΠΈ $\bf j$ do для плоскости $x$-$y$.ΠšΠΎΠ½Π΅Ρ‡Π½ΠΎ, $\bf N$ ΠΈ $\bf B$ Ρ„ΡƒΠ½ΠΊΡ†ΠΈΠΈ ΠΎΡ‚ $u$, ΠΈΠ·ΠΌΠ΅Π½ΡΡŽΡ‰ΠΈΠ΅ΡΡ ΠΏΠΎ ΠΌΠ΅Ρ€Π΅ двиТСния вдоль ΠΊΡ€ΠΈΠ²ΠΎΠΉ ${\bf r}(u)$. Π’Π°ΠΊ, Π½Π°ΠΏΡ€ΠΈΠΌΠ΅Ρ€, ${\bf c}(u,v)={\bf N}\cos v+{\bf B}\sin v$ Π²Π΅ΠΊΡ‚ΠΎΡ€Π½ΠΎΠ΅ ΡƒΡ€Π°Π²Π½Π΅Π½ΠΈΠ΅ для Π΅Π΄ΠΈΠ½ΠΈΡ‡Π½ΠΎΠ³ΠΎ ΠΊΡ€ΡƒΠ³Π° Π½Π° плоскости пСрпСндикулярно ΠΊΡ€ΠΈΠ²ΠΎΠΉ, описываСмой $\bf r$, Π·Π° ΠΈΡΠΊΠ»ΡŽΡ‡Π΅Π½ΠΈΠ΅ΠΌ Ρ‚ΠΎΠ³ΠΎ, Ρ‡Ρ‚ΠΎ ΠΎΠ±Ρ‹Ρ‡Π½ΠΎ интСрпрСтация $\bf c$ помСстила Π±Ρ‹ Π΅Π³ΠΎ Ρ†Π΅Π½Ρ‚Ρ€ Π² Π½Π°Ρ‡Π°Π»ΠΎ ΠΊΠΎΠΎΡ€Π΄ΠΈΠ½Π°Ρ‚. ΠœΡ‹ ΠΌΠΎΠΆΠ΅ΠΌ ΠΈΡΠΏΡ€Π°Π²ΠΈΡ‚ΡŒ это, просто Π΄ΠΎΠ±Π°Π²ΠΈΠ² $\bf c$ ΠΊ исходному $\bf r$: ΠΏΡƒΡΡ‚ΡŒ ${\bf f}={\bf r}(u) +{\bf c}(u,v)$. Для фиксированный $u$ рисуСт ΠΎΠΊΡ€ΡƒΠΆΠ½ΠΎΡΡ‚ΡŒ Π²ΠΎΠΊΡ€ΡƒΠ³ Ρ‚ΠΎΡ‡ΠΊΠΈ ${\bf r}(u)$; ΠΊΠ°ΠΊ $u$ мСняСтся, ΠΌΡ‹ ΠΏΠΎΠ»ΡƒΡ‡Π°Π΅ΠΌ ΠΏΠΎΡΠ»Π΅Π΄ΠΎΠ²Π°Ρ‚Π΅Π»ΡŒΠ½ΠΎΡΡ‚ΡŒ Ρ‚Π°ΠΊΠΈΡ… окруТностСй Π²ΠΎΠΊΡ€ΡƒΠ³ ΠΊΡ€ΠΈΠ²ΠΎΠΉ $\bf r$, Ρ‚ΠΎ Π΅ΡΡ‚ΡŒ Ρ‚Ρ€ΡƒΠ±ΠΊΠ° радиуса 1 с $\bf r$ Π² Ρ†Π΅Π½Ρ‚Ρ€Π΅. ΠœΡ‹ ΠΌΠΎΠΆΠ΅ΠΌ Π»Π΅Π³ΠΊΠΎ ΠΈΠ·ΠΌΠ΅Π½ΠΈΡ‚ΡŒ радиус; Π½Π°ΠΏΡ€ΠΈΠΌΠ΅Ρ€ ${\bf r}(u) +a{\bf c}(u,v)$ Π΄Π°Π΅Ρ‚ радиус Ρ‚Ρ€ΡƒΠ±Ρ‹ $a$; ΠΌΡ‹ ΠΌΠΎΠΆΠ΅ΠΌ ΠΈΠ·ΠΌΠ΅Π½ΠΈΡ‚ΡŒ радиус, ΠΊΠ°ΠΊ ΠΌΡ‹ Π΄Π²ΠΈΠ³Π°Ρ‚ΡŒΡΡ ΠΏΠΎ ΠΊΡ€ΠΈΠ²ΠΎΠΉ с ${\bf r}(u) +g(u){\bf c}(u,v)$, Π³Π΄Π΅ $g(u)$ β€” функция ΠΎΡ‚ $u$. Как ΠΏΠΎΠΊΠ°Π·Π°Π½ΠΎ Π² 16.6.3, Ρ‚Ρ€ΡƒΠ΄Π½ΠΎ ΡƒΠ²ΠΈΠ΄Π΅Ρ‚ΡŒ, Ρ‡Ρ‚ΠΎ простой ΡƒΠ·Π΅Π» завязанный; Ρ‚Ρ€ΡƒΠ±ΠΊΠ° Π΄Π΅Π»Π°Π΅Ρ‚ структуру ΠΎΡ‡Π΅Π²ΠΈΠ΄Π½ΠΎΠΉ. Π•ΡΡ‚ΡŒ ΠΊΠΎΠ½Π΅Ρ‡Π½ΠΎ Π½ΠΈΡ‡Π΅Π³ΠΎ особСнного Π² ΡƒΠ·Π»Π΅-трилистникС Π² этом ΠΏΡ€ΠΈΠΌΠ΅Ρ€Π΅; ΠΌΡ‹ ΠΌΠΎΠΆΠ΅ΠΌ ΠΏΠΎΡΡ‚Π°Π²ΠΈΡ‚ΡŒ Ρ‚Ρ€ΡƒΠ±ΠΊΠ° Π²ΠΎΠΊΡ€ΡƒΠ³ (ΠΏΠΎΡ‡Ρ‚ΠΈ) любой ΠΊΡ€ΠΈΠ²ΠΎΠΉ Ρ‚Π°ΠΊΠΈΠΌ ΠΆΠ΅ ΠΎΠ±Ρ€Π°Π·ΠΎΠΌ. $\ΠΊΠ²Π°Π΄Ρ€Π°Ρ‚$

        Рисунок 16.6.3. Π’Ρ€ΡƒΠ±ΠΊΠΈ Π²ΠΎΠΊΡ€ΡƒΠ³ ΡƒΠ·Π»Π°-трилистника радиусом $1/2$ ΠΈ $3\cos(u)/4$.

        Π Π°Π½Π΅Π΅ ΠΌΡ‹ рассматривали повСрхности, Π·Π°Π΄Π°Π½Π½Ρ‹Π΅ Π² Π²ΠΈΠ΄Π΅ $f(x,y)$. Иногда Π±Ρ‹Π²Π°Π΅Ρ‚ ΠΏΠΎΠ»Π΅Π·Π½ΠΎ ΠΏΡ€Π΅Π΄ΡΡ‚Π°Π²ΠΈΡ‚ΡŒ Ρ‚Π°ΠΊΠΈΠ΅ повСрхности Π² Π±ΠΎΠ»Π΅Π΅ общая вСкторная Ρ„ΠΎΡ€ΠΌΠ°, которая довольно проста: ${\bf r}(u,v)=\langle u,v,f(u,v)\rangle$. ИмСна ΠΏΠ΅Ρ€Π΅ΠΌΠ΅Π½Π½Ρ‹Ρ… Π½Π΅ Π²Π°ΠΆΠ½Ρ‹, ΠΊΠΎΠ½Π΅Ρ‡Π½ΠΎ; вмСсто Ρ‚ΠΎΠ³ΠΎ, Ρ‡Ρ‚ΠΎΠ±Ρ‹ ΠΌΠ°ΡΠΊΠΈΡ€ΠΎΠ²Π°Ρ‚ΡŒ $x$ ΠΈ $y$, ΠΌΡ‹ ΠΌΠΎΠΆΠ½ΠΎ просто Π½Π°ΠΏΠΈΡΠ°Ρ‚ΡŒ ${\bf r}(x,y)=\langle x,y,f(x,y)\rangle$.

        Π Π°Π½Π΅Π΅ ΠΌΡ‹ Ρ‚Π°ΠΊΠΆΠ΅ ΠΈΠΌΠ΅Π»ΠΈ Π΄Π΅Π»ΠΎ с повСрхностями, ΠΊΠΎΡ‚ΠΎΡ€Ρ‹Π΅ Π½Π΅ ΡΠ²Π»ΡΡŽΡ‚ΡΡ функциями $x$ ΠΈ $y$; ΠΌΠ½ΠΎΠ³ΠΈΠ΅ ΠΈΠ· Π½ΠΈΡ… Π»Π΅Π³ΠΊΠΎ ΠΏΡ€Π΅Π΄ΡΡ‚Π°Π²ΠΈΡ‚ΡŒ Π² Π²Π΅ΠΊΡ‚ΠΎΡ€Π½ΠΎΠΉ Ρ„ΠΎΡ€ΠΌΠ΅. 2}$ Π²ΠΎΠΊΡ€ΡƒΠ³ оси $z$ Π½Π° высотС $v$. ΠœΡ‹ ΠΌΠΎΠ³Π»ΠΈ Π±Ρ‹ Ρ‚Π°ΠΊΠΆΠ΅ Π²Π·ΡΡ‚ΡŒ ΠΏΡ€ΠΈΠΌΠ΅Ρ€ со сфСричСских ΠΊΠΎΠΎΡ€Π΄ΠΈΠ½Π°Ρ‚ ΠΈ Π½Π°ΠΏΠΈΡΠ°Ρ‚ΡŒ $\langle \sin u\cos v,\sin u\sin v,\cos u\rangle$, Π³Π΄Π΅ фактичСски $u$ ΠΈ $v$ β€” это замаскированныС $\phi$ ΠΈ $\theta$.

        Π’ Sage ΠΎΡ‡Π΅Π½ΡŒ просто ΠΏΠΎΡΡ‚Ρ€ΠΎΠΈΡ‚ΡŒ Π»ΡŽΠ±ΡƒΡŽ ΠΏΠΎΠ²Π΅Ρ€Ρ…Π½ΠΎΡΡ‚ΡŒ, для ΠΊΠΎΡ‚ΠΎΡ€ΠΎΠΉ Ρƒ вас Π΅ΡΡ‚ΡŒ Π²Π΅ΠΊΡ‚ΠΎΡ€Π½ΠΎΠ΅ прСдставлСниС. Иногда использованиС Ρ€Π°Π·Π»ΠΈΡ‡Π½Ρ‹Ρ… Π²Π΅ΠΊΡ‚ΠΎΡ€Π½Ρ‹Ρ… Ρ„ΡƒΠ½ΠΊΡ†ΠΈΠΉ Π΄Π°Π΅Ρ‚ Ρ€Π°Π·Π½Ρ‹Π΅ ΡΡŽΠΆΠ΅Ρ‚Ρ‹, ΠΏΠΎΡ‚ΠΎΠΌΡƒ Ρ‡Ρ‚ΠΎ ΠœΡƒΠ΄Ρ€Π΅Ρ†, ΠΏΠΎ сути, рисуСт ΠΏΠΎΠ²Π΅Ρ€Ρ…Π½ΠΎΡΡ‚ΡŒ, удСрТивая ΠΎΠ΄Π½Ρƒ ΠΏΠ΅Ρ€Π΅ΠΌΠ΅Π½Π½ΡƒΡŽ постоянной, Π° Π·Π°Ρ‚Π΅ΠΌ Π΄Ρ€ΡƒΠ³ΡƒΡŽ. Π—Π° Π½Π°ΠΏΡ€ΠΈΠΌΠ΅Ρ€, Π½Π° рисункС 16.6.2 ΠΊΡ€ΠΈΠ²Ρ‹Π΅ Π½Π° Π΄Π²ΡƒΡ… ΠΏΡ€Π°Π²Ρ‹Ρ… Π³Ρ€Π°Ρ„ΠΈΠΊΠ°Ρ… Π½Π°Π»ΠΎΠΆΠ΅Π½Ρ‹ Π½Π° Π»Π΅Π²Ρ‹ΠΉ Π³Ρ€Π°Ρ„ΠΈΠΊ; Π³Ρ€Π°Ρ„ΠΈΠΊ повСрхности – это просто комбинация Π΄Π²Π° Π½Π°Π±ΠΎΡ€Π° ΠΊΡ€ΠΈΠ²Ρ‹Ρ… с ΠΏΡ€ΠΎΠ±Π΅Π»Π°ΠΌΠΈ, Π·Π°ΠΏΠΎΠ»Π½Π΅Π½Π½Ρ‹ΠΌΠΈ Ρ†Π²Π΅Ρ‚ΠΎΠΌ.

        Π’ΠΎΡ‚ простой, Π½ΠΎ яркий ΠΏΡ€ΠΈΠΌΠ΅Ρ€: ΠΏΠ»ΠΎΡΠΊΠΎΡΡ‚ΡŒ $x+y+z=1$ ΠΌΠΎΠΆΠ΅Ρ‚ Π±Ρ‹Ρ‚ΡŒ Π²ΠΏΠΎΠ»Π½Π΅ СстСствСнно прСдставляСтся ΠΊΠ°ΠΊ $\langle u,v,1-u-v\rangle$. Но ΠΌΡ‹ ΠΌΠΎΠΆΠ½ΠΎ Ρ‚Π°ΠΊΠΆΠ΅ ΠΏΠΎΠ΄ΡƒΠΌΠ°Ρ‚ΡŒ ΠΎ Ρ‚ΠΎΠΌ, Ρ‡Ρ‚ΠΎΠ±Ρ‹ Π½Π°Ρ€ΠΈΡΠΎΠ²Π°Ρ‚ΡŒ ΠΎΠ΄ΠΈΠ½ ΠΈ Ρ‚ΠΎΡ‚ ΠΆΠ΅ самолСт, Π²Ρ‹Π±Ρ€Π°Π² ΠΊΠΎΠ½ΠΊΡ€Π΅Ρ‚Π½Ρ‹ΠΉ Ρ‚ΠΎΡ‡ΠΊΡƒ Π½Π° плоскости, скаТСм, $(1,0,0)$, Π° Π·Π°Ρ‚Π΅ΠΌ рисованиС ΠΊΡ€ΡƒΠ³ΠΎΠ² ΠΈΠ»ΠΈ эллипсы (ΠΈΠ»ΠΈ Π»ΡŽΠ±Ρ‹Π΅ Π΄Ρ€ΡƒΠ³ΠΈΠ΅ ΠΊΡ€ΠΈΠ²Ρ‹Π΅), ΠΊΠ°ΠΊ Ссли Π±Ρ‹ эта Ρ‚ΠΎΡ‡ΠΊΠ° Π±Ρ‹Π»Π° происхоТдСниС Π² плоскости. НапримСр, $\langle 1-v\cos u-v\sin u,v\sin u,v\cos u\rangle$ β€” ΠΎΠ΄Π½Π° ΠΈΠ· Ρ‚Π°ΠΊΠΈΡ… Π²Π΅ΠΊΡ‚ΠΎΡ€Π½Ρ‹Ρ… Ρ„ΡƒΠ½ΠΊΡ†ΠΈΠΉ. ΠžΠ±Ρ€Π°Ρ‚ΠΈΡ‚Π΅ Π²Π½ΠΈΠΌΠ°Π½ΠΈΠ΅, Ρ‡Ρ‚ΠΎ, хотя это ΠΌΠΎΠΆΠ΅Ρ‚ Π½Π΅ ΠΎΡ‡Π΅Π²ΠΈΠ΄Π½ΠΎ, ΠΎΡ‚ΠΊΡƒΠ΄Π° это взялось, довольно Π»Π΅Π³ΠΊΠΎ ΡƒΠ²ΠΈΠ΄Π΅Ρ‚ΡŒ, Ρ‡Ρ‚ΠΎ сумма ΠΊΠΎΠΌΠΏΠΎΠ½Π΅Π½Ρ‚ $x$, $y$ ΠΈ $z$ Π²Π΅ΠΊΡ‚ΠΎΡ€Π° всСгда Ρ€Π°Π²Π½Π° 1. Показана ΠΊΠΎΠΌΠΏΡŒΡŽΡ‚Π΅Ρ€Π½Π°Ρ визуализация самолСта с использованиСм этих Π΄Π²ΡƒΡ… Ρ„ΡƒΠ½ΠΊΡ†ΠΈΠΉ. Π½Π° рисункС 16.6.4.

        Рисунок 16.6.4. Π”Π²Π° изобраТСния ΠΎΠ΄Π½ΠΎΠΉ плоскости.

        ΠŸΡ€Π΅Π΄ΠΏΠΎΠ»ΠΎΠΆΠΈΠΌ, ΠΌΡ‹ Π·Π½Π°Π΅ΠΌ, Ρ‡Ρ‚ΠΎ ΠΏΠ»ΠΎΡΠΊΠΎΡΡ‚ΡŒ содСрТит ΠΎΠΏΡ€Π΅Π΄Π΅Π»Π΅Π½Π½ΡƒΡŽ Ρ‚ΠΎΡ‡ΠΊΡƒ $(x_0,y_0,z_0)$ ΠΈ Π΄Π²Π° Π²Π΅ΠΊΡ‚ΠΎΡ€Π° ${\bf u}=\langle u_0,u_1,u_2\rangle$ ΠΈ ${\bf v}=\langle v_0,v_1,v_2\rangle$ ΠΏΠ°Ρ€Π°Π»Π»Π΅Π»ΡŒΠ½Ρ‹ плоскости, Π½ΠΎ Π½Π΅ Π΄Ρ€ΡƒΠ³ Π΄Ρ€ΡƒΠ³Ρƒ Ρ€Π°Π·Π½ΠΎΠ΅.ΠœΡ‹ Π·Π½Π°Π΅ΠΌ, ΠΊΠ°ΠΊ ΠΏΠΎΠ»ΡƒΡ‡ΠΈΡ‚ΡŒ ΡƒΡ€Π°Π²Π½Π΅Π½ΠΈΠ΅ плоскости Π² Π²ΠΈΠ΄Π΅ $ax+by+cz=d$, сначала вычислив ${\bf u}\times{\bf v}$. это Π΄Π°ΠΆΠ΅ ΠΏΡ€ΠΎΡ‰Π΅ ΠΏΠΎΠ»ΡƒΡ‡ΠΈΡ‚ΡŒ Π²Π΅ΠΊΡ‚ΠΎΡ€Π½ΠΎΠ΅ ΡƒΡ€Π°Π²Π½Π΅Π½ΠΈΠ΅: $${\bf r}(u,v) = \langle x_0,y_0,z_0\rangle + u{\bf u} + v{\bf v}.$$ ΠŸΠ΅Ρ€Π²Ρ‹ΠΉ Π²Π΅ΠΊΡ‚ΠΎΡ€ ΠΏΠΎΠΏΠ°Π΄Π°Π΅Ρ‚ Π² Ρ‚ΠΎΡ‡ΠΊΡƒ $(x_0,y_0,z_0)$ ΠΈ Π·Π°Ρ‚Π΅ΠΌ, Π²Π°Ρ€ΡŒΠΈΡ€ΡƒΡ $u$ ΠΈ $v$, $u{\bf u} + v{\bf v}$ ΠΏΠΎΠΏΠ°Π΄Π°ΡŽΡ‚ Π² ΠΊΠ°ΠΆΠ΄ΡƒΡŽ Ρ‚ΠΎΡ‡ΠΊΡƒ плоскости.

        Π’ΠΎΠ·Π²Ρ€Π°Ρ‰Π°ΡΡΡŒ ΠΊ $x+y+z=1$, Ρ‚ΠΎΡ‡ΠΊΠΈ $(1,0,0)$, $(0,1,0)$ ΠΈ $(0,0,1)$ всС Π² самолСтС. Вычитая ΠΊΠΎΠΎΡ€Π΄ΠΈΠ½Π°Ρ‚Ρ‹, ΠΌΡ‹ Π²ΠΈΠ΄ΠΈΠΌ, Ρ‡Ρ‚ΠΎ $\langle -1,0,1\rangle$ ΠΈ $\langle -1,1,0\rangle$ ΠΏΠ°Ρ€Π°Π»Π»Π΅Π»ΡŒΠ½Ρ‹ плоскости, поэтому Ρ‚Ρ€Π΅Ρ‚ΡŒΡ вСкторная Ρ„ΠΎΡ€ΠΌΠ° для этой плоскости $$\langle 1,0,0\rangle + u\langle -1,0,1\rangle + v\langle -1,1,0\ΡƒΠ³ΠΎΠ» = \langle 1-u-v,v,u\rangle.$$ Π­Ρ‚ΠΎ явно ΠΎΡ‡Π΅Π½ΡŒ ΠΏΠΎΡ…ΠΎΠΆΠ΅ Π½Π° ΠΏΠ΅Ρ€Π²ΡƒΡŽ Π½Π°ΠΉΠ΄Π΅Π½Π½ΡƒΡŽ Π½Π°ΠΌΠΈ Ρ„ΠΎΡ€ΠΌΡƒ.

        ΠœΡ‹ ΡƒΠΆΠ΅ Π²ΠΈΠ΄Π΅Π»ΠΈ (Ρ€Π°Π·Π΄Π΅Π» 15.4) ΠΊΠ°ΠΊ Π½Π°ΠΉΡ‚ΠΈ ΠΏΠ»ΠΎΡ‰Π°Π΄ΡŒ повСрхности, Ссли ΠΎΠ½Π° опрСдСляСтся Π² Π²ΠΈΠ΄Π΅ $f(x,y)$. НахоТдСниС ΠΏΠ»ΠΎΡ‰Π°Π΄ΠΈ, ΠΊΠΎΠ³Π΄Π° ΠΏΠΎΠ²Π΅Ρ€Ρ…Π½ΠΎΡΡ‚ΡŒ Π² Π²ΠΈΠ΄Π΅ Π²Π΅ΠΊΡ‚ΠΎΡ€Π½ΠΎΠΉ Ρ„ΡƒΠ½ΠΊΡ†ΠΈΠΈ ΠΎΡ‡Π΅Π½ΡŒ ΠΏΠΎΡ…ΠΎΠΆ. Глядя Π½Π° ΡΡŽΠΆΠ΅Ρ‚Ρ‹ повСрхностСй, ΠΊΠΎΡ‚ΠΎΡ€Ρ‹Π΅ ΠΌΡ‹ Ρ‚ΠΎΠ»ΡŒΠΊΠΎ Ρ‡Ρ‚ΠΎ Π²ΠΈΠ΄Π΅Π»ΠΈ, ΠΎΡ‡Π΅Π²ΠΈΠ΄Π½ΠΎ, Ρ‡Ρ‚ΠΎ Π΄Π²Π° Π½Π°Π±ΠΎΡ€Π° ΠΊΡ€ΠΈΠ²Ρ‹Ρ… ΠΊΠΎΡ‚ΠΎΡ€Ρ‹Π΅ Π·Π°ΠΏΠΎΠ»Π½ΡΡŽΡ‚ ΠΏΠΎΠ²Π΅Ρ€Ρ…Π½ΠΎΡΡ‚ΡŒ, дСлят Π΅Π΅ Π½Π° сСтку, ΠΈ Ρ‡Ρ‚ΠΎ ΠΏΡ€ΠΎΠ±Π΅Π»Ρ‹ Π² сСткС ΠΏΡ€ΠΈΠΌΠ΅Ρ€Π½ΠΎ ΠΏΠ°Ρ€Π°Π»Π»Π΅Π»ΠΎΠ³Ρ€Π°ΠΌΠΌΡ‹. Как ΠΈ ΠΏΡ€Π΅ΠΆΠ΄Π΅, это ΠΊΠ»ΡŽΡ‡: ΠΌΡ‹ ΠΌΠΎΠΆΠ΅ΠΌ Π·Π°ΠΏΠΈΡΠ°Ρ‚ΡŒ ΠΏΠ»ΠΎΡ‰Π°Π΄ΡŒ Ρ‚ΠΈΠΏΠΈΡ‡Π½ΠΎΠ³ΠΎ малСнького ΠΏΠ°Ρ€Π°Π»Π»Π΅Π»ΠΎΠ³Ρ€Π°ΠΌΠΌΠ° ΠΈ слоТитС ΠΈΡ… всС с ΠΈΠ½Ρ‚Π΅Π³Ρ€Π°Π»ΠΎΠΌ.

        ΠŸΡ€Π΅Π΄ΠΏΠΎΠ»ΠΎΠΆΠΈΠΌ, ΠΌΡ‹ Ρ…ΠΎΡ‚ΠΈΠΌ Π°ΠΏΠΏΡ€ΠΎΠΊΡΠΈΠΌΠΈΡ€ΠΎΠ²Π°Ρ‚ΡŒ ΠΏΠ»ΠΎΡ‰Π°Π΄ΡŒ повСрхности ${\bf r}(u,v)$ Π²Π±Π»ΠΈΠ·ΠΈ ${\bf r}(u_0,v_0)$. Π€ΡƒΠ½ΠΊΡ†ΠΈΠΈ ${\bf r}(u,v_0)$ ΠΈ ${\bf r}(u_0,v)$ ΠΎΠΏΡ€Π΅Π΄Π΅Π»ΡΡŽΡ‚ Π΄Π²Π΅ ΠΊΡ€ΠΈΠ²Ρ‹Π΅, ΠΏΠ΅Ρ€Π΅ΡΠ΅ΠΊΠ°ΡŽΡ‰ΠΈΠ΅ΡΡ Π² Ρ‚ΠΎΡ‡ΠΊΠ΅ ${\bf Π³}(ΠΈ_0,v_0)$. ΠŸΡ€ΠΎΠΈΠ·Π²ΠΎΠ΄Π½Ρ‹Π΅ ΠΎΡ‚ $\bf r$ Π΄Π°ΡŽΡ‚ Π½Π°ΠΌ Π²Π΅ΠΊΡ‚ΠΎΡ€Ρ‹, ΠΊΠ°ΡΠ°ΡŽΡ‰ΠΈΠ΅ΡΡ эти Π΄Π²Π΅ ΠΊΡ€ΠΈΠ²Ρ‹Π΅: ${\bf r}_u(u_0,v_0)$ ΠΈ ${\bf r}_v(u_0,v_0)$, ΠΈ Ρ‚ΠΎΠ³Π΄Π° ${\bf r}_u(u_0,v_0)\,du$ ΠΈ ${\bf r}_v(u_0,v_0)\,dv$ Π΄Π²Π° ΠΌΠ°Π»Ρ‹Ρ… ΠΊΠ°ΡΠ°Ρ‚Π΅Π»ΡŒΠ½Ρ‹Ρ… Π²Π΅ΠΊΡ‚ΠΎΡ€Π°, Π΄Π»ΠΈΠ½Ρ‹ ΠΊΠΎΡ‚ΠΎΡ€Ρ‹Ρ… ΠΌΠΎΠΆΠ½ΠΎ ΠΈΡΠΏΠΎΠ»ΡŒΠ·ΠΎΠ²Π°Ρ‚ΡŒ Π² качСствС Π΄Π»ΠΈΠ½Ρ‹ сторон ΠΏΡ€ΠΈΠ±Π»ΠΈΠΆΠ°ΡŽΡ‰Π΅Π³ΠΎ ΠΏΠ°Ρ€Π°Π»Π»Π΅Π»ΠΎΠ³Ρ€Π°ΠΌΠΌΠ°.2}\,dv\,du={\pi\sqrt2\over2}+ {\ ΠΏΠΈ \ ΠΏΠ΅Ρ€ (\ sqrt2 + 1) \ Π±ΠΎΠ»Π΅Π΅ 2}. $ $ $\ΠΊΠ²Π°Π΄Ρ€Π°Ρ‚$

        УпраТнСния 16.6

        Π’Ρ‹ ΠΌΠΎΠΆΠ΅Ρ‚Π΅ ΠΈΡΠΏΠΎΠ»ΡŒΠ·ΠΎΠ²Π°Ρ‚ΡŒ эти ячСйки Sage для графичСских повСрхностСй. ΠŸΠ΅Ρ€Π²Ρ‹ΠΉ ΠΏΡ€ΠΈΠΌΠ΅Ρ€ прСдставляСт собой Ρ‚Ρ€ΡƒΠ±ΠΊΡƒ Π²ΠΎΠΊΡ€ΡƒΠ³ ΡƒΠ·Π»Π° Врилистник, Π²Ρ‚ΠΎΡ€ΠΎΠΉ прСдставляСт собой конус.

        ΠŸΡ€ΠΈΠΌΠ΅Ρ€ 16.6.1 ΠžΠΏΠΈΡˆΠΈΡ‚Π΅ ΠΈΠ»ΠΈ зарисуйтС ΠΏΠΎΠ²Π΅Ρ€Ρ…Π½ΠΎΡΡ‚ΡŒ с Π·Π°Π΄Π°Π½Π½ΠΎΠΉ Π²Π΅ΠΊΡ‚ΠΎΡ€Π½ΠΎΠΉ Ρ„ΡƒΠ½ΠΊΡ†ΠΈΠ΅ΠΉ.

          Π°. ${\bf r}(u,v) = \langle u+v,3-v,1+4u+5v\rangle$

          Π±. ${\bf r}(u,v) = \langle 2\sin u, 3\cos u, v\rangle$

          Π².2$. (ΠΎΡ‚Π²Π΅Ρ‡Π°Ρ‚ΡŒ)

          ΠŸΡ€ΠΈΠΌΠ΅Ρ€ 16.6.16 ΠŸΠΎΠ²Π΅Ρ€Ρ…Π½ΠΎΡΡ‚ΡŒ $f(x,y)$ ΠΌΠΎΠΆΠ΅Ρ‚ Π±Ρ‹Ρ‚ΡŒ прСдставлСна ​​вСктором функция $\langle x,y,f(x,y)\rangle$. НастройтС ΠΈΠ½Ρ‚Π΅Π³Ρ€Π°Π» ΠΏΠ»ΠΎΡ‰Π°Π΄ΠΈ повСрхности, ΠΈΡΠΏΠΎΠ»ΡŒΠ·ΡƒΡ эту Π²Π΅ΠΊΡ‚ΠΎΡ€Π½ΡƒΡŽ Ρ„ΡƒΠ½ΠΊΡ†ΠΈΡŽ ΠΈ ΡΡ€Π°Π²Π½ΠΈΡ‚ΡŒ с ΠΈΠ½Ρ‚Π΅Π³Ρ€Π°Π»ΠΎΠΌ ΠΎΡ‚ Ρ€Π°Π·Π΄Π΅Π» 15.4.

          Π€ΠΎΡ€ΠΌΡƒΠ»Π° радиуса ΠΊΡ€ΠΈΠ²ΠΈΠ·Π½Ρ‹ – Π£Π·Π½Π°ΠΉΡ‚Π΅ Ρ„ΠΎΡ€ΠΌΡƒΠ»Ρƒ для радиуса ΠΊΡ€ΠΈΠ²ΠΈΠ·Π½Ρ‹

          Радиусом ΠΊΡ€ΠΈΠ²ΠΈΠ·Π½Ρ‹ ΠΊΡ€ΠΈΠ²ΠΎΠΉ называСтся любой ΠΏΡ€ΠΈΠ±Π»ΠΈΠ·ΠΈΡ‚Π΅Π»ΡŒΠ½Ρ‹ΠΉ радиус окруТности Π² любой Π΄Π°Π½Π½ΠΎΠΉ Ρ‚ΠΎΡ‡ΠΊΠ΅. По ΠΌΠ΅Ρ€Π΅ двиТСния ΠΏΠΎ ΠΊΡ€ΠΈΠ²ΠΎΠΉ радиус ΠΊΡ€ΠΈΠ²ΠΈΠ·Π½Ρ‹ измСняСтся.Π€ΠΎΡ€ΠΌΡƒΠ»Π° радиуса ΠΊΡ€ΠΈΠ²ΠΈΠ·Π½Ρ‹ обозначаСтся ΠΊΠ°ΠΊ Β«RΒ». Π’Π΅Π»ΠΈΡ‡ΠΈΠ½Π°, Π½Π° ΠΊΠΎΡ‚ΠΎΡ€ΡƒΡŽ кривая прСвращаСтся ΠΈΠ· плоской Π² β€‹β€‹ΠΊΡ€ΠΈΠ²ΡƒΡŽ ΠΈ ΠΈΠ· ΠΊΡ€ΠΈΠ²ΠΎΠΉ ΠΎΠ±Ρ€Π°Ρ‚Π½ΠΎ Π² ΠΏΡ€ΡΠΌΡƒΡŽ, называСтся ΠΊΡ€ΠΈΠ²ΠΈΠ·Π½ΠΎΠΉ. Π­Ρ‚ΠΎ скалярная Π²Π΅Π»ΠΈΡ‡ΠΈΠ½Π°. Радиус ΠΊΡ€ΠΈΠ²ΠΈΠ·Π½Ρ‹ ΠΎΠ±Ρ€Π°Ρ‚Π½ΠΎ ΠΏΡ€ΠΎΠΏΠΎΡ€Ρ†ΠΈΠΎΠ½Π°Π»Π΅Π½ ΠΊΡ€ΠΈΠ²ΠΈΠ·Π½Π΅. Радиус ΠΊΡ€ΠΈΠ²ΠΈΠ·Π½Ρ‹ β€” это Π½Π΅ Ρ€Π΅Π°Π»ΡŒΠ½Π°Ρ Ρ„ΠΎΡ€ΠΌΠ° ΠΈΠ»ΠΈ Ρ„ΠΈΠ³ΡƒΡ€Π°, Π° Π²ΠΎΠΎΠ±Ρ€Π°ΠΆΠ°Π΅ΠΌΡ‹ΠΉ ΠΊΡ€ΡƒΠ³. Π”Π°Π²Π°ΠΉΡ‚Π΅ ΠΏΠΎΠ΄Ρ€ΠΎΠ±Π½ΠΎ Ρ€Π°Π·Π±Π΅Ρ€Π΅ΠΌ Ρ„ΠΎΡ€ΠΌΡƒΠ»Ρƒ радиуса ΠΊΡ€ΠΈΠ²ΠΈΠ·Π½Ρ‹, ΠΈΡΠΏΠΎΠ»ΡŒΠ·ΡƒΡ Ρ€Π΅ΡˆΠ΅Π½Π½Ρ‹Π΅ ΠΏΡ€ΠΈΠΌΠ΅Ρ€Ρ‹ Π² ΡΠ»Π΅Π΄ΡƒΡŽΡ‰Π΅ΠΌ Ρ€Π°Π·Π΄Π΅Π»Π΅.

          Π§Ρ‚ΠΎ Ρ‚Π°ΠΊΠΎΠ΅ радиус Ρ„ΠΎΡ€ΠΌΡƒΠ»Ρ‹ ΠΊΡ€ΠΈΠ²ΠΈΠ·Π½Ρ‹?

          РасстояниС ΠΎΡ‚ Π²Π΅Ρ€ΡˆΠΈΠ½Ρ‹ Π΄ΠΎ Ρ†Π΅Π½Ρ‚Ρ€Π° ΠΊΡ€ΠΈΠ²ΠΈΠ·Π½Ρ‹ извСстно ΠΊΠ°ΠΊ радиус ΠΊΡ€ΠΈΠ²ΠΈΠ·Π½Ρ‹ (обозначаСтся R).{2}} |}\)

        Π³Π΄Π΅ K β€” ΠΊΡ€ΠΈΠ²ΠΈΠ·Π½Π° ΠΊΡ€ΠΈΠ²ΠΎΠΉ, K = dT/ds, (функция тангСнса)

        R радиус ΠΊΡ€ΠΈΠ²ΠΈΠ·Π½Ρ‹

        Π Π°Π·Π±ΠΈΠ²Π°ΠΉΡ‚Π΅ слоТныС ΠΊΠΎΠ½Ρ†Π΅ΠΏΡ†ΠΈΠΈ с ΠΏΠΎΠΌΠΎΡ‰ΡŒΡŽ простых Π²ΠΈΠ·ΡƒΠ°Π»ΡŒΠ½Ρ‹Ρ… срСдств.

        ΠœΠ°Ρ‚Π΅ΠΌΠ°Ρ‚ΠΈΠΊΠ° большС Π½Π΅ Π±ΡƒΠ΄Π΅Ρ‚ слоТным ΠΏΡ€Π΅Π΄ΠΌΠ΅Ρ‚ΠΎΠΌ, особСнно ΠΊΠΎΠ³Π΄Π° Π²Ρ‹ ΠΏΠΎΠ½ΠΈΠΌΠ°Π΅Ρ‚Π΅ ΠΊΠΎΠ½Ρ†Π΅ΠΏΡ†ΠΈΠΈ с ΠΏΠΎΠΌΠΎΡ‰ΡŒΡŽ Π²ΠΈΠ·ΡƒΠ°Π»ΠΈΠ·Π°Ρ†ΠΈΠΉ.

        Π—Π°Π±Ρ€ΠΎΠ½ΠΈΡ€ΡƒΠΉΡ‚Π΅ бСсплатный ΠΏΡ€ΠΎΠ±Π½Ρ‹ΠΉ ΡƒΡ€ΠΎΠΊ

        Π”Π°Π²Π°ΠΉΡ‚Π΅ быстро рассмотрим ΠΏΠ°Ρ€Ρƒ ΠΏΡ€ΠΈΠΌΠ΅Ρ€ΠΎΠ², Ρ‡Ρ‚ΠΎΠ±Ρ‹ Π»ΡƒΡ‡ΡˆΠ΅ ΠΏΠΎΠ½ΡΡ‚ΡŒ Ρ„ΠΎΡ€ΠΌΡƒΠ»Ρƒ радиуса ΠΊΡ€ΠΈΠ²ΠΈΠ·Π½Ρ‹.{2}} |}\).

        ΠšΡ€ΠΈΠ²ΠΈΠ·Π½Π° плоских ΠΊΡ€ΠΈΠ²Ρ‹Ρ…

        ΠŸΡƒΡΡ‚ΡŒ плоская кривая \(C\) парамСтричСски Π·Π°Π΄Π°Π½Π° радиус-Π²Π΅ΠΊΡ‚ΠΎΡ€ΠΎΠΌ \(\mathbf{r}\left( t \right).\), Π° Ρ‚ΠΎΡ‡ΠΊΠ° \(M\) двиТСтся ΠΏΠΎ ΠΊΡ€ΠΈΠ²ΠΎΠΉ \(C\ ), Π½Π°ΠΏΡ€Π°Π²Π»Π΅Π½ΠΈΠ΅ ΠΊΠ°ΡΠ°Ρ‚Π΅Π»ΡŒΠ½ΠΎΠΉ мСняСтся (рис. \(1\)).

        Рис. 1.

        ΠšΡ€ΠΈΠ²ΠΈΠ·Π½Ρƒ ΠΊΡ€ΠΈΠ²ΠΎΠΉ ΠΌΠΎΠΆΠ½ΠΎ ΠΎΠΏΡ€Π΅Π΄Π΅Π»ΠΈΡ‚ΡŒ ΠΊΠ°ΠΊ ΠΎΡ‚Π½ΠΎΡˆΠ΅Π½ΠΈΠ΅ ΡƒΠ³Π»Π° ΠΏΠΎΠ²ΠΎΡ€ΠΎΡ‚Π° ΠΊΠ°ΡΠ°Ρ‚Π΅Π»ΡŒΠ½ΠΎΠΉ \(\Delta \varphi\) ΠΊ Π΄Π»ΠΈΠ½Π΅ ΠΏΡ€ΠΎΠΉΠ΄Π΅Π½Π½ΠΎΠΉ Π΄ΡƒΠ³ΠΈ \(\Delta s = M{M_1}.\) Π­Ρ‚ΠΎ ΠΎΡ‚Π½ΠΎΡˆΠ΅Π½ΠΈΠ΅ \(\frac{{\Delta \varphi}}{{\Delta s}}\) называСтся срСднСй ΠΊΡ€ΠΈΠ²ΠΈΠ·Π½ΠΎΠΉ ΠΊΡ€ΠΈΠ²ΠΎΠΉ.Когда Ρ‚ΠΎΡ‡ΠΊΠ° \({M_1}\) приблиТаСтся ΠΊ Ρ‚ΠΎΡ‡ΠΊΠ΅ \(M,\), ΠΌΡ‹ ΠΏΠΎΠ»ΡƒΡ‡Π°Π΅ΠΌ ΠΊΡ€ΠΈΠ²ΠΈΠ·Π½Ρƒ ΠΊΡ€ΠΈΠ²ΠΎΠΉ Π² Ρ‚ΠΎΡ‡ΠΊΠ΅ \(M:\)

        \[k = \lim\limits_{\Delta s \to 0} \frac{{\Delta \varphi}}{{\Delta s}} = \frac{{d\varphi}}{{ds}}. \]

        Ясно, Ρ‡Ρ‚ΠΎ ΠΊΡ€ΠΈΠ²ΠΈΠ·Π½Π° \(ΠΊ\) Π² ΠΎΠ±Ρ‰Π΅ΠΌ случаС ΠΌΠΎΠΆΠ΅Ρ‚ Π±Ρ‹Ρ‚ΡŒ ΠΊΠ°ΠΊ ΠΏΠΎΠ»ΠΎΠΆΠΈΡ‚Π΅Π»ΡŒΠ½ΠΎΠΉ, Ρ‚Π°ΠΊ ΠΈ ΠΎΡ‚Ρ€ΠΈΡ†Π°Ρ‚Π΅Π»ΡŒΠ½ΠΎΠΉ Π² зависимости ΠΎΡ‚ направлСния вращСния ΠΊΠ°ΡΠ°Ρ‚Π΅Π»ΡŒΠ½ΠΎΠΉ. {\prime\prime}\) β€” пСрвая ΠΈ вторая ΠΏΡ€ΠΎΠΈΠ·Π²ΠΎΠ΄Π½Ρ‹Π΅ радиус-Π²Π΅ΠΊΡ‚ΠΎΡ€Π°.{\ Π³ΠΈΠ΄Ρ€ΠΎΡ€Π°Π·Ρ€Ρ‹Π²Π° {3} {2}}}}}.\]

        ΠžΠ±Ρ€Π°Ρ‚Π½Π°Ρ Π²Π΅Π»ΠΈΡ‡ΠΈΠ½Π° ΠΊΡ€ΠΈΠ²ΠΈΠ·Π½Ρ‹ называСтся радиусом ΠΊΡ€ΠΈΠ²ΠΈΠ·Π½Ρ‹:

        \[R = \frac{1}{{\left| ΠΊ \ΠΏΡ€Π°Π²ΠΎ|}}.\]

        ΠžΠΊΡ€ΡƒΠΆΠ½ΠΎΡΡ‚ΡŒ с Ρ‚Π°ΠΊΠΈΠΌ радиусом ΠΈ Ρ†Π΅Π½Ρ‚Ρ€ΠΎΠΌ, располоТСнным Π½Π° Π²Π½ΡƒΡ‚Ρ€Π΅Π½Π½Π΅ΠΉ Π½ΠΎΡ€ΠΌΠ°Π»ΠΈ, Π±ΡƒΠ΄Π΅Ρ‚ Π½Π°ΠΈΠ±ΠΎΠ»Π΅Π΅ Ρ‚ΠΎΡ‡Π½ΠΎ Π°ΠΏΠΏΡ€ΠΎΠΊΡΠΈΠΌΠΈΡ€ΠΎΠ²Π°Ρ‚ΡŒ ΠΏΠ»ΠΎΡΠΊΡƒΡŽ ΠΊΡ€ΠΈΠ²ΡƒΡŽ Π² Π΄Π°Π½Π½ΠΎΠΉ Ρ‚ΠΎΡ‡ΠΊΠ΅ (рис. \(2\)).

        Рис. 2.

        Π’Π°ΠΊΠΎΠΉ ΠΊΡ€ΡƒΠ³ называСтся ΡΠΎΠΏΡ€ΠΈΠΊΠ°ΡΠ°ΡŽΡ‰ΠΈΠΌΡΡ ΠΊΡ€ΡƒΠ³ΠΎΠΌ.

        Π‘ΠΌ. Ρ€Π΅ΡˆΠ΅Π½Π½Ρ‹Π΅ ΠΏΡ€ΠΎΠ±Π»Π΅ΠΌΡ‹ Π½Π° стр. 2.

        Π’Π΅ΠΊΡ‚ΠΎΡ€ полоТСния – объяснСниС ΠΈ ΠΏΡ€ΠΈΠΌΠ΅Ρ€Ρ‹

        ΠœΡ‹ ΠΌΠΎΠΆΠ΅ΠΌ ΠΈΡΠΏΠΎΠ»ΡŒΠ·ΠΎΠ²Π°Ρ‚ΡŒ Π²Π΅ΠΊΡ‚ΠΎΡ€ полоТСния , Ρ‡Ρ‚ΠΎΠ±Ρ‹ ΡΠΎΠΎΠ±Ρ‰ΠΈΡ‚ΡŒ Π½Π°ΠΌ ΠΏΠΎΠ»ΠΎΠΆΠ΅Π½ΠΈΠ΅ ΠΎΠ΄Π½ΠΎΠ³ΠΎ ΠΎΠ±ΡŠΠ΅ΠΊΡ‚Π° ΠΎΡ‚Π½ΠΎΡΠΈΡ‚Π΅Π»ΡŒΠ½ΠΎ Π΄Ρ€ΡƒΠ³ΠΎΠ³ΠΎ.Π’ частности, Π²Π΅ΠΊΡ‚ΠΎΡ€ полоТСния:

        Β«Π’Π΅ΠΊΡ‚ΠΎΡ€, ΠΊΠΎΡ‚ΠΎΡ€Ρ‹ΠΉ ΡƒΠΊΠ°Π·Ρ‹Π²Π°Π΅Ρ‚ мСстополоТСниС ΠΈΠ»ΠΈ ΠΏΠΎΠ»ΠΎΠΆΠ΅Π½ΠΈΠ΅ Π΄Π°Π½Π½ΠΎΠΉ Ρ‚ΠΎΡ‡ΠΊΠΈ ΠΎΡ‚Π½ΠΎΡΠΈΡ‚Π΅Π»ΡŒΠ½ΠΎ ΠΏΡ€ΠΎΠΈΠ·Π²ΠΎΠ»ΡŒΠ½ΠΎΠΉ ΠΊΠΎΠ½Ρ‚Ρ€ΠΎΠ»ΡŒΠ½ΠΎΠΉ Ρ‚ΠΎΡ‡ΠΊΠΈ, Ρ‚Π°ΠΊΠΎΠΉ ΠΊΠ°ΠΊ Π½Π°Ρ‡Π°Π»ΠΎ ΠΊΠΎΠΎΡ€Π΄ΠΈΠ½Π°Ρ‚Β».

        Π’ этом Ρ€Π°Π·Π΄Π΅Π»Π΅ ΠΌΡ‹ обсудим ΡΠ»Π΅Π΄ΡƒΡŽΡ‰ΠΈΠ΅ аспСкты Π²Π΅ΠΊΡ‚ΠΎΡ€ΠΎΠ² полоТСния:

        • Π§Ρ‚ΠΎ Ρ‚Π°ΠΊΠΎΠ΅ Π²Π΅ΠΊΡ‚ΠΎΡ€ полоТСния?
        • Как Π½Π°ΠΉΡ‚ΠΈ Π²Π΅ΠΊΡ‚ΠΎΡ€ полоТСния

        Π§Ρ‚ΠΎ Ρ‚Π°ΠΊΠΎΠ΅ Π²Π΅ΠΊΡ‚ΠΎΡ€ полоТСния?

        Часто Π²Π΅ΠΊΡ‚ΠΎΡ€Ρ‹, ΠΊΠΎΡ‚ΠΎΡ€Ρ‹Π΅ Π½Π°Ρ‡ΠΈΠ½Π°ΡŽΡ‚ΡΡ Π² Π½Π°Ρ‡Π°Π»Π΅ ΠΊΠΎΠΎΡ€Π΄ΠΈΠ½Π°Ρ‚ ΠΈ Π·Π°ΠΊΠ°Π½Ρ‡ΠΈΠ²Π°ΡŽΡ‚ΡΡ Π² любой ΠΏΡ€ΠΎΠΈΠ·Π²ΠΎΠ»ΡŒΠ½ΠΎΠΉ Ρ‚ΠΎΡ‡ΠΊΠ΅, Π½Π°Π·Ρ‹Π²Π°ΡŽΡ‚ΡΡ Π²Π΅ΠΊΡ‚ΠΎΡ€Π°ΠΌΠΈ полоТСния. Они ΠΈΡΠΏΠΎΠ»ΡŒΠ·ΡƒΡŽΡ‚ΡΡ для опрСдСлСния полоТСния Ρ‚ΠΎΡ‡ΠΊΠΈ ΠΎΡ‚Π½ΠΎΡΠΈΡ‚Π΅Π»ΡŒΠ½ΠΎ Π½Π°Ρ‡Π°Π»Π° ΠΊΠΎΠΎΡ€Π΄ΠΈΠ½Π°Ρ‚.

        НаправлСниС Π²Π΅ΠΊΡ‚ΠΎΡ€Π° полоТСния ΡƒΠΊΠ°Π·Ρ‹Π²Π°Π΅Ρ‚ ΠΎΡ‚ Π½Π°Ρ‡Π°Π»Π° ΠΊΠΎΠΎΡ€Π΄ΠΈΠ½Π°Ρ‚ ΠΊ Π·Π°Π΄Π°Π½Π½ΠΎΠΉ Ρ‚ΠΎΡ‡ΠΊΠ΅. Π’ Π΄Π΅ΠΊΠ°Ρ€Ρ‚ΠΎΠ²ΠΎΠΉ систСмС ΠΊΠΎΠΎΡ€Π΄ΠΈΠ½Π°Ρ‚, Ссли Ρ‚ΠΎΡ‡ΠΊΠ° O являСтся Π½Π°Ρ‡Π°Π»ΠΎΠΌ ΠΊΠΎΠΎΡ€Π΄ΠΈΠ½Π°Ρ‚, Π° Q являСтся Π½Π΅ΠΊΠΎΡ‚ΠΎΡ€ΠΎΠΉ Ρ‚ΠΎΡ‡ΠΊΠΎΠΉ (x1, y1), Ρ‚ΠΎ Π²Π΅ΠΊΡ‚ΠΎΡ€ полоТСния, Π½Π°ΠΏΡ€Π°Π²Π»Π΅Π½Π½Ρ‹ΠΉ ΠΈΠ· Ρ‚ΠΎΡ‡ΠΊΠΈ O Π² Ρ‚ΠΎΡ‡ΠΊΡƒ Q, прСдставляСтся ΠΊΠ°ΠΊ OQ . Π’ Ρ‚Ρ€Π΅Ρ…ΠΌΠ΅Ρ€Π½ΠΎΠΌ пространствС, Ссли O = (0,0,0) ΠΈ Q = (x1, y1, z1), Ρ‚ΠΎ Π²Π΅ΠΊΡ‚ΠΎΡ€ полоТСния r Ρ‚ΠΎΡ‡ΠΊΠΈ Q прСдставляСтся ΡΠ»Π΅Π΄ΡƒΡŽΡ‰ΠΈΠΌ ΠΎΠ±Ρ€Π°Π·ΠΎΠΌ:

        r = x1i + y1j + z1k

        ΠŸΡ€Π΅Π΄ΠΏΠΎΠ»ΠΎΠΆΠΈΠΌ, Ρƒ нас Π΅ΡΡ‚ΡŒ Π΄Π²Π° Π²Π΅ΠΊΡ‚ΠΎΡ€Π°, A ΠΈ B, с Π²Π΅ΠΊΡ‚ΠΎΡ€Π°ΠΌΠΈ полоТСния a = (2,4) ΠΈ b = (3, 5) соотвСтствСнно.Π’ΠΎΠ³Π΄Π° ΠΌΡ‹ ΠΌΠΎΠΆΠ΅ΠΌ Π·Π°ΠΏΠΈΡΠ°Ρ‚ΡŒ ΠΊΠΎΠΎΡ€Π΄ΠΈΠ½Π°Ρ‚Ρ‹ Π²Π΅ΠΊΡ‚ΠΎΡ€ΠΎΠ² A ΠΈ B ΠΊΠ°ΠΊ:

        A = (2,4), B = (3, 5)

        опрСдСляя Π²Π΅ΠΊΡ‚ΠΎΡ€ полоТСния Ρ‚ΠΎΡ‡ΠΊΠΈ, Π½Π°ΠΌ сначала Π½ΡƒΠΆΠ½ΠΎ ΠΎΠΏΡ€Π΅Π΄Π΅Π»ΠΈΡ‚ΡŒ ΠΊΠΎΠΎΡ€Π΄ΠΈΠ½Π°Ρ‚Ρ‹ этой Ρ‚ΠΎΡ‡ΠΊΠΈ.

        ΠŸΡ€Π΅Π΄ΠΏΠΎΠ»ΠΎΠΆΠΈΠΌ, Ρƒ нас Π΅ΡΡ‚ΡŒ Π΄Π²Π΅ Ρ‚ΠΎΡ‡ΠΊΠΈ, M ΠΈ N, Π³Π΄Π΅ M = (x1, y1) ΠΈ N = (x2, y2). Π”Π°Π»Π΅Π΅ ΠΌΡ‹ Ρ…ΠΎΡ‚ΠΈΠΌ Π½Π°ΠΉΡ‚ΠΈ Π²Π΅ΠΊΡ‚ΠΎΡ€ полоТСния ΠΈΠ· Ρ‚ΠΎΡ‡ΠΊΠΈ M Π² Ρ‚ΠΎΡ‡ΠΊΡƒ N, Π²Π΅ΠΊΡ‚ΠΎΡ€ MN . Π§Ρ‚ΠΎΠ±Ρ‹ ΠΎΠΏΡ€Π΅Π΄Π΅Π»ΠΈΡ‚ΡŒ этот Π²Π΅ΠΊΡ‚ΠΎΡ€ полоТСния, ΠΌΡ‹ Π²Ρ‹Ρ‡ΠΈΡ‚Π°Π΅ΠΌ ΡΠΎΠΎΡ‚Π²Π΅Ρ‚ΡΡ‚Π²ΡƒΡŽΡ‰ΠΈΠ΅ ΠΊΠΎΠΌΠΏΠΎΠ½Π΅Π½Ρ‚Ρ‹ M ΠΈΠ· N :

        MN = (x2-x1, y2-y1)

        Π€ΠΎΡ€ΠΌΡƒΠ»Π° Π²Π΅ΠΊΡ‚ΠΎΡ€Π° полоТСния

        Π˜ΡΠΏΠΎΠ»ΡŒΠ·ΡƒΡ ΠΏΡ€ΠΈΠ²Π΅Π΄Π΅Π½Π½ΡƒΡŽ Π²Ρ‹ΡˆΠ΅ ΠΈΠ½Ρ„ΠΎΡ€ΠΌΠ°Ρ†ΠΈΡŽ, ΠΌΡ‹ ΠΌΠΎΠΆΠ΅ΠΌ ΠΎΠ±ΠΎΠ±Ρ‰ΠΈΡ‚ΡŒ Ρ„ΠΎΡ€ΠΌΡƒΠ»Π°, которая Π±ΡƒΠ΄Π΅Ρ‚ ΠΎΠΏΡ€Π΅Π΄Π΅Π»ΡΡ‚ΡŒ Π²Π΅ΠΊΡ‚ΠΎΡ€ полоТСния ΠΌΠ΅ΠΆΠ΄Ρƒ двумя Ρ‚ΠΎΡ‡ΠΊΠ°ΠΌΠΈ, Ссли Π±Ρ‹ ΠΌΡ‹ Π·Π½Π°Π»ΠΈ ΠΏΠΎΠ»ΠΎΠΆΠ΅Π½ΠΈΠ΅ Ρ‚ΠΎΡ‡Π΅ΠΊ Π² плоскости xy.

        НапримСр, рассмотрим Ρ‚ΠΎΡ‡ΠΊΡƒ P с ΠΊΠΎΠΎΡ€Π΄ΠΈΠ½Π°Ρ‚Π°ΠΌΠΈ (xk, yk) Π² плоскости xy ΠΈ Π΄Ρ€ΡƒΠ³ΡƒΡŽ Ρ‚ΠΎΡ‡ΠΊΡƒ Q с ΠΊΠΎΠΎΡ€Π΄ΠΈΠ½Π°Ρ‚Π°ΠΌΠΈ (xk+1, yk+1). Π€ΠΎΡ€ΠΌΡƒΠ»Π° для опрСдСлСния Π²Π΅ΠΊΡ‚ΠΎΡ€Π° полоТСния ΠΎΡ‚ P Π΄ΠΎ Q:

        PQ = ((xk+1)-xk, (yk+1)-yk)

        ΠŸΠΎΠΌΠ½ΠΈΡ‚Π΅ Π²Π΅ΠΊΡ‚ΠΎΡ€ полоТСния PQ относится ΠΊ Π²Π΅ΠΊΡ‚ΠΎΡ€Ρƒ, ΠΊΠΎΡ‚ΠΎΡ€Ρ‹ΠΉ начинаСтся Π² Ρ‚ΠΎΡ‡ΠΊΠ΅ P ΠΈ заканчиваСтся Π² Ρ‚ΠΎΡ‡ΠΊΠ΅ Q. Аналогично, Ссли ΠΌΡ‹ Ρ…ΠΎΡ‚ΠΈΠΌ Π½Π°ΠΉΡ‚ΠΈ Π²Π΅ΠΊΡ‚ΠΎΡ€ полоТСния ΠΈΠ· Ρ‚ΠΎΡ‡ΠΊΠΈ Q Π² Ρ‚ΠΎΡ‡ΠΊΡƒ P, ΠΌΡ‹ ΠΌΠΎΠΆΠ΅ΠΌ Π½Π°ΠΏΠΈΡΠ°Ρ‚ΡŒ:

        QP = (xk – (xk+1), yk – (yk+1))

        ΠŸΡ€ΠΈΠΌΠ΅Ρ€Ρ‹

        Π’ этом Ρ€Π°Π·Π΄Π΅Π»Π΅ ΠΌΡ‹ обсудим Π½Π΅ΠΊΠΎΡ‚ΠΎΡ€Ρ‹Π΅ ΠΏΡ€ΠΈΠΌΠ΅Ρ€Ρ‹ Π·Π°Π΄Π°Ρ‡ Π²Π΅ΠΊΡ‚ΠΎΡ€Π° полоТСния ΠΈ ΠΈΡ… ΠΏΠΎΡˆΠ°Π³ΠΎΠ²Ρ‹Π΅ Ρ€Π΅ΡˆΠ΅Π½ΠΈΡ. Π­Ρ‚ΠΎ ΠΏΠΎΠΌΠΎΠΆΠ΅Ρ‚ Π³Π»ΡƒΠ±ΠΆΠ΅ ΠΏΠΎΠ½ΡΡ‚ΡŒ Π²Π΅ΠΊΡ‚ΠΎΡ€Ρ‹ полоТСния.

        Β ΠŸΡ€ΠΈΠΌΠ΅Ρ€ 1

        Для Π΄Π²ΡƒΡ… Ρ‚ΠΎΡ‡Π΅ΠΊ A = (-4, 6) ΠΈ B = (5, 12) ΠΎΠΏΡ€Π΅Π΄Π΅Π»ΠΈΡ‚Π΅ Π²Π΅ΠΊΡ‚ΠΎΡ€ полоТСния AB. Π—Π°Ρ‚Π΅ΠΌ , Π²Ρ‹Ρ‡ΠΈΡΠ»ΡΡŽΡ‚ Π²Π΅Π»ΠΈΡ‡ΠΈΠ½Ρƒ Π²Π΅ΠΊΡ‚ΠΎΡ€Π° AB .

        РСшСниС

        ИмСя Π΄Π²Π΅ Ρ‚ΠΎΡ‡ΠΊΠΈ Π² систСмС ΠΊΠΎΠΎΡ€Π΄ΠΈΠ½Π°Ρ‚ xy, ΠΌΡ‹ ΠΌΠΎΠΆΠ΅ΠΌ ΠΈΡΠΏΠΎΠ»ΡŒΠ·ΠΎΠ²Π°Ρ‚ΡŒ ΡΠ»Π΅Π΄ΡƒΡŽΡ‰ΡƒΡŽ Ρ„ΠΎΡ€ΠΌΡƒΠ»Ρƒ для нахоТдСния Π²Π΅ΠΊΡ‚ΠΎΡ€Π° полоТСния AB :

        AB = (x2-x1, y2-y1)

        3

        3

        x1, y1 ΠΏΡ€Π΅Π΄ΡΡ‚Π°Π²Π»ΡΡŽΡ‚ ΠΊΠΎΠΎΡ€Π΄ΠΈΠ½Π°Ρ‚Ρ‹ Ρ‚ΠΎΡ‡ΠΊΠΈ A, Π° x2, y2 ΠΏΡ€Π΅Π΄ΡΡ‚Π°Π²Π»ΡΡŽΡ‚ ΠΊΠΎΠΎΡ€Π΄ΠΈΠ½Π°Ρ‚Ρ‹ Ρ‚ΠΎΡ‡ΠΊΠΈ B.Π’Π°ΠΊΠΈΠΌ ΠΎΠ±Ρ€Π°Π·ΠΎΠΌ, просто подставив значСния Ρ‚ΠΎΡ‡Π΅ΠΊ A ΠΈ B Π² ΠΏΡ€ΠΈΠ²Π΅Π΄Π΅Π½Π½ΠΎΠ΅ Π²Ρ‹ΡˆΠ΅ ΡƒΡ€Π°Π²Π½Π΅Π½ΠΈΠ΅, ΠΌΡ‹ ΠΌΠΎΠΆΠ΅ΠΌ Π½Π°ΠΉΡ‚ΠΈ Π²Π΅ΠΊΡ‚ΠΎΡ€ полоТСния AB :

        AB = (5-(-4), 12-6)

        AB = ((5+ 4), 12-6)

        AB = (9, 6)

        Π’Π°ΠΊΠΈΠΌ ΠΎΠ±Ρ€Π°Π·ΠΎΠΌ, Π²Π΅ΠΊΡ‚ΠΎΡ€ полоТСния AB эквивалСнтСн Π²Π΅ΠΊΡ‚ΠΎΡ€Ρƒ, Π½Π°Ρ‡ΠΈΠ½Π°ΡŽΡ‰Π΅ΠΌΡƒΡΡ Π² Π½Π°Ρ‡Π°Π»Π΅ ΠΊΠΎΠΎΡ€Π΄ΠΈΠ½Π°Ρ‚ ΠΈ Π½Π°ΠΏΡ€Π°Π²Π»Π΅Π½Π½ΠΎΠΌΡƒ Π² Ρ‚ΠΎΡ‡ΠΊΡƒ Π½Π° расстоянии 9 Π΅Π΄ΠΈΠ½ΠΈΡ† ΠΎΡ‚ Π²ΠΏΡ€Π°Π²ΠΎ ΠΏΠΎ оси x ΠΈ Π½Π° 6 Π΅Π΄ΠΈΠ½ΠΈΡ† Π²Π²Π΅Ρ€Ρ… ΠΏΠΎ оси y. 2

        | АВ | = √81 + 36

        | АВ | = √117

        | АВ | = 3√13 Β 

        ΠŸΡ€ΠΈΠΌΠ΅Ρ€ 2

        ИмСя Π΄Π²Π΅ Ρ‚ΠΎΡ‡ΠΊΠΈ A = (-4, 6) ΠΈ B = (5, 12), ΠΎΠΏΡ€Π΅Π΄Π΅Π»ΠΈΡ‚Π΅ Π²Π΅ΠΊΡ‚ΠΎΡ€ полоТСния BA. Π—Π°Ρ‚Π΅ΠΌ вычислитС ΠΌΠΎΠ΄ΡƒΠ»ΡŒ Π²Π΅ΠΊΡ‚ΠΎΡ€Π° BA Β ΠΈ ΠΎΠΏΠΈΡˆΠΈΡ‚Π΅ взаимосвязь ΠΌΠ΅ΠΆΠ΄Ρƒ Π²Π΅ΠΊΡ‚ΠΎΡ€ΠΎΠΌ полоТСния AB ΠΈ Π²Π΅ΠΊΡ‚ΠΎΡ€ΠΎΠΌ полоТСния BA .

        РСшСниС

        ИмСя Π΄Π²Π΅ Ρ‚ΠΎΡ‡ΠΊΠΈ Π² систСмС ΠΊΠΎΠΎΡ€Π΄ΠΈΠ½Π°Ρ‚ xy, ΠΌΡ‹ ΠΌΠΎΠΆΠ΅ΠΌ ΠΈΡΠΏΠΎΠ»ΡŒΠ·ΠΎΠ²Π°Ρ‚ΡŒ ΡΠ»Π΅Π΄ΡƒΡŽΡ‰ΡƒΡŽ Ρ„ΠΎΡ€ΠΌΡƒΠ»Ρƒ, Ρ‡Ρ‚ΠΎΠ±Ρ‹ Π½Π°ΠΉΡ‚ΠΈ Π²Π΅ΠΊΡ‚ΠΎΡ€ полоТСния BA :

        BA = (x1-x2, y1-y2)

        3

        3

        x1, y1 ΠΏΡ€Π΅Π΄ΡΡ‚Π°Π²Π»ΡΡŽΡ‚ ΠΊΠΎΠΎΡ€Π΄ΠΈΠ½Π°Ρ‚Ρ‹ Ρ‚ΠΎΡ‡ΠΊΠΈ A, Π° x2, y2 ΠΏΡ€Π΅Π΄ΡΡ‚Π°Π²Π»ΡΡŽΡ‚ ΠΊΠΎΠΎΡ€Π΄ΠΈΠ½Π°Ρ‚Ρ‹ Ρ‚ΠΎΡ‡ΠΊΠΈ B.ΠžΠ±Ρ€Π°Ρ‚ΠΈΡ‚Π΅ Π²Π½ΠΈΠΌΠ°Π½ΠΈΠ΅, Ρ‡Ρ‚ΠΎ Π²Π΅ΠΊΡ‚ΠΎΡ€ полоТСния BA прСдставляСт собой Π²Π΅ΠΊΡ‚ΠΎΡ€, Π½Π°ΠΏΡ€Π°Π²Π»Π΅Π½Π½Ρ‹ΠΉ ΠΎΡ‚ Ρ‚ΠΎΡ‡ΠΊΠΈ B ΠΊ Ρ‚ΠΎΡ‡ΠΊΠ΅ A. Он отличаСтся ΠΎΡ‚ Π²Π΅ΠΊΡ‚ΠΎΡ€Π° полоТСния AB, , ΠΊΠΎΡ‚ΠΎΡ€Ρ‹ΠΉ Π½Π°ΠΏΡ€Π°Π²Π»Π΅Π½ ΠΎΡ‚ A ΠΊ B. Π’Π°ΠΊΠΈΠΌ ΠΎΠ±Ρ€Π°Π·ΠΎΠΌ, просто поставив значСния Ρ‚ΠΎΡ‡Π΅ΠΊ A ΠΈ B Π² ΠΏΡ€ΠΈΠ²Π΅Π΄Π΅Π½Π½ΠΎΠΌ Π²Ρ‹ΡˆΠ΅ ΡƒΡ€Π°Π²Π½Π΅Π½ΠΈΠΈ ΠΌΡ‹ ΠΌΠΎΠΆΠ΅ΠΌ Π½Π°ΠΉΡ‚ΠΈ Π²Π΅ΠΊΡ‚ΠΎΡ€ полоТСния BA:

        BA = (-4-5), 6-12)

        BA = (-9, -6)

        Π’Π°ΠΊΠΈΠΌ ΠΎΠ±Ρ€Π°Π·ΠΎΠΌ, Π²Π΅ΠΊΡ‚ΠΎΡ€ полоТСния BA эквивалСнтСн Π²Π΅ΠΊΡ‚ΠΎΡ€Ρƒ, ΠΊΠΎΡ‚ΠΎΡ€Ρ‹ΠΉ начинаСтся Π² Π½Π°Ρ‡Π°Π»Π΅ ΠΊΠΎΠΎΡ€Π΄ΠΈΠ½Π°Ρ‚ ΠΈ Π½Π°ΠΏΡ€Π°Π²Π»Π΅Π½ Π² Ρ‚ΠΎΡ‡ΠΊΡƒ Π½Π° 9 Π΅Π΄ΠΈΠ½ΠΈΡ† Π²Π»Π΅Π²ΠΎ ΠΏΠΎ оси x ΠΈ Π½Π° 6 Π΅Π΄ΠΈΠ½ΠΈΡ† Π²Π½ΠΈΠ· ΠΏΠΎ оси y. 2

        | ВА | = √81 + 36

        | ВА | = √117

        | ВА | = 3√13

        Напомним, Ρ‡Ρ‚ΠΎ Π² ΠΏΠ΅Ρ€Π²ΠΎΠΌ ΠΏΡ€ΠΈΠΌΠ΅Ρ€Π΅ ΠΌΡ‹ нашли Π²Π΅ΠΊΡ‚ΠΎΡ€ полоТСния AB для Ρ‚Π΅Ρ… ΠΆΠ΅ Ρ‚ΠΎΡ‡Π΅ΠΊ, Π° Π² этом ΠΏΡ€ΠΈΠΌΠ΅Ρ€Π΅ ΠΌΡ‹ ΠΎΠΏΡ€Π΅Π΄Π΅Π»ΠΈΠ»ΠΈ Π²Π΅ΠΊΡ‚ΠΎΡ€ полоТСния BA. Π”Π²Π° Π²Π΅ΠΊΡ‚ΠΎΡ€Π° полоТСния ΠΈΠΌΠ΅ΡŽΡ‚ ΠΎΠ΄ΠΈΠ½Π°ΠΊΠΎΠ²ΡƒΡŽ Π²Π΅Π»ΠΈΡ‡ΠΈΠ½Ρƒ. ΠŸΠΎΡΠΊΠΎΠ»ΡŒΠΊΡƒ ΠΎΠ½ΠΈ ΠΈΠΌΠ΅ΡŽΡ‚ ΠΏΡ€ΠΎΡ‚ΠΈΠ²ΠΎΠΏΠΎΠ»ΠΎΠΆΠ½Ρ‹Π΅ направлСния, ΡΠΎΠΎΡ‚Π½ΠΎΡˆΠ΅Π½ΠΈΠ΅ ΠΌΠ΅ΠΆΠ΄Ρƒ Π½ΠΈΠΌΠΈ:

        BA = -1*(9, 6)

        BA = -1* AB

        BA = – 5

        Π’Π°ΠΊΠΈΠΌ ΠΎΠ±Ρ€Π°Π·ΠΎΠΌ, Π΄Π²Π° Π²Π΅ΠΊΡ‚ΠΎΡ€Π° полоТСния ΠΏΠ°Ρ€Π°Π»Π»Π΅Π»ΡŒΠ½Ρ‹ Π΄Ρ€ΡƒΠ³ Π΄Ρ€ΡƒΠ³Ρƒ ΠΈ ΠΏΡ€ΠΎΡ‚ΠΈΠ²ΠΎΠΏΠΎΠ»ΠΎΠΆΠ½Ρ‹ Π΄Ρ€ΡƒΠ³ Π΄Ρ€ΡƒΠ³Ρƒ.Π’ΠΎ Π΅ΡΡ‚ΡŒ ΠΎΠ½ΠΈ ΡΠ²Π»ΡΡŽΡ‚ΡΡ Π½Π΅Π³Π°Ρ‚ΠΈΠ²Π°ΠΌΠΈ Π΄Ρ€ΡƒΠ³ Π΄Ρ€ΡƒΠ³Π°.

        ΠŸΡ€ΠΈΠΌΠ΅Ρ€ 3

        Учитывая, Ρ‡Ρ‚ΠΎ Π²Π΅ΠΊΡ‚ΠΎΡ€ полоТСния Ρ‚ΠΎΡ‡ΠΊΠΈ S1 Ρ€Π°Π²Π΅Π½ OS1 = (2, 3) ΠΈ Ρ‡Ρ‚ΠΎ Π²Π΅ΠΊΡ‚ΠΎΡ€ S1S2 = (-3, 6), ΠΎΠΏΡ€Π΅Π΄Π΅Π»ΠΈΡ‚Π΅ Π²Π΅ΠΊΡ‚ΠΎΡ€ полоТСния Ρ‚ΠΎΡ‡ΠΊΠΈ Ρ‚ΠΎΡ‡ΠΊΠ° S2, OS2 .

        РСшСниС

        Π‘Π½Π°Ρ‡Π°Π»Π° ΠΌΡ‹ наносим Π²Π΅ΠΊΡ‚ΠΎΡ€ OS1 с Π½Π°Ρ‡Π°Π»ΡŒΠ½ΠΎΠΉ Ρ‚ΠΎΡ‡ΠΊΠΎΠΉ Π² ​​началС ΠΊΠΎΠΎΡ€Π΄ΠΈΠ½Π°Ρ‚ (0,0) ΠΈ ΠΊΠΎΠ½Π΅Ρ‡Π½ΠΎΠΉ Ρ‚ΠΎΡ‡ΠΊΠΎΠΉ Π² ​​(2,3). ΠœΡ‹ Ρ‚Π°ΠΊΠΆΠ΅ наносим Π²Π΅ΠΊΡ‚ΠΎΡ€ OS2, , ΠΊΠΎΡ‚ΠΎΡ€Ρ‹ΠΉ начинаСтся Π² Π½Π°Ρ‡Π°Π»Π΅ ΠΊΠΎΠΎΡ€Π΄ΠΈΠ½Π°Ρ‚ ΠΈ заканчиваСтся Π² Ρ‚ΠΎΡ‡ΠΊΠ΅ S2.ΠžΠ±ΠΎΠ·Π½Π°Ρ‡ΠΈΠΌ нСизвСстноС ΠΏΠΎΠ»ΠΎΠΆΠ΅Π½ΠΈΠ΅ S2 ΠΏΡ€ΠΎΠΈΠ·Π²ΠΎΠ»ΡŒΠ½Ρ‹ΠΌΠΈ ΠΊΠΎΠΎΡ€Π΄ΠΈΠ½Π°Ρ‚Π½Ρ‹ΠΌΠΈ Ρ‚ΠΎΡ‡ΠΊΠ°ΠΌΠΈ (x,y). ΠŸΠΎΡΠΊΠΎΠ»ΡŒΠΊΡƒ ΠΌΡ‹ Π·Π½Π°Π΅ΠΌ Π²Π΅ΠΊΡ‚ΠΎΡ€ полоТСния S1S2 ΠΈ Π·Π½Π°Π΅ΠΌ, Ρ‡Ρ‚ΠΎ ΠΎΠ½ Π΄Π°Π΅Ρ‚ ΠΎΡ‚Π½ΠΎΡˆΠ΅Π½ΠΈΠ΅ ΠΌΠ΅ΠΆΠ΄Ρƒ S1 ΠΈ S2, ΠΌΡ‹ Ρ‚Π°ΠΊΠΆΠ΅ ΠΌΠΎΠΆΠ΅ΠΌ Π½Π°Ρ€ΠΈΡΠΎΠ²Π°Ρ‚ΡŒ S1S2. Π­Ρ‚ΠΎ Π½Π°ΠΏΡ€Π°Π²Π»Π΅Π½Π½Ρ‹ΠΉ Π²Π΅ΠΊΡ‚ΠΎΡ€, Π½Π°Ρ‡Π°Π»ΡŒΠ½Π°Ρ Ρ‚ΠΎΡ‡ΠΊΠ° ΠΊΠΎΡ‚ΠΎΡ€ΠΎΠ³ΠΎ находится Π² Ρ‚ΠΎΡ‡ΠΊΠ΅ S1 ΠΈ ΠΊΠΎΡ‚ΠΎΡ€Ρ‹ΠΉ Π½Π°ΠΏΡ€Π°Π²Π»Π΅Π½ Π½Π° Ρ‚Ρ€ΠΈ Π΅Π΄ΠΈΠ½ΠΈΡ†Ρ‹ Π²Π»Π΅Π²ΠΎ ΠΈ Π½Π° ΡˆΠ΅ΡΡ‚ΡŒ Π΅Π΄ΠΈΠ½ΠΈΡ† Π²Π²Π΅Ρ€Ρ…. На ΠΈΠ·ΠΎΠ±Ρ€Π°ΠΆΠ΅Π½ΠΈΠΈ Π½ΠΈΠΆΠ΅ Π²ΠΈΠ΄Π½ΠΎ, Ρ‡Ρ‚ΠΎ Ρƒ нас Π΅ΡΡ‚ΡŒ Ρ‚Ρ€Π΅ΡƒΠ³ΠΎΠ»ΡŒΠ½ΠΈΠΊ 0S1S2. Π’Π°ΠΊΠΈΠΌ ΠΎΠ±Ρ€Π°Π·ΠΎΠΌ, Ρ‚Π΅ΠΏΠ΅Ρ€ΡŒ ΠΌΡ‹ ΠΌΠΎΠΆΠ΅ΠΌ ΠΈΡΠΏΠΎΠ»ΡŒΠ·ΠΎΠ²Π°Ρ‚ΡŒ Π·Π°ΠΊΠΎΠ½ Ρ‚Ρ€Π΅ΡƒΠ³ΠΎΠ»ΡŒΠ½ΠΈΠΊΠ° (ΠΈΠ»ΠΈ ΠΏΡ€Π°Π²ΠΈΠ»ΠΎ Β«Π³ΠΎΠ»ΠΎΠ²Π° ΠΊ хвосту») слоТСния Π²Π΅ΠΊΡ‚ΠΎΡ€ΠΎΠ² для опрСдСлСния ΠΊΠΎΠΎΡ€Π΄ΠΈΠ½Π°Ρ‚ Ρ‚ΠΎΡ‡ΠΊΠΈ S2 ΡΠ»Π΅Π΄ΡƒΡŽΡ‰ΠΈΠΌ ΠΎΠ±Ρ€Π°Π·ΠΎΠΌ: S1S2 – OS1

        ΠŸΠΎΠ΄ΡΡ‚Π°Π²Π»ΡΡ Π΄Π°Π½Π½Ρ‹Π΅ значСния Π² это ΡƒΡ€Π°Π²Π½Π΅Π½ΠΈΠ΅, ΠΏΠΎΠ»ΡƒΡ‡Π°Π΅ΠΌ:

        OS2 = (-3, 6) – (2, 3)

        OS2 = (-3, 6) + ( -2, -3)

        OS2 = (-3-2, 6-3)

        OS2 = (-5, 3)

        Π’Π°ΠΊΠΈΠΌ ΠΎΠ±Ρ€Π°Π·ΠΎΠΌ, OS2 =(-5, 3) Π΅ΡΡ‚ΡŒ Π²Π΅ΠΊΡ‚ΠΎΡ€ полоТСния Ρ‚ΠΎΡ‡ΠΊΠΈ S2.

        ΠŸΡ€ΠΈΠΌΠ΅Ρ€ 4

        ИмСя Π΄Π²Π΅ Ρ‚ΠΎΡ‡ΠΊΠΈ M = (4, m) ΠΈ Q = (-n, -3), ΠΎΠΏΡ€Π΅Π΄Π΅Π»ΠΈΡ‚Π΅ Π²Π΅ΠΊΡ‚ΠΎΡ€ полоТСния QM.

        РСшСниС

        Учитывая Π΄Π²Π΅ Ρ‚ΠΎΡ‡ΠΊΠΈ Π² систСмС ΠΊΠΎΠΎΡ€Π΄ΠΈΠ½Π°Ρ‚ xy, ΠΌΡ‹ ΠΌΠΎΠΆΠ΅ΠΌ ΠΈΡΠΏΠΎΠ»ΡŒΠ·ΠΎΠ²Π°Ρ‚ΡŒ ΡΠ»Π΅Π΄ΡƒΡŽΡ‰ΡƒΡŽ Ρ„ΠΎΡ€ΠΌΡƒΠ»Ρƒ для опрСдСлСния Π²Π΅ΠΊΡ‚ΠΎΡ€Π° полоТСния Q :

        QM = (-n-4, -3-m) .

        ΠŸΠΎΡΠΊΠΎΠ»ΡŒΠΊΡƒ Π½Π°ΠΌ нСизвСстны ΠΊΠΎΠΎΡ€Π΄ΠΈΠ½Π°Ρ‚Ρ‹ QM ΠΈΠ»ΠΈ значСния n ΠΈ m, ΠΌΡ‹ Π½Π΅ ΠΌΠΎΠΆΠ΅ΠΌ ΡƒΠΏΡ€ΠΎΡΡ‚ΠΈΡ‚ΡŒ ΡƒΡ€Π°Π²Π½Π΅Π½ΠΈΠ΅.2

        | Р | = √100 + 25 + 9

        | Р | = √100 + 25 + 9

        | Р | = √134

        ΠŸΡ€ΠΈΠΌΠ΅Ρ€ 6

        Учитывая Ρ‚ΠΎΡ‡ΠΊΠΈ c = 5i + 6j +3k ΠΈ d = 2i +5j – 2k Π² ΠΎΡ€Ρ‚ΠΎΠ³ΠΎΠ½Π°Π»ΡŒΠ½ΠΎΠΉ систСмС, ΠΎΠΏΡ€Π΅Π΄Π΅Π»ΠΈΡ‚Π΅ Π²Π΅ΠΊΡ‚ΠΎΡ€ полоТСния ΠΌΠ΅ΠΆΠ΄Ρƒ этими двумя Ρ‚ΠΎΡ‡ΠΊΠ°ΠΌΠΈ, CD.

        РСшСниС

        Учитывая Π΄Π²Π΅ Ρ‚ΠΎΡ‡ΠΊΠΈ, ΠΌΡ‹ ΠΌΠΎΠΆΠ΅ΠΌ ΠΈΡΠΏΠΎΠ»ΡŒΠ·ΠΎΠ²Π°Ρ‚ΡŒ ΡΠ»Π΅Π΄ΡƒΡŽΡ‰ΡƒΡŽ Ρ„ΠΎΡ€ΠΌΡƒΠ»Ρƒ для опрСдСлСния Π²Π΅ΠΊΡ‚ΠΎΡ€Π° полоТСния CD :

        CD = (2-5, 5-5, -2-3)

        CD = (-3, 0, -5)

        CD = -3i + 0j -5k

        ΠŸΡ€Π°ΠΊΡ‚ΠΈΡ‡Π΅ΡΠΊΠΈΠ΅ вопросы

        1. ΠŸΡƒΡΡ‚ΡŒ u = (-1, 4 , 5). ΠžΠΏΡ€Π΅Π΄Π΅Π»ΠΈΡ‚Π΅ Π²Π΅ΠΊΡ‚ΠΎΡ€ полоТСния, прСдставлСнный UV .
        2. ΠŸΡƒΡΡ‚ΡŒ u = (-1, 4) ΠΈ v = (2, 5). ΠžΠΏΡ€Π΅Π΄Π΅Π»ΠΈΡ‚Π΅ Π²Π΅ΠΊΡ‚ΠΎΡ€ полоТСния, прСдставлСнный VU .
        3. ΠŸΡƒΡΡ‚ΡŒ v = (3, 5) ΠΈ VM = (-6, 3). НайдитС Π²Π΅ΠΊΡ‚ΠΎΡ€ полоТСния Ρ‚ΠΎΡ‡ΠΊΠΈ m.
        4. Учитывая b = (3,2,5), ΠΎΠΏΡ€Π΅Π΄Π΅Π»ΠΈΡ‚Π΅ Π΅Π³ΠΎ Π²Π΅ΠΊΡ‚ΠΎΡ€ полоТСния, R. Π—Π°Ρ‚Π΅ΠΌ Π½Π°ΠΉΠ΄ΠΈΡ‚Π΅ Π΄Π»ΠΈΠ½Ρƒ Π²Π΅ΠΊΡ‚ΠΎΡ€Π°
        5. ΠŸΡƒΡΡ‚ΡŒ Π²Π΅ΠΊΡ‚ΠΎΡ€ AB начинаСтся Π² a = (1, 2) ΠΈ заканчиваСтся Π² Π± = (2, 3). ΠžΠΏΡ€Π΅Π΄Π΅Π»ΠΈΡ‚Π΅ Π΅Π³ΠΎ Π²Π΅ΠΊΡ‚ΠΎΡ€ полоТСния ΠΈ Π΅Π³ΠΎ Π΄Π»ΠΈΠ½Ρƒ.
        6. ΠŸΡƒΡΡ‚ΡŒ Π²Π΅ΠΊΡ‚ΠΎΡ€ OB начинаСтся Π² Ρ‚ΠΎΡ‡ΠΊΠ΅ o = (0,0) ΠΈ заканчиваСтся Π² Ρ‚ΠΎΡ‡ΠΊΠ΅ b = (-2, 6). ΠžΠΏΡ€Π΅Π΄Π΅Π»ΠΈΡ‚Π΅ Π²Π΅ΠΊΡ‚ΠΎΡ€ Π΅Π³ΠΎ полоТСния.

        ΠžΡ‚Π²Π΅Ρ‚Ρ‹

        1. Π£Π€ = (3,1). НаправлСниС UV β€” Π½Π° 3 Π΅Π΄ΠΈΠ½ΠΈΡ†Ρ‹ Π²ΠΏΡ€Π°Π²ΠΎ ΠΏΠΎ оси X ΠΈ Π½Π° 1 Π΅Π΄ΠΈΠ½ΠΈΡ†Ρƒ Π²Π²Π΅Ρ€Ρ….
        2. Π’Π£ = (-3,-1). НаправлСниС Π’Π£ Π½Π° 3 Π΅Π΄ΠΈΠ½ΠΈΡ†Ρ‹ Π²Π»Π΅Π²ΠΎ ΠΏΠΎ оси абсцисс ΠΈ Π½Π° 1 Π΅Π΄ΠΈΠ½ΠΈΡ†Ρƒ Π²Π½ΠΈΠ·. Π”Π²Π° Π²Π΅ΠΊΡ‚ΠΎΡ€Π° Π£Π€ ΠΈ Π’Π£, ΠΏΡ€ΠΎΡ‚ΠΈΠ²ΠΎΠΏΠΎΠ»ΠΎΠΆΠ½Ρ‹ ΠΏΠΎ Π½Π°ΠΏΡ€Π°Π²Π»Π΅Π½ΠΈΡŽ. 2)) для расчСта Π΄Π»ΠΈΠ½Ρ‹, радиуса Π²Π΅ΠΊΡ‚ΠΎΡ€Π° эллипса ΠΎΡ‚ Ρ†Π΅Π½Ρ‚Ρ€Π° Π² Π·Π°Π΄Π°Π½Π½ΠΎΠΌ Π½Π°ΠΏΡ€Π°Π²Π»Π΅Π½ΠΈΠΈ – это расстояниС ΠΌΠ΅ΠΆΠ΄Ρƒ Ρ†Π΅Π½Ρ‚Ρ€ΠΎΠΌ ΠΈ Ρ‚ΠΎΡ‡ΠΊΠΎΠΉ, Ρ‡ΡŒΡ линия, ΡΠΎΠ΅Π΄ΠΈΠ½ΡΡŽΡ‰Π°Ρ Ρ†Π΅Π½Ρ‚Ρ€ ΠΈ эту Ρ‚ΠΎΡ‡ΠΊΡƒ составляСт ΡƒΠ³ΠΎΠ» Ρ‚Π΅Ρ‚Π°.Π”Π»ΠΈΠ½Π° обозначаСтся символом L .

          Как Ρ€Π°ΡΡΡ‡ΠΈΡ‚Π°Ρ‚ΡŒ радиус-Π²Π΅ΠΊΡ‚ΠΎΡ€ эллипса ΠΎΡ‚ Ρ†Π΅Π½Ρ‚Ρ€Π° Π² Π·Π°Π΄Π°Π½Π½ΠΎΠΌ Π½Π°ΠΏΡ€Π°Π²Π»Π΅Π½ΠΈΠΈ с ΠΏΠΎΠΌΠΎΡ‰ΡŒΡŽ этого ΠΎΠ½Π»Π°ΠΉΠ½-ΠΊΠ°Π»ΡŒΠΊΡƒΠ»ΡΡ‚ΠΎΡ€Π°? Π§Ρ‚ΠΎΠ±Ρ‹ ΠΈΡΠΏΠΎΠ»ΡŒΠ·ΠΎΠ²Π°Ρ‚ΡŒ этот ΠΎΠ½Π»Π°ΠΉΠ½-ΠΊΠ°Π»ΡŒΠΊΡƒΠ»ΡΡ‚ΠΎΡ€ для радиуса-Π²Π΅ΠΊΡ‚ΠΎΡ€Π° эллипса ΠΎΡ‚ Ρ†Π΅Π½Ρ‚Ρ€Π° Π² Π·Π°Π΄Π°Π½Π½ΠΎΠΌ Π½Π°ΠΏΡ€Π°Π²Π»Π΅Π½ΠΈΠΈ, Π²Π²Π΅Π΄ΠΈΡ‚Π΅ Π±ΠΎΠ»ΡŒΡˆΡƒΡŽ ΠΏΠΎΠ»ΡƒΠΎΡΡŒ (a) , ΠΌΠ°Π»ΡƒΡŽ ΠΏΠΎΠ»ΡƒΠΎΡΡŒ (b) ΠΈ ΡƒΠ³ΠΎΠ» (Ξ±) ΠΈ Π½Π°ΠΆΠΌΠΈΡ‚Π΅ ΠΊΠ½ΠΎΠΏΠΊΡƒ расчСта. Π’ΠΎΡ‚ ΠΊΠ°ΠΊ вычислСниС Π²Π΅ΠΊΡ‚ΠΎΡ€Π° радиуса эллипса ΠΎΡ‚ Ρ†Π΅Π½Ρ‚Ρ€Π° Π² Π·Π°Π΄Π°Π½Π½ΠΎΠΌ Π½Π°ΠΏΡ€Π°Π²Π»Π΅Π½ΠΈΠΈ ΠΌΠΎΠΆΠ½ΠΎ ΠΎΠ±ΡŠΡΡΠ½ΠΈΡ‚ΡŒ с Π·Π°Π΄Π°Π½Π½Ρ‹ΠΌΠΈ Π²Ρ…ΠΎΠ΄Π½Ρ‹ΠΌΠΈ значСниями -> 0.2)) .

          сфСр, ΠΊΠ°ΡΠ°Ρ‚Π΅Π»ΡŒΠ½Ρ‹Ρ… плоскостСй Π² 3-Ρ… пространствах

          сфСр, ΠΊΠ°ΡΠ°Ρ‚Π΅Π»ΡŒΠ½Ρ‹Ρ… плоскостСй Π² 3-Ρ… пространствах
          Π‘Π€Π•Π Π« И ΠšΠΠ‘ΠΠ’Π•Π›Π¬ΠΠ«Π• ΠŸΠ›ΠžΠ‘ΠšΠžΠ‘Π’Π˜ Π’ Π’Π Π•Π₯ΠŸΠ ΠžΠ‘Π’Π ΠΠΠ‘Π’Π’Π•

          ΠŸΡ€ΠΈΠΌΠ΅Ρ‡Π°Π½ΠΈΡ ΠΈ ΠΏΡ€ΠΈΠΌΠ΅Ρ€Ρ‹

          Π’Π΅ΠΌΡ‹ этого ΡƒΡ€ΠΎΠΊΠ°

          1) УравнСния сфСр Π² Ρ‚Ρ€Π΅Ρ…ΠΌΠ΅Ρ€Π½ΠΎΠΌ пространствС

          2) УравнСния ΠΊΠ°ΡΠ°Ρ‚Π΅Π»ΡŒΠ½Ρ‹Ρ… плоскостСй ΠΊ Π‘Ρ„Π΅Ρ€Π°ΠΌ Π² Ρ‚Ρ€Π΅Ρ…ΠΌΠ΅Ρ€Π½ΠΎΠΌ пространствС

          .

          УравнСния сфСр Π² Ρ‚Ρ€Π΅Ρ…ΠΌΠ΅Ρ€Π½ΠΎΠΌ пространствС

          Π‘Ρ„Π΅Ρ€Π° β€” это просто Ρ‚Ρ€Π΅Ρ…ΠΌΠ΅Ρ€Π½Ρ‹ΠΉ ΠΊΡ€ΡƒΠ³, поэтому ΠΎΠ½ опрСдСляСтся ΠΊΠ°ΠΊ ΠΊΡ€ΡƒΠ³
          ΠΏΠΎ Ρ†Π΅Π½Ρ‚Ρ€Π°Π»ΡŒΠ½ΠΎΠΉ Ρ‚ΠΎΡ‡ΠΊΠ΅
          ΠΈ радиусу , ΠΏΠΎΡΠΊΠΎΠ»ΡŒΠΊΡƒ сфСра являСтся гСомСтричСским мСстом всСх Ρ‚ΠΎΡ‡Π΅ΠΊ
          это Ρ‚ΠΎΡ‡Π½ΠΎΠ΅ расстояниС радиуса ΠΎΡ‚ Ρ†Π΅Π½Ρ‚Ρ€Π°Π»ΡŒΠ½ΠΎΠΉ Ρ‚ΠΎΡ‡ΠΊΠΈ.

          .

          Бтандартная Ρ„ΠΎΡ€ΠΌΠ° уравнСния сфСры

          Учитывая Ρ†Π΅Π½Ρ‚Ρ€ ( x o , y o , z o ) ΠΈ радиус = r 91 сфСры ΠΊΠ°ΠΊ ΡƒΡ‚Π²Π΅Ρ€ΠΆΠ΄Π΅Π½ΠΈΠ΅ ΠΎ ΠΊΠ²Π°Π΄Ρ€Π°Ρ‚Π΅ радиуса, Ρ‚Π°ΠΊ ΠΊΠ°ΠΊ ΠΌΡ‹ Π½Π΅ Ρ…ΠΎΡ‚ΠΈΠΌ
          ΠΈΠΌΠ΅Ρ‚ΡŒ Π΄Π΅Π»ΠΎ с этим большим старым ΡƒΡ€ΠΎΠ΄Π»ΠΈΠ²Ρ‹ΠΌ ΠΊΠ²Π°Π΄Ρ€Π°Ρ‚Π½Ρ‹ΠΌ ΠΊΠΎΡ€Π½Π΅ΠΌ Π² Ρ„ΠΎΡ€ΠΌΡƒΠ»Π΅ расстояния.

          Π’ΠΎΠ³Π΄Π° ΡƒΡ€Π°Π²Π½Π΅Π½ΠΈΠ΅ сфСры Π±ΡƒΠ΄Π΅Ρ‚ Ρ‚Π°ΠΊΠΈΠΌ:

          ( Ρ… – Ρ… ΠΈΠ»ΠΈ ) Β² + ( Ρƒ – Ρƒ ΠΈΠ»ΠΈ ) Β² + ( z – z ΠΈΠ»ΠΈ ) Β² =

          94

          , Π² ΠΊΠΎΡ‚ΠΎΡ€ΠΎΠΌ говорится, Ρ‡Ρ‚ΠΎ ΠΊΠ²Π°Π΄Ρ€Π°Ρ‚ расстояния ΠΎΡ‚ любой Ρ‚ΠΎΡ‡ΠΊΠΈ
          Π½Π° сфСрС ΠΊ Ρ†Π΅Π½Ρ‚Ρ€Ρƒ Ρ€Π°Π²Π½ΠΎ ΠΊΠ²Π°Π΄Ρ€Π°Ρ‚Ρƒ радиуса .

          .

          ΠŸΡ€ΠΈΠΌΠ΅Ρ€ 1: ΠΠ°ΠΏΠΈΡˆΠΈΡ‚Π΅ ΡƒΡ€Π°Π²Π½Π΅Π½ΠΈΠ΅ сфСры с Ρ†Π΅Π½Ρ‚Ρ€ΠΎΠΌ (-1, 3, 6) ΠΈ радиусом = 5.

          Ρ€Π΅ΡˆΠ΅Π½ΠΈΠ΅: ( Ρ… + 1 ) Β² + ( Ρƒ – 3) Β² + ( z – 6) Β² = 25.

          ΠΏΡ€ΠΈΠΌΠ΅Ρ€ 2: Π—Π°ΠΏΠΈΡˆΠΈΡ‚Π΅ ΡƒΡ€Π°Π²Π½Π΅Π½ΠΈΠ΅ сфСры с Ρ†Π΅Π½Ρ‚Ρ€ΠΎΠΌ (- 4, 3, 7) Ρ‡Π΅Ρ€Π΅Π· Π½Π°Ρ‡Π°Π»ΠΎ ΠΊΠΎΠΎΡ€Π΄ΠΈΠ½Π°Ρ‚.

          Ρ€Π΅ΡˆΠ΅Π½ΠΈΠ΅: сначала ΠΌΡ‹ Π΄ΠΎΠ»ΠΆΠ½Ρ‹ Π½Π°ΠΉΡ‚ΠΈ ΠΊΠ²Π°Π΄Ρ€Π°Ρ‚ радиуса ΠΊΠΎΡ‚ΠΎΡ€Ρ‹ΠΉ являСтся расстояниСм
          ΠΌΠ΅ΠΆΠ΄Ρƒ Ρ†Π΅Π½Ρ‚Ρ€Π°Π»ΡŒΠ½ΠΎΠΉ Ρ‚ΠΎΡ‡ΠΊΠΎΠΉ ΠΈ Π½Π°Ρ‡Π°Π»ΠΎΠΌ ΠΊΠΎΠΎΡ€Π΄ΠΈΠ½Π°Ρ‚.

          rΒ² = 16 + 9 + 49 = 74 с использованиСм Ρ„ΠΎΡ€ΠΌΡƒΠ»Ρ‹ расстояния (которая Π½Π° самом Π΄Π΅Π»Π΅ являСтся пифагорСйской).
          поэтому ΡƒΡ€Π°Π²Π½Π΅Π½ΠΈΠ΅ сфСры ( x + 4 ) Β² + ( y – 3) Β² + ( z – 7) Β² = 74.

          .

          ΠžΠ±Ρ‰Π°Ρ Ρ„ΠΎΡ€ΠΌΠ° уравнСния сфСры

          Π§Ρ‚ΠΎΠ±Ρ‹ ΠΏΠΎΠ»ΡƒΡ‡ΠΈΡ‚ΡŒ ΠΎΠ±Ρ‰ΡƒΡŽ Ρ„ΠΎΡ€ΠΌΡƒ, ΠΌΡ‹ просто Ρ€Π°ΡΡˆΠΈΡ€ΡΠ΅ΠΌ ΠΎΠΏΠ΅Ρ€Π°Ρ‚ΠΎΡ€ стандартной Ρ„ΠΎΡ€ΠΌΡ‹, Ρ‡Ρ‚ΠΎΠ±Ρ‹ ΠΏΠΎΠ»ΡƒΡ‡ΠΈΡ‚ΡŒ

          xΒ² + yΒ² + zΒ² – 2 x O x 2 y O Y 2 Z O Z + [( x o ) Β² + ( y ΠΎ )Β² + ( z ΠΎ )Β² – rΒ² ] = 0.

          Π’Π΅ΠΏΠ΅Ρ€ΡŒ ΠΌΡ‹ собираСм константы ( x O ) Β² + ( y O ) Β² + ( Z O ) Β² – RΒ² ΠΈ ΡƒΡΡ‚Π°Π½ΠΎΠ²ΠΈΡ‚ΡŒ ΠΈΡ… = D
          ΠšΠΎΡΡ„Ρ„ΠΈΡ†ΠΈΠ΅Π½Ρ‚Ρ‹ x, y ΠΈ z , Ρ€Π°Π²Π½Ρ‹Π΅ -2 (ΠΊΠΎΠΎΡ€Π΄ΠΈΠ½Π°Ρ‚Ρ‹ Ρ†Π΅Π½Ρ‚Ρ€Π°), Π½Π°Π·Ρ‹Π²Π°ΡŽΡ‚ΡΡ a, b ΠΈ c .
          Π’Π΅ΠΏΠ΅Ρ€ΡŒ ΡƒΡ€Π°Π²Π½Π΅Π½ΠΈΠ΅ становится

          xΒ² + yΒ² + zΒ² + a x + by + c z + d = 0

          Π˜Ρ‚Π°ΠΊ, ΠΊΠΎΠΎΡ€Π΄ΠΈΠ½Π°Ρ‚Ρ‹ Ρ†Π΅Π½Ρ‚Ρ€Π° ΠΈΠ· ΠΎΠ±Ρ‰Π΅Π³ΠΎ Π²ΠΈΠ΄Π°, Ρ€Π°Π²Π½Ρ‹ ( -Β½ a, -Β½ b, -Β½ c )
          ΠΈ ΠΊΠΎΠ³Π΄Π° Ρƒ нас Π΅ΡΡ‚ΡŒ Ρ†Π΅Π½Ρ‚Ρ€, ΠΌΡ‹ Π½Π°Ρ…ΠΎΠ΄ΠΈΠΌ радиус, ΠΈΡΠΏΠΎΠ»ΡŒΠ·ΡƒΡ d = ( x o )Β² + ( y o )Β² + ( z o )Β² 911

          ΠŸΡ€ΠΈΠΌΠ΅Ρ‡Π°Π½ΠΈΠ΅: коэффициСнты xΒ², yΒ² ΠΈ zΒ² Π΄ΠΎΠ»ΠΆΠ½Ρ‹ = 1 , Ρ‡Ρ‚ΠΎΠ±Ρ‹ это Ρ€Π°Π±ΠΎΡ‚Π°Π»ΠΎ.Если ΠΎΠ½ΠΈ Π½Π΅ = 1,
          Ρ€Π°Π·Π΄Π΅Π»ΠΈΡ‚Π΅ ΡƒΡ€Π°Π²Π½Π΅Π½ΠΈΠ΅ Π½Π° Ρ‚ΠΎ, Ρ‡Ρ‚ΠΎ ΠΎΠ½ΠΈ Π΅ΡΡ‚ΡŒ, Π·Π°Ρ‚Π΅ΠΌ ΠΈΡΠΏΠΎΠ»ΡŒΠ·ΡƒΠΉΡ‚Π΅ этот ΠΏΠΎΠ΄Ρ…ΠΎΠ΄.

          .

          ΠΏΡ€ΠΈΠΌΠ΅Ρ€ 3: Π½Π°ΠΉΡ‚ΠΈ Ρ†Π΅Π½Ρ‚Ρ€ ΠΈ радиус сфСры

          Ρ€Π΅ΡˆΠ΅Π½ΠΈΠ΅: Ρ†Π΅Π½Ρ‚Ρ€ Ρ€Π°Π²Π΅Π½ ( -Β½ Π°, -Β½ b, -Β½ c ) = ( -1, 4, -6 )
          Π’Π°ΠΊ ΠΊΠ°ΠΊ d = 69 = ( -1 )Β² + ( 4 )Β² + ( -6 )Β² – r Β² Π½Π°Ρ…ΠΎΠ΄ΠΈΠΌ r ΠΈ ΠΏΠΎΠ»ΡƒΡ‡Π°Π΅ΠΌ r = 4 941
          Бтандартная Ρ„ΠΎΡ€ΠΌΠ° уравнСния этой сфСры: ( x + 1) Β² + ( y – 4) Β² + ( z + 6) Β² = 16

          Если Π±Ρ‹ ΡƒΡ€Π°Π²Π½Π΅Π½ΠΈΠ΅ ΠΏΡ€ΠΈΠΌΠ΅Ρ€Π° 3 Π±Ρ‹Π»ΠΎ прСдставлСно Π² Π²ΠΈΠ΄Π΅

          3 xΒ² + 3 yΒ² + 3 zΒ² + 6 x 24 y + 36 Z +

          0 217 = 0
          ΠΌΡ‹ Π±Ρ‹ Ρ€Π°Π·Π΄Π΅Π»ΠΈΠ»ΠΈ ΡƒΡ€Π°Π²Π½Π΅Π½ΠΈΠ΅ Π½Π° 3.

          .

          Π£Ρ€Π°Π²Π½Π΅Π½ΠΈΠ΅ ΠΊΠ°ΡΠ°Ρ‚Π΅Π»ΡŒΠ½ΠΎΠΉ плоскости ΠΊ сфСрС с Π·Π°Π΄Π°Π½Π½ΠΎΠΉ Ρ‚ΠΎΡ‡ΠΊΠΎΠΉ касания

          ΠŸΠΎΡΠΊΠΎΠ»ΡŒΠΊΡƒ ΠΊΠ°ΡΠ°Ρ‚Π΅Π»ΡŒΠ½Π°Ρ ΠΏΠ»ΠΎΡΠΊΠΎΡΡ‚ΡŒ пСрпСндикулярна радиусу сфСры ΠΊ Ρ‚ΠΎΡ‡ΠΊΠ΅ касания, радиус-Π²Π΅ΠΊΡ‚ΠΎΡ€ слуТит Π½ΠΎΡ€ΠΌΠ°Π»ΡŒΡŽ для ΠΊΠ°ΡΠ°Ρ‚Π΅Π»ΡŒΠ½ΠΎΠΉ плоскости.Как Ρ‚ΠΎΠ»ΡŒΠΊΠΎ ΠΌΡ‹ ΡƒΠ·Π½Π°Π΅ΠΌ Ρ‚ΠΎΡ‡ΠΊΡƒ ΠΊΠΎΠ½Ρ‚Π°ΠΊΡ‚Π° ΠΈ ΠΊΠΎΠΎΡ€Π΄ΠΈΠ½Π°Ρ‚Ρ‹ Ρ†Π΅Π½Ρ‚Ρ€Π° сфСры, Ρƒ нас Π΅ΡΡ‚ΡŒ Π²Π΅ΠΊΡ‚ΠΎΡ€ Π½ΠΎΡ€ΠΌΠ°Π»ΠΈ ΠΈ Ρ‚ΠΎΡ‡ΠΊΠ° Π½Π° плоскости, поэтому ΠΌΡ‹ ΠΌΠΎΠΆΠ΅ΠΌ Π½Π°ΠΉΡ‚ΠΈ Π΅Π΅ ΡƒΡ€Π°Π²Π½Π΅Π½ΠΈΠ΅.

          .

          ΠŸΡ€ΠΈΠΌΠ΅Ρ€ 4: ΠΠ°ΠΏΠΈΡˆΠΈΡ‚Π΅ ΡƒΡ€Π°Π²Π½Π΅Π½ΠΈΠ΅ ΠΊΠ°ΡΠ°Ρ‚Π΅Π»ΡŒΠ½ΠΎΠΉ плоскости ΠΊ сфСрС
          с ΡƒΡ€Π°Π²Π½Π΅Π½ΠΈΠ΅ΠΌ ( Ρ… + 1) Β² + ( Ρƒ – 4) Β² + ( z + 6) Β² = 16 Π² Ρ‚ΠΎΡ‡ΠΊΠ΅ P (- 4, 4, – 10).

          Ρ€Π΅ΡˆΠ΅Π½ΠΈΠ΅: Π¦Π΅Π½Ρ‚Ρ€ C Ρ€Π°Π²Π΅Π½ ( -1, 4, -6 ).Радиус-Π²Π΅ΠΊΡ‚ΠΎΡ€ ΠΈΠ»ΠΈ Π½ΠΎΡ€ΠΌΠ°Π»ΡŒ CP = .

          Π’Π΅ΠΊΡ‚ΠΎΡ€ Π² искомой плоскости Ρ€Π°Π²Π΅Π½ v = . ΠŸΠΎΡΠΊΠΎΠ»ΡŒΠΊΡƒ Π½ΠΎΡ€ΠΌΠ°Π»ΡŒ β€” это ΠΏΠ»ΠΎΡΠΊΠΎΡΡ‚ΡŒ z, n $ v = 0 .

          Π˜Ρ‚Π°ΠΊ,

          Π£Ρ€Π°Π²Π½Π΅Π½ΠΈΠ΅ ΠΊΠ°ΡΠ°Ρ‚Π΅Π»ΡŒΠ½ΠΎΠΉ плоскости: – 3x – 4z – 52 = 0 .

          Π‘Π»Π΅Π΄ΠΎΠ²Π°Ρ‚Π΅Π»ΡŒΠ½ΠΎ, Ρ‡Ρ‚ΠΎΠ±Ρ‹ Π½Π°ΠΉΡ‚ΠΈ ΡƒΡ€Π°Π²Π½Π΅Π½ΠΈΠ΅ ΠΊΠ°ΡΠ°Ρ‚Π΅Π»ΡŒΠ½ΠΎΠΉ плоскости ΠΊ Π·Π°Π΄Π°Π½Π½ΠΎΠΉ сфСрС, Π½Π°Π΄ΠΎ ΠΏΠΎΡΡ‚Π°Π²ΠΈΡ‚ΡŒ Ρ‚ΠΎΡ‡ΠΊΠΈ Π½Π°Π΄ радиус-Π²Π΅ΠΊΡ‚ΠΎΡ€ΠΎΠΌ любого Π²Π΅ΠΊΡ‚ΠΎΡ€Π° плоскости, ΠΏΡ€ΠΈΡ€Π°Π²Π½ΡΡ‚ΡŒ Π΅Π³ΠΎ ΠΊ Π½ΡƒΠ»ΡŽ. ΠœΡ‹ просто пишСм ΡƒΡ€Π°Π²Π½Π΅Π½ΠΈΠ΅ плоскости Ρ‡Π΅Ρ€Π΅Π· Ρ‚ΠΎΡ‡ΠΊΡƒ P с Π²Π΅ΠΊΡ‚ΠΎΡ€ΠΎΠΌ Π½ΠΎΡ€ΠΌΠ°Π»ΠΈ, Ρ€Π°Π²Π½Ρ‹ΠΌ Π²Π΅ΠΊΡ‚ΠΎΡ€Ρƒ, ΡΠΎΠ΅Π΄ΠΈΠ½ΡΡŽΡ‰Π΅ΠΌΡƒ Ρ†Π΅Π½Ρ‚Ρ€ сфСры с Ρ‚ΠΎΡ‡ΠΊΠΎΠΉ касания.

          .

          ΠŸΡ€Π°ΠΊΡ‚ΠΈΠΊΠ°

          1/ Π°) НайдитС ΡƒΡ€Π°Π²Π½Π΅Π½ΠΈΠ΅ сфСры с Π΄ΠΈΠ°ΠΌΠ΅Ρ‚Ρ€Π°ΠΌΠΈ Π² ΠΊΠΎΠ½Ρ†Π°Ρ… А (2, 3, 1) ΠΈ Π’ (-2, 1, 3).

          Π±) НайдитС ΡƒΡ€Π°Π²Π½Π΅Π½ΠΈΠ΅ ΠΊΠ°ΡΠ°Ρ‚Π΅Π»ΡŒΠ½ΠΎΠΉ ΠΊ этой сфСрС Π² Ρ‚ΠΎΡ‡ΠΊΠ΅ Π’.

          .

          2/ НайдитС Ρ†Π΅Π½Ρ‚Ρ€ ΠΈ радиус этих сфСр:

          Π°) xΒ² + yΒ² + zΒ² + 4x – 6z = 0 Π±) Ρ…Β² + ΡƒΒ² + zΒ² – 2Ρ… + 4Ρƒ = 0 Π²) xΒ² + yΒ² + zΒ² + 6x + 2z + 7 = 0

          .

          3/ ΠΠ°ΠΏΠΈΡˆΠΈΡ‚Π΅ уравнСния этих сфСр: (C – Ρ†Π΅Π½Ρ‚Ρ€Π°Π»ΡŒΠ½Π°Ρ Ρ‚ΠΎΡ‡ΠΊΠ°)

          Π°) Π‘ (4, 3, 12) Ρ‡Π΅Ρ€Π΅Π· Π½Π°Ρ‡Π°Π»ΠΎ ΠΊΠΎΠΎΡ€Π΄ΠΈΠ½Π°Ρ‚. Π±) Π‘ (4, 3, 12) ΠΏΠΎ (0, 0, 2).
          c) C (1, 2, 3) ΠΊΠ°ΡΠ°Ρ‚Π΅Π»ΡŒΠ½ΠΎΠΉ ΠΊ плоскости xy . d) C (1, – 2, 1) ΠΏΠΎ ΠΊΠ°ΡΠ°Ρ‚Π΅Π»ΡŒΠ½ΠΎΠΉ ΠΊ оси z .
          e) радиус = 2, Ρ†Π΅Π½Ρ‚Ρ€ Π² ΠΎΠΊΡ‚Π°Π½Ρ‚Π΅-1, ΠΊΠ°ΡΠ°Ρ‚Π΅Π»ΡŒΠ½Π°Ρ ΠΊΠΎ всСм Ρ‚Ρ€Π΅ΠΌ ΠΊΠΎΠΎΡ€Π΄ΠΈΠ½Π°Ρ‚Π½Ρ‹ΠΌ плоскостям.

          .

          4/ НайдитС ΡƒΡ€Π°Π²Π½Π΅Π½ΠΈΠ΅ ΠΊΠ°ΡΠ°Ρ‚Π΅Π»ΡŒΠ½ΠΎΠΉ плоскости ΠΊ xΒ² + yΒ² + zΒ² – 2x – 4y -4 = 0 Π² P (0, 0, 2).

          .

          .

          РСшСния

          1/ Π°) C (0, 2, 2), радиус = , поэтому ( x – 0 )Β² + ( y – 2 )Β² + ( z – 2 )Β² = 6

          Π±) ΠšΠ°ΡΠ°Ρ‚Π΅Π»ΡŒΠ½Π°Ρ ΠΏΠ»ΠΎΡΠΊΠΎΡΡ‚ΡŒ ΠΈΠΌΠ΅Π΅Ρ‚ Π½ΠΎΡ€ΠΌΠ°Π»ΡŒ n = , Π° Ρ‚ΠΎΡ‡ΠΊΠ° касания (- 2, 1, 3), поэтому:

          Π£Ρ€Π°Π²Π½Π΅Π½ΠΈΠ΅ плоскости 2( x + 2) + ( y – 1) – ( z – 3) = 0

          2Ρ… + Ρƒ – Π³ + 6 = 0

          .

          2/

          Π°) Π‘ (-2, 0, 3), радиус = Π±) Π‘ (1, – 1, 0), радиус = Π²) Π‘ (0, – 3, – 1), радиус =

          .

          3/

          Π°) ( x – 4 ) Β² + ( y – 3) Β² + ( z – 12 ) Β² = 169 b) ( x – 1 ) Β² + ( y – 2) Β² + z Β² = 9
          c) ( x – 1 ) Β² + ( 0 1 ) Β² + ( + ( z – 3 ) Β² = 9 d) ( x – 1 ) Β² + ( y + 2) Β² + ( z – 1 ) Β² = 5
          e) C Ρ€Π°Π²Π½ΠΎ (2, 2, 2) u ( x – 2 ) Β² + ( Π³ – 2) Β² + ( z – 2 ) Β² = 4

          .

          4/ Π¦Π΅Π½Ρ‚Ρ€ C ( 1, 2, 0 ). Радиус/Π½ΠΎΡ€ΠΌΠ°Π»ΡŒΠ½Ρ‹ΠΉ Π²Π΅ΠΊΡ‚ΠΎΡ€ Ρ€Π°Π²Π΅Π½ CP = .

          Π›ΡŽΠ±ΠΎΠΉ Π²Π΅ΠΊΡ‚ΠΎΡ€ Π² плоскости Ρ‡Π΅Ρ€Π΅Π· (0, 0, 2) Ρ€Π°Π²Π΅Π½ = . Π Π°ΡΡΡ‚Π°Π²ΡŒΡ‚Π΅ Ρ‚ΠΎΡ‡ΠΊΠΈ ΠΈ установитС = 0

          Π˜Ρ‚Π°ΠΊ,

          Π£Ρ€Π°Π²Π½Π΅Π½ΠΈΠ΅ этой ΠΊΠ°ΡΠ°Ρ‚Π΅Π»ΡŒΠ½ΠΎΠΉ плоскости: x + 2y – 2z + 4 = 0 .

ΠžΡΡ‚Π°Π²ΠΈΡ‚ΡŒ ΠΊΠΎΠΌΠΌΠ΅Π½Ρ‚Π°Ρ€ΠΈΠΉ